You are on page 1of 75

B. Labor Organizations ● Samahan, through its authorized representative, Alfie F.

Alipio, filed an
1. Policy application for registration of its name "Samahan ng Mga Manggagawa sa
Articles 218(A)(b); (c); (d); and (g), Labor Code. Hanjin Shipyard" with the DOLE. With a total of 120 members.
● DOLE issued the corresponding certificate of registration.
A) To promote and emphasize the primacy of free collective bargaining and ● Then respondent Hanjin Heavy Industries (Hanjin), filed a petition with
negotiations, including voluntary arbitration, mediation and conciliation, as DOLE praying for the cancellation of registration of Samahan's association
modes of settling labor or industrial disputes; on the ground that its members did not fall under any of the types of workers
B) To promote free trade unionism as an instrument for the enhancement of enumerated in the second sentence of Article 243 (now 249).
democracy and the promotion of social justice and development; ● Hanjin opined that only ambulant, intermittent, itinerant, rural workers, self-
C) To foster the free and voluntary organization of a strong and united labor employed, and those without definite employers may form a workers'
movement; association.
D) To promote the enlightenment of workers concerning their rights and ● It further posited that one third (1/3) of the members of the association had
obligations as union members and as employees; definite employers and the continued existence and registration of the
G) To ensure the participation of workers in decision and policy-making association would prejudice the company's goodwill.
processes affecting their rights, duties and welfare. ● Hanjin filed a supplemental petition, adding the alternative ground that
Samahan committed a misrepresentation in connection with the list of
2. Definition members and/or voters who took part in the ratification of their constitution
Article 219(g), Labor Code. and by-laws in its application for registration. Hanjin claimed that Samahan
G. "Labor organization" means any union or association of employees which exists made it appear that its members were all qualified to become members of
in whole or in part for the purpose of collective bargaining or of dealing with the workers' association.
employers concerning terms and conditions of employment. ● DOLE-Pampanga called for a conference, wherein Samahan requested for
3. Categories a 10-day period to file a responsive pleading. No pleading, however, was
Articles 219(g); (h); (i); and (j), Labor Code. submitted. Instead, Samahan filed a motion to dismiss on April 14, 2010.
G. "Labor organization" means any union or association of employees which exists ● DOLE Regional Director Ernesto Bihis ruled in favor of Hanjin
in whole or in part for the purpose of collective bargaining or of dealing with ● Samahan failed to adduce evidence that the remaining 63 members were
employers concerning terms and conditions of employment. also employees of Hanjin
● Bureau of Labor Relations ruled in favour of Samahan
H. "Legitimate labor organization" means any labor organization duly registered with ● In the MR of Hanjin decision is affirmed but with a DIRECTIVE for
the Department of Labor and Employment, and includes any branch or local thereof. SAMAHAN to remove "HANJIN SHIPYARD" from its name.
● CA rendered its decision that Samahan as a legitimate workers' association
I. "Company union" means any labor organization whose formation, function or was contrary to the provisions of Article 243 of the Labor Code. It stressed
administration has been assisted by any act defined as unfair labor practice by this that only 57 out of the 120 members were actually working in Hanjin.
Code. ● In the preamble of Samahan's Constitution and By-laws, "KAMI, ang mga
Manggagawa sa Hanjin Shipyard" created an impression that all its
Samahan ng Manggagawa sa Hanjin Shipyard v. Bureau of Labor Relations, members were employees of HHIC.
G.R. No. 211145, October 14, 2015. ● Samahan could not register it as a legitimate worker's association because
the place where Hanjin's industry was located was not a rural area.
FACTS:

1
ISSUE: CA erred in finding that samahan cannot form a workers' association of remitting checked-off union dues to said union until February, 1987 when a group
employees in hanjin and instead should have formed a union, hence their composed of members of said union, introducing itself to be its new set of officers,
registration as a workers' association should be cancelled. went to the company and claimed that they are entitled to the remittance and custody
of such union dues. This group, headed by Manuel Gabayoyo claims that they were
HELD: The petition is partly meritorious. elected as such under the supervision of the SEC. Another group headed by
Dominica C. Nacua, claiming as the duly elected set of officers of the union in an
No misrepresentation on the part of Samahan to warrant cancellation of registration election held on 20 December 1986, filed a complaint, for and in behalf of the union,
against the Cebu Seamen's Association, Inc. (CSAI) as represented by Manuel
In this case, Samahan's registration was cancelled not because its members were Gabayoyo for the security of the aforementioned CBA, seeking such relief, among
prohibited from forming a workers' association but because they allegedly committed others, as an order restraining the respondent from acting on behalf of the union and
misrepresentation for using the phrase, "KAMI, ang mga Manggagawa sa HAN JIN directing the Aboitiz Shipping Corp. to remit the checked-off union dues for the
Shipyard." months of March and April 1987.
Mediator- Arbiter directing the Aboitiz Shipping Corporation to remit the already
Misrepresentation, as a ground for the cancellation of registration of a labor checked-off union dues to the complainant union through its officers end to continue
organization, is committed "in connection with the adoption, or ratification of the remitting any checked-off union dues until further notice.
constitution and by-laws or amendments thereto, the minutes of ratification, the list BLR affirmed Med Arb
of members who took part in the ratification of the constitution and by-laws or
amendments thereto, and those in connection with the election of officers, minutes ISSUES: who is entitled to the collection and custody of the union dues? Cebu
of the election of officers, and the list of voters,." Seamen's Association headed by Gabayoyo or Seamen's Association of the
Philippines headed by Nacua.
The records of this case reveal no deliberate or malicious intent to commit
misrepresentation on the part of Samahan. RULING: There is no doubt that the controversy between the aforesaid two
sets of officers is an intra-union dispute. Both sets of officers claim to be
Removal of the word "Hanjin Shipyard" from the association's name, however, does entitled to the release of the union dues collected by the company with whom
not infringe on Samahan's right to self-organization it had an existing CBA.

It would be misleading for the members of Samahan to use "Hanjin Shipyard" in its private respondent Dominica Nacua was elected president of the labor union, SAPI.
name as it could give the wrong impression that all of its members are employed by It had an existing CBA with Aboitiz Shipping Corporation. Before the end of the term
Hanjin. in the directive of the BLR removing the words "Hanjin Shipyard," no of private respondent Nacua, some members of the union which included Domingo
abridgement of Samahan's right to self-organization was committed. Machacon and petitioner Manuel Gabayoyo showed signs of discontentment with
the leadership of Nacua. This break-away group revived the moribund corporation
Cebu Seamen’s Association, Inc. v. Ferrer-Calleja, G.R. No. 83190, August 4, and issued an undated resolution expelling Nacua from association (pp. 58-59,
1992. Rollo). Sometime in February, 1987, it held its own election of officers supervised
FACTS: A group of deck officers and engineers on board vessels plying Cebu by the Securities and Exchange Commission. It also filed a case of estafa against
registered the same as a non-stock corporation known as Cebu Seamen's Nacua sometime in May, 1986 (p. 52, Rollo).
Association, Inc. (CSAI), with the SEC. Later, the same group registered its
association with this Bureau as a labor union known as the Seamen's Association The expulsion of Nacua from the corporation, of which she denied being a member,
of the Philippines, Incorporated (SAPI). SAPI has an existing CBA with the Aboitiz has however, not affected her membership with the labor union. In fact, in the
Corporation which will expire on 31 December 1988. The company has been elections of officers for 1987-1989, she was re-elected as the president of the labor
2
union. In this connections, We cannot agree with the contention of Gabayoyo that dismissal of the complainants herein is because their said dismissal was asked by
Nacua was already expelled from the union. Whatever acts their group had done in the USWU-ILO of which union respondent company has a valid and existing
the corporation do not bind the labor union. Moreover, Gabayoyo cannot claim collective bargaining contract with a closed-shop provision to the effect that those
leadership of the labor group by virtue of his having been elected as a president of laborers who are no longer members of good standing in the union may be
the dormant corporation CSAI. dismissed by the respondent company if their dismissal is sought by the union; that
respondent company has never committed acts of unfair labor practice against its
Under the principles of administrative law in force in this jurisdiction, decisions of employees or workers much less against the complainants herein but that it has a
administrative officers shall not be disturbed by courts, except when the former acted solemn obligation to comply with the terms and conditions of the contract; and that
without or in excess of their jurisdiction or with grave abuse of discretion. a closed-shop agreement is sanctioned under this jurisdiction for such kind of
agreement is expressly allowed under the provisions of Republic Act 875 known as
Public respondent Bureau of Labor Relations correctly ruled on the basis of the Industrial Peace Act and the dismissal of complainants is merely an exercise of
the evidence presented by the parties that SAPI, the legitimate labor union, a right allowed by said law.
registered with its office, is not the same association as CSAI, the corporation,
insofar as their rights under the Labor Code are concerned. Hence, the former ISSUES: WON the dismissal of the employees is proper under the closed-shop
and not the latter association is entitled to the release and custody of union agreement in the CBA
fees with Aboitiz Shipping and other shipping companies with whom it had an
existing CBA. RULING: The closed-shop agreement authorized under sec. 4, subsec. a(4) of the
A record check with the Labor Organizations (LOD), this Bureau, shows that Industrial Peace Act above quoted should however, apply to persons to be hired or
SAPI has submitted to it for file the list of this new set of officers, in to employees who are not yet members of any labor organization. It is inapplicable
compliance with the second paragraph of Article 242 (c) of the Labor Code. to those already in the service who are members of another union. To hold
This list sufficiently sustains the view that said officers were lawfully elected, otherwise, i.e., that the employees in a company who are members of a minority
in the absence of clear and convincing proof to the contrary. union may be compelled to disaffiliate from their union and join the majority or
contracting union, would render nugatory the right of all employees to self-
organization and to form, join or assist labor organizations of their own choosing, a
4. Rationale right guaranteed by the Industrial Peace Act) as well as by the Constitution. Nothing
Guijarno v. Court of Industrial Relations, G.R. Nos. L-28791-93, August 27, can be clearer therefore than that this Court looks with disfavor on a provision of this
1973. character being utilized as an excuse for the termination of employment.
FACTS: Three unfair labor practice cases for unlawful dismissal allegedly based on
legitimate union activity were filed against respondent Central Santos Lopez Co., Petition granted. Dec of CIR reversed, backwages to be paid by the labor union.
Inc. and respondent United Sugar Workers Union-ILO, with eight of the present Central Santos Lopez Company, Inc., would not have taken the action it did had it
petitioners as complainants in the first, 2 six of them in the second, 3 and five, in the not been for the insistence of the labor union seeking to give effect to its
third. 4 There was a consolidated hearing and a consolidated decision not only for interpretation of a closed shop provision.
convenience, but also due to there being hardly any difference as to the nature of
the alleged grievance and the defense of management. There was no question
about the expulsion from respondent labor union of the former. In view of a closed- 5. Regulation
shop provision in the then existing collective bargaining contract, respondent Central Electromat Manufacturing and Recording Corporation v. Lagunzad, G.R. No.
Santos Lopez Co., Inc. assumed it had to dismiss them. So it was noted in the 172699, July 27, 2011.
decision of the then associate Judge Joaquin M. Salvador of respondent Court. FACTS: The private respondent Nagkakaisang Samahan ng Manggagawa ng
Thus: "The respondent company, in its answer, alleged that the only reason for the Electromat-Wasto (union), a charter affiliate of the Workers Advocates for Struggle,
3
Transformation and Organization (WASTO), applied for registration with the Bureau be entitled to the rights and privileges granted by law to legitimate labor
of Labor Relations (BLR). Supporting the application were the following documents: organizations upon issuance of the certificate of registration based on the following
(1) copies of its ratified constitution and by-laws (CBL); (2) minutes of the CBL's requirements:
adoption and ratification; (3) minutes of the organizational meetings; (4) names and
addresses of the union officers; (5) list of union members; (6) list of rank-and-file (a) Fifty pesos (P50.00) registration fee;
employees in the company; (7) certification of non-existence of a collective (b) The names of its officers, their addresses, the principal address of the labor
bargaining agreement (CBA) in the company; (8) resolution of affiliation with organization, the minutes of the organizational meetings and the list of the workers
WASTO, a labor federation; (9) WASTO's resolution of acceptance; (10) Charter who participated in such meetings;
Certificate; and (11) Verification under oath. (c) The names of all its members comprising at least twenty percent (20%) of all the
employees in the bargaining unit where it seeks to operate;
The BLR thereafter issued the union a Certification of Creation of Local Chapter. (d) If the applicant union has been in existence for one or more years, copies of its
Petitioner Electromat Manufacturing and Recording Corporation (company) filed a annual financial reports; and
petition for cancellation of the union's registration certificate, for the union's failure (e) Four (4) copies of the constitution and by-laws of the applicant union, minutes of
to comply with Article 234 of the Labor Code. It argued that D.O. 40-03 is an its adoption or ratification, and the list of the members who participated in it.
unconstitutional diminution of the Labor Code's union registration requirements
under Article 234. The company contends that the enumeration of the requirements for union
registration under the law is exclusive and should not be diminished, and that the
Department of Labor and Employment (DOLE)-National Capital Region dismissed same requirements should apply to all labor unions whether they be independent
the petition. labor organizations, federations or local chapters. It adds that in making a different
rule for local chapters, D.O. 40-03 expanded or amended Article 234 of the Labor
BLR Director Hans Leo J. Cacdac affirmed the dismissal. The company thereafter Code, resulting in an invalid exercise by the DOLE of its delegated rule-making
sought relief from the CA through a petition for certiorari, contending that the BLR power.
committed grave abuse of discretion in affirming the union's registration despite its
non-compliance with the requirements for registration under Article 234 of the Labor ISSUES: whether D.O. 40-03 is a valid exercise of the rule-making power of the
Code. It assailed the validity of D.O. 40-03 which amended the rules of Book V DOLE.
(Labor Relations) of the Labor Code. It posited that the BLR should have strictly
adhered to the union registration requirements under the Labor Code, instead of RULING: Interpreting these provisions of the old rules, the Court said that by force
relying on D.O. 40-03 which it considered as an invalid amendment of the law since of law, the local or chapter of a labor federation or national union becomes a
it reduced the requirements under Article 234 of the Labor Code. It maintained that legitimate labor organization upon compliance with Section 3, Rule II, Book V of the
the BLR should not have granted the union's registration through the issuance of a Rules Implementing the Labor Code, the only requirement being the submission of
Certification of Creation of Local Chapter since the union submitted only the Charter the charter certificate to the BLR. Further, the Court noted that Section 3 omitted
Certificate issued to it by WASTO. CA affirmed the judgment. several requirements which are otherwise required for union registration.

The company points out that D.O. 40-03 delisted some of the requirements under It was this same Section 3 of the old rules that D.O. 40-03 fine-tuned when the DOLE
Article 234 of the Labor Code for the registration of a local chapter. Article 234 amended the rules on Book V of the Labor Code, thereby modifying the
states: government's implementing policy on the registration of locals or chapters of labor
federations or national unions. The company now assails this particular amendment
ART. 234. Requirements of Registration. [13] Any applicant labor organization, as an invalid exercise of the DOLE's rule-making power.
association or group of unions or workers shall acquire legal personality and shall
4
We disagree. As in the case of D.O. 9 (which introduced the above-cited Section 3 If the applicant union has been in existence for one or more years, copies of its
of the old rules) in Progressive Development, D.O. 40-03 represents an expression annual financial reports; and
of the government's implementing policy on trade unionism. It builds upon the old
rules by further simplifying the requirements for the establishment of locals or Four (4) copies of the constitution and by-laws of the applicant union, minutes of its
chapters. As in D.O. 9, we see nothing contrary to the law or the Constitution in the adoption or ratification, and the list of the members who participated in it.
adoption by the Secretary of Labor and Employment of D.O. 40-03 as this
department order is consistent with the intent of the government to encourage the Article 244. Additional requirements for federations or national unions. Subject to
affiliation of a local union with a federation or national union to enhance the local's Article 238, if the applicant for registration is a federation or a national union, it shall,
bargaining power. If changes were made at all, these were those made to recognize in addition to the requirements of the preceding Articles, submit the following:
the distinctions made in the law itself between federations and their local chapters,
and independent unions; local chapters seemingly have lesser requirements Proof of the affiliation of at least ten (10) locals or chapters, each of which must be
because they and their members are deemed to be direct members of the federation a duly recognized collective bargaining agent in the establishment or industry in
to which they are affiliated, which federations are the ones subject to the strict which it operates, supporting the registration of such applicant federation or national
registration requirements of the law. union; and

In any case, the local union in the present case has more than satisfied the The names and addresses of the companies where the locals or chapters operate
requirements the petitioner complains about. Petition denied. and the list of all the members in each company involved.

a. Registration Article 249. Equity of the incumbent. All existing federations and national unions
i. Requirements which meet the qualifications of a legitimate labor organization and none of the
Articles 240; 241; 244; and 249, Labor Code. grounds for cancellation shall continue to maintain their existing affiliates regardless
of the nature of the industry and the location of the affiliates.
Article 240. Requirements of registration. Any applicant labor organization,
association or group of unions or workers shall acquire legal personality and shall Takata (Philippines) Corporation v. Bureau of Labor Relations, G.R. No.
be entitled to the rights and privileges granted by law to legitimate labor 192676, June 4, 2014.
organizations upon issuance of the certificate of registration based on the following FACTS: On July 7, 2009, petitioner filed with the Department of Labor and
requirements. Employment (DOLE) Regional Office a Petition3 for Cancellation of the Certificate
of Union Registration of Respondent Samahang Lakas Manggagawa ng Takata
Fifty pesos (P50.00) registration fee; (SALAMAT) on the ground that the latter is guilty of misrepresentation, false
statement and fraud with respect to the number of those who participated in the
The names of its officers, their addresses, the principal address of the labor organizational meeting, the adoption and ratification of its Constitution and By-Laws,
organization, the minutes of the organizational meetings and the list of the workers and in the election of its officers. It contended that in the May 1, 2009 organizational
who participated in such meetings; meeting of respondent, only 68 attendees signed the attendance sheet, and which
number comprised only 17% of the total number of the 396 regular rank- and-file
The names of all its members comprising at least twenty percent (20%) of all the employees which respondent sought to represent, and hence, respondent failed to
employees in the bargaining unit where it seeks to operate; (As amended by comply with the 20% minimum membership requirement. Petitioner insisted that the
Executive Order No. 111, December 24, 1986) document “Pangalan ng mga Kasapi ng Unyon” bore no signatures of the alleged
119 union members; and that employees were not given sufficient information on
the documents they signed; that the document “Sama-Samang Pahayag ng
5
Pagsapi” was not submitted at the time of the filing of respondent's application for in the organizational meeting. Indeed, Article 234 (b) and (c) provide for separate
union registration; that the 119 union members were actually only 117; and, that the requirements, which must be submitted for the union's registration, and which
total number of petitioner's employees as of May 1, 2009 was 470, and not 396 as respondent did submit. Here, the total number of employees in the bargaining unit
respondent claimed. was 396, and 20% of which was about 79. Respondent submitted a document
entitled “Pangalan ng Mga Kasapi ng Unyon” showing the names of 119 employees
DOLE Regional Director, Atty. Ricardo S. Martinez, Sr., issued a Decision8 granting as union members, thus respondent sufficiently complied even beyond the 20%
the petition for cancellation of respondent's certificate of registration. minimum membership requirement. Respondent also submitted the attendance
sheet of the organizational meeting which contained the names and signatures of
BLR rendered its Decision reversing the Order of the Regional Director. Petitioner the 68 union members who attended the meeting. Considering that there are 119
failed to prove that respondent deliberately and maliciously misrepresented the union members which are more than 20% of all the employees of the bargaining
number of rank-and-file employees. unit, and since the law does not provide for the required number of members to
attend the organizational meeting, the 68 attendees which comprised at least the
CA rendered its assailed decision which denied the petition and affirmed the majority of the 119 union members would already constitute a quorum for the
decision of the BLR. meeting to proceed and to validly ratify the Constitution and By-laws of the union.
There is, therefore, no basis for petitioner to contend that grounds exist for the
ISSUES: won ca erred in finding that the application for registration of samahang cancellation of respondent's union registration. For fraud and misrepresentation to
lakas manggagawa sa takata (salamat) was compliant with the law. be grounds for cancellation of union registration under Article 239 of the Labor Code,
the nature of the fraud and misrepresentation must be grave and compelling enough
RULING: Petition should be denied. Petitioner's charge that respondent committed to vitiate the consent of a majority of union members.
misrepresentation and fraud in securing its certificate of registration is a serious
charge and must be carefully evaluated. Allegations thereof should be compounded Petitioner claims that in the list of members, there was an employee whose name
with supporting circumstances and evidence.21 We find no evidence on record to appeared twice and another employee who was merely a project employee. Such
support petitioner's accusation. could not be considered a misrepresentation in the absence of showing that
respondent deliberately did so for the purpose of increasing their union membership.
Petitioner's allegation of misrepresentation and fraud is based on its claim that In fact, even if those two names were not included in the list of union members, there
during the organizational meeting on May 1, 2009, only 68 employees attended, would still be 117 members which was still more than 20% of the 396 rank-and-file
while respondent claimed that it has 119 members as shown in the document employees.
denominated as “Pangalan ng mga Kasapi ng Unyon;” hence, respondent
misrepresented on the 20% requirement of the law as to its membership. Mariwasa Siam Ceramics, Inc. v. The Secretary of the Department of Labor and
Employment, G.R. No. 183317, December 21, 2009.
We do not agree. FACTS: On May 4, 2005, respondent Samahan Ng Mga Manggagawa Sa Mariwasa
Siam Ceramics, Inc. (SMMSC-Independent) was issued a Certificate of Registration
It does not appear in Article 234 (b) of the Labor Code that the attendees in the as a legitimate labor organization by the DOLE. Mariwasa Siam Ceramics, Inc. filed
organizational meeting must comprise 20% of the employees in the bargaining unit. a Petition for Cancellation of Union Registration against respondent, claiming that
In fact, even the Implementing Rules and Regulations of the Labor Code does not the latter violated Article 234 of the Labor Code for not complying with the 20%
so provide. It is only under Article 234 (c) that requires the names of all its members requirement, and that it committed massive fraud and misrepresentation in violation
comprising at least twenty percent (20%) of all the employees in the bargaining unit of Article 239 of the same code.
where it seeks to operate. Clearly, the 20% minimum requirement pertains to the
employees’ membership in the union and not to the list of workers who participated
6
Regional Director of DOLE IV-A granted the petition, revoking the registration of
respondent, and delisting it from the roster of active labor unions. Evidently, these affidavits were written and prepared in advance, and the pro forma
affidavits were ready to be filled out with the employees' names and signatures. It is
BLR granted respondent's appeal. worthy to note, however, that the affidavit does not mention the identity of the people
who allegedly forced and deceived the affiant into joining the union, much less the
CA denied petitioners certiorari and MR circumstances that constituted such force and deceit. Indeed, not only was this
allegation couched in very general terms and sweeping in nature, but more
ISSUES: WON CA erred that private respondent union complied with the 20% importantly, it was not supported by any evidence whatsoever.
membership requirement
On the second issue whether or not the withdrawal of 31 union members from NATU
RULING: petition should be denied. affected the petition for certification election insofar as the 30% requirement is
concerned, We reserve the Order of the respondent Director of the Bureau of Labor
The petitioner insists that respondent failed to comply with the 20% union Relations, it appearing undisputably that the 31 union members had withdrawn their
membership requirement for its registration as a legitimate labor organization support to the petition before the filing of said petition. It would be otherwise if the
because of the disaffiliation from the total number of union members of 102 withdrawal was made after the filing of the petition for it would then be presumed
employees who executed affidavits recanting their union membership. that the withdrawal was not free and voluntary. The presumption would arise that
the withdrawal was procured through duress, coercion or for valuable consideration.
The affidavits uniformly state' In other words, the distinction must be that withdrawals made before the filing of the
petition are presumed voluntary unless there is convincing proof to the contrary,
Ako, _____________, Pilipino, may sapat na gulang, regular na empleyado whereas withdrawals made after the filing of the petition are deemed involuntary.
bilang Rank & File sa Mariwasa Siam Ceramics, Inc., Bo. San Antonio, Sto.
Tomas, Batangas, matapos na makapanumpa ng naaayon sa batas ay The reason for such distinction is that if the withdrawal or retraction is made before
malaya at kusang loob na nagsasaad ng mga sumusunod: the filing of the petition, the names of employees supporting the petition are
supposed to be held secret to the opposite party. Logically, any such withdrawal or
1. Ako ay napilitan at nilinlang sa pagsapi sa Samahan ng mga retraction shows voluntariness in the absence of proof to the contrary. Moreover, it
Manggagawa sa Mariwasa Siam Ceramics, Inc. o SMMSC-Independent sa becomes apparent that such employees had not given consent to the filing of the
kabila ng aking pag-aalinlangan[;] petition, hence the subscription requirement has not been met.

2. Aking lubos na pinagsisihan ang aking pagpirma sa sipi ng samahan, at When the withdrawal or retraction is made after the petition is filed, the employees
handa ako[ng] tumalikod sa anumang kasulatan na aking nalagdaan sa who are supporting the petition become known to the opposite party since their
kadahilanan na hindi angkop sa aking pananaw ang mga mungkahi o names are attached to the petition at the time of filing. Therefore, it would not be
adhikain ng samahan. unexpected that the opposite party would use foul means for the subject employees
to withdraw their support.
SA KATUNAYAN NANG LAHAT, ako ay lumagda ng aking pangalan
ngayong ika - ____ ng ______, 2005 dito sa Lalawigan ng Batangas, Bayan In the instant case, the affidavits of recantation were executed after the identities of
ng Sto. Tomas. the union members became public, i.e., after the union filed a petition for certification
election on May 23, 2005, since the names of the members were attached to the
____________________ petition. The purported withdrawal of support for the registration of the union was
Nagsasalaysay made after the documents were submitted to the DOLE, Region IV-A. The logical
7
conclusion, therefore, following jurisprudence, is that the employees were not totally Employees Union – Pambansang Diwa ng Manggagawang Pilipino (SMPEU-
free from the employer's pressure, and so the voluntariness of the employees' PDMP), G.R. No. 171153, September 12, 2007.
execution of the affidavits becomes suspect.
FACTS
It is likewise notable that the first batch of 25 pro forma affidavits shows that the Petitioner is the incumbent bargaining agent for the bargaining unit comprised of the
affidavits were executed by the individual affiants on different dates from May 26, regular monthly-paid rank and file employees of the three divisions of San Miguel
2005 until June 3, 2005, but they were all sworn before a notary public on June 8, Corporation (SMC), namely, the San Miguel Corporate Staff Unit (SMCSU), San
2005. There was also a second set of standardized affidavits executed on different Miguel Brewing Philippines (SMBP), and the San Miguel Packaging Products
dates from May 26, 2005 until July 6, 2005. While these 77 affidavits were notarized (SMPP), in all offices and plants of SMC, including the Metal Closure and
on different dates, 56 of these were notarized on June 8, 2005, the very same date Lithography Plant in Laguna. It had been the certified bargaining agent for 20 years
when the first set of 25 was notarized. – from 1987 to 1997.

Considering that the first set of 25 affidavits was submitted to the DOLE on June 14, Respondent is registered as a chapter of Pambansang Diwa ng Manggagawang
2005, it is surprising why petitioner was able to submit the second set of affidavits Pilipino (PDMP). PDMP issued Charter Certificate No. 112 to respondent on 15 June
only on July 12, 2005. 1999. In compliance with registration requirements, respondent submitted the
requisite documents to the BLR for the purpose of acquiring legal personality.6 Upon
Accordingly, we cannot give full credence to these affidavits, which were executed submission of its charter certificate and other documents, respondent was issued
under suspicious circumstances, and which contain allegations unsupported by Certificate of Creation of Local or Chapter PDMP-01 by the BLR on 6 July 1999.
evidence. At best, these affidavits are self-serving. They possess no probative
value. A retraction does not necessarily negate an earlier declaration. For this Respondent filed with the Med-Arbiter of the DOLE Regional Officer in the National
reason, retractions are looked upon with disfavor and do not automatically exclude Capital Region (DOLE-NCR), three separate petitions for certification election to
the original statement or declaration based solely on the recantation. It is imperative represent SMPP, SMCSU, and SMBP. All three petitions were dismissed, on the
that a determination be first made as to which between the original and the new ground that the separate petitions fragmented a single bargaining unit.
statements should be given weight or accorded belief, applying the general rules on
evidence. In this case, inasmuch as they remain bare allegations, the purported Petitioner filed with the DOLE-NCR a petition seeking the cancellation of
recantations should not be upheld. respondent's registration and its dropping from the rolls of legitimate labor
organizations. In its petition, petitioner accused respondent of committing fraud and
Nevertheless, even assuming the veracity of the affidavits of recantation, the falsification, and non-compliance with registration requirements in obtaining its
legitimacy of respondent as a labor organization must be affirmed. While it is true certificate of registration. It raised allegations that respondent violated Articles
that the withdrawal of support may be considered as a resignation from the union, 239(a), (b) and (c)10 and 234(c)11 of the Labor Code. Moreover, petitioner claimed
the fact remains that at the time of the union's application for registration, the affiants that PDMP is not a legitimate labor organization, but a trade union center, hence, it
were members of respondent and they comprised more than the required 20% cannot directly create a local or chapter.
membership for purposes of registration as a labor union. Article 234 of the Labor
Code merely requires a 20% minimum membership during the application for union
registration. It does not mandate that a union must maintain the 20% minimum DOLE REGIONAL DIRECTOR
membership requirement all throughout its existence. issued an Order dismissing the allegations of fraud and misrepresentation, and
irregularity in the submission of documents by respondent. Regional Director Lim
San Miguel Corporation Employees Union – Philippine Transport and General further ruled that respondent is allowed to directly create a local or chapter.
Workers Organization (SMCEU-PTGWO) v. San Miguel Packaging Products However, he found that respondent did not comply with the 20% membership
8
requirement and, thus, ordered the cancellation of its certificate of registration and
removal from the rolls of legitimate labor organizations. a. Fifty pesos (P50.00) registration fee;

BLR b. The names of its officers, their addresses, the principal address of the labor
Respondent appealed to BLR. BLR granted appeal. Ruled that As a chartered local organization, the minutes of the organizational meetings and the list of the workers
union, appellant is not required to submit the number of employees and names of who participated in such meetings;
all its members comprising at least 20% of the employees in the bargaining unit
where it seeks to operate. Thus, the revocation of its registration based on non- c. The names of all its members comprising at least twenty percent (20%) of all the
compliance with the 20% membership requirement does not have any basis in the employees in the bargaining unit where it seeks to operate;
rules. Petitioner filed MR but BLR denied.
d. If the applicant union has been in existence for one or more years, copies of its
CA annual financial reports; and
Petitioner filed Rule 65 certiorari before CA. CA affirned BLR, ruled that a registered
federation or national union may directly create a local by submitting to the BLR e. Four (4) copies of the constitution and by-laws of the applicant union, minutes of
copies of the charter certificate, the local's constitution and by-laws, the principal its adoption or ratification and the list of the members who participated in it
office address of the local, and the names of its officers and their addresses. Upon
complying with the documentary requirements, the local shall be issued a certificate A legitimate labor organization19 is defined as "any labor organization duly
and included in the roster of legitimate labor organizations. The [herein respondent] registered with the Department of Labor and Employment, and includes any branch
is an affiliate of a registered federation PDMP, having been issued a charter or local thereof." The mandate of the Labor Code is to ensure strict compliance with
certificate. Under the rules we have reviewed, there is no need for SMPPEU to show the requirements on registration because a legitimate labor organization is entitled
a membership of 20% of the employees of the bargaining unit in order to be to specific rights under the Labor Code, and are involved in activities directly
recognized as a legitimate labor union. affecting matters of public interest. Registration requirements are intended to afford
a measure of protection to unsuspecting employees who may be lured into joining
ISSUE: unscrupulous or fly-by-night unions whose sole purpose is to control union funds or
Whether or not respondent met requirements to be a legitimate labor use the labor organization for illegitimate ends. Legitimate labor organizations have
organization exclusive rights under the law which cannot be exercised by non-legitimate unions,
one of which is the right to be certified as the exclusive representativeof all the
RULING:NO ON ACCOUNT OF TRADE UNION NOT BEING ALLOWED TO employees in an appropriate collective bargaining unit for purposes of collective
CHARTER bargaining. The acquisition of rights by any union or labor organization, particularly
the right to file a petition for certification election, first and foremost, depends on
NOTE: Focus on registration requirements whether or not the labor organization has attained the status of a legitimate labor
organization.
Petitioner posits that respondent is required to submit a list of members comprising
at least 20% of the employees in the bargaining unit before it may acquire legitimacy, The applicable Implementing Rules enunciates a two-fold procedure for the creation
citing Article 234(c) of the Labor Code which stipulates that any applicant labor of a chapter or a local. The first involves the affiliation of an independent union with
organization, association or group of unions or workers shall acquire legal a federation or national union or industry union. The second, finding application in
personality and shall be entitled to the rights and privileges granted by law to the instant petition, involves the direct creation of a local or a chapter through the
legitimate labor organizations upon issuance of the certificate of registration based process of chartering.27
on the following requirements:
9
A duly registered federation or national union may directly create a local or chapter
by submitting to the DOLE Regional Office or to the BLR two copies of the following: (b) The resolution of membership of each member organization, approved by the
Board of Directors of such union;
(a) A charter certificate issued by the federation or national union indicating the
creation or establishment of the local/chapter; (c) The name and principal address of the applicant, the names of its officers and
their addresses, the minutes of its organizational meeting/s, and the list of member
(b) The names of the local/chapter's officers, their addresses, and the principal office organizations and their representatives who attended such meeting/s; and
of the local/chapter; and
(d) A copy of its constitution and by-laws and minutes of its ratification by a majority
(c) The local/chapter's constitution and by-laws; Provided, That where the of the presidents of the member organizations, provided that where the ratification
local/chapter's constitution and by-laws is the same as that of the federation or was done simultaneously with the organizational meeting, it shall be sufficient that
national union, this fact shall be indicated accordingly. the fact of ratification be included in the minutes of the organizational meeting.47

All the foregoing supporting requirements shall be certified under oath by the Evidently, while a "national union" or "federation" is a labor organization with at least
Secretary or the Treasurer of the local/chapter and attested to by its President.28 ten locals or chapters or affiliates, each of which must be a duly certified or
recognized collective bargaining agent;48 a trade union center, on the other hand,
The Implementing Rules stipulate that a local or chapter may be directly created by is composed of a group of registered national unions or federations.49
a federation or national union. A duly constituted local or chapter created in
accordance with the foregoing shall acquire legal personality from the date of filing The Implementing Rules, as amended by Department Order No. 9, provide that "a
of the complete documents with the BLR. The issuance of the certificate of duly registered federation or national union" may directly create a local or chapter.
registration by the BLR or the DOLE Regional Office is not the operative act that The provision reads:
vests legal personality upon a local or a chapter under Department Order No. 9.
Such legal personality is acquired from the filing of the complete documentary Section 1. Chartering and creation of a local/chapter. – A duly registered federation
requirements enumerated in Section 1, Rule VI.30 or national union may directly create a local/chapter by submitting to the Regional
Office or to the Bureau two (2) copies of the following:
However SC agreed with petitioners that respondent cannot be registered as it is a
chapter of a trade union center, not a legitimate labor organization. (a) A charter certificate issued by the federation or national union indicating the
creation or establishment of the local/chapter;
Culling from its definition as provided by Department Order No. 9, a trade union
center is any group of registered national unions or federations organized for the (b) The names of the local/chapter's officers, their addresses, and the principal office
mutual aid and protection of its members; for assisting such members in collective of the local/chapter; and
bargaining; or for participating in the formulation of social and employment policies,
standards, and programs, and is duly registered with the DOLE in accordance with (c) The local/chapter's constitution and by-laws; provided that where the
Rule III, Section 2 of the Implementing Rules.46 The same rule provides that the local/chapter's constitution and by-laws is the same as that of the federation or
application for registration of an industry or trade union center shall be supported by national union, this fact shall be indicated accordingly.
the following:
All the foregoing supporting requirements shall be certified under oath by the
(a) The list of its member organizations and their respective presidents and, in the Secretary or the Treasurer of the local/chapter and attested to by its President.50
case of an industry union, the industry where the union seeks to operate;
10
Department Order No. 9 mentions two labor organizations either of which is allowed created under the more lenient requirements for chartering, but must have complied
to directly create a local or chapter through chartering – a duly registered federation with the more stringent rules for creation and registration of an independent union,
or a national union. Department Order No. 9 defines a "chartered local" as a labor including the 20% membership requirement.
organization in the private sector operating at the enterprise level that acquired legal
personality through a charter certificate, issued by a duly registered federation or
national union and reported to the Regional Office in accordance with Rule III, Progressive Development Corporation v. The Honorable Secretary,
Section 2-E of these Rules Department of Labor and Employment, G.R. No. 96425, February 4, 1992.

FACTS
ART. 234-A. Chartering and Creation of a Local Chapter. — A duly registered Respondent Pambansang Kilusan ng Paggawa (KILUSAN) -TUCP (hereinafter
federation or national union may directly create a local chapter by issuing a charter referred to as Kilusan) filed with the Department of Labor and Employment (DOLE)
certificate indicating the establishment of the local chapter. The chapter shall acquire a petition for certification election among the rank-and-file employees of the
legal personality only for purposes of filing a petition for certification election from petitioner alleging that it is a legitimate labor federation and its local chapter,
the date it was issued a charter certificate. Progressive Development Employees Union, was issued a charter certificate.
Kilusan claimed that there was no existing collective bargaining agreement and that
The chapter shall be entitled to all other rights and privileges of a legitimate labor no other legitimate labor organization existed in the bargaining unit.
organization only upon the submission of the following documents in addition to its
charter certificate: Petitioner PDC filed its motion to dismiss dated July 11, 1990 contending that the
local union failed to comply with Rule II Section 3, Book V of the Rules Implementing
(a) The names of the chapter's officers, their addresses, and the principal office of the Labor Code, as amended, which requires the submission of: (a) the constitution
the chapter; and and by-laws; (b) names, addresses and list of officers and/or members; and (c)
books of accounts.
(b) The chapter's constitution and by-laws: Provided, That where the chapter's
constitution and by-laws are the same as that of the federation or the national union, Respondent Kilusan submitted a rejoinder to PDC's motion to dismiss claiming that
this fact shall be indicated accordingly. it had submitted the necessary documentary requirements for registration, such as
the constitution and by-laws of the local union, and the list of officers/members with
The additional supporting requirements shall be certified under oath by the secretary their addresses. Kilusan further averred that no books of accounts could be
or treasurer of the chapter and attested by its president. (Emphasis ours.) submitted as the local union was only recently organized

Article 234 now includes the term trade union center, but interestingly, the provision Petitioner insisted that upon verification with the Bureau of Labor Relations (BLR), it
indicating the procedure for chartering or creating a local or chapter, namely Article found that the alleged minutes of the organizational meeting was unauthenticated,
234-A, still makes no mention of a "trade union center." the list of members did not bear the corresponding signatures of the purported
members, and the constitution and by-laws did not bear the signature of the
Also worth emphasizing is that even in the most recent amendment of the members and was not duly subscribed. It argued that the private respondent
implementing rules,54 there was no mention of a trade union center as being among therefore failed to substantially comply with the registration requirements provided
the labor organizations allowed to charter. by the rules.

In sum, although PDMP as a trade union center is a legitimate labor organization, it MED-ARBITER
has no power to directly create a local or chapter. Thus, SMPPEU-PDMP cannot be
11
Held that there was substantial compliance with the requirements for the formation Article 242 enumerates the exclusive rights of a legitimate labor organization among
of the chapter. He further stated that mere issuance of the charter certificate by the which is the right to be certified as the exclusive representative of all the employees
federation was sufficient compliance with the rules. Considering that the in an appropriate collective bargaining unit for purposes of collective bargaining
establishment was unorganized, he maintained that a certification election should
be conducted to resolve the question of representation Ordinarily, a labor organization acquires legitimacy only upon registration with the
BLR. Under Article 234 (Requirements of Registration):
LABOR USEC
Treating the motion for reconsideration filed by the PDC as an appeal to the Office Any applicant labor organization, association or group of unions or workers shall
of the Secretary, Undersecretary Laguesma held that the same was merely a acquire legal personality and shall be entitled to the rights and privileges granted by
"reiteration of the issues already ventilated in the proceedings before the Med- law to legitimate labor organizations upon issuance of the certificate of registration
Arbiter, specifically, the matter involving the formal organization of the chapter. Also based on the following requirements:
denied MR
(a) Fifty-pesos (P50.00) registration fee;
ISSUE:
Whether or not private respondent complied with requirements for (b) The names of its officers, their addresses, the principal address of the labor
registration: organization, the minutes of the organizational meeting and the list of the workers
who participated in such meetings;
RULING:
NO (c) The names of all its members comprising at least twenty 20% percent of all the
employees in the bargaining unit where it seek to operate;
Petitioner's contention is that a labor organization may not validly invest the status
of legitimacy upon a local or chapter through the mere expedient of issuing a charter (d) If the applicant has been in existence for one or more years, copies , of its annual
certificate and submitting such certificate to the BLR Petitioner PDC posits that such financial reports; and
local or chapter must at the same time comply with the requirement of submission
of duly subscribed constitution and by-laws, list of officers and books of accounts. (e) Four copies of the constitution and by-laws of the applicant union, the minutes of
PDC points out that the constitution and by-laws and list of officers submitted were its adoption or ratification and the list of the members who participated in it.
not duly subscribed. Likewise, the petitioner claims that the mere filing of the
aforementioned documents is insufficient; that there must be due recognition or And under Article 235 (Action on Application)
acknowledgment accorded to the local or chapter by BLR through a certificate of
registration or any communication emanating from it. The Solicitor General, in behalf The Bureau shall act on all applications for registration within thirty (30) days from
of the public respondent, avers that there was a substantial compliance with the filing.
requirements for the formation of a chapter. Moreover, he invokes Article 257 of the
Labor Code which mandates the automatic conduct by the Med-Arbiter of a All requisite documents and papers shall be certified under oath by the secretary or
certification election in any establishment where there is no certified bargaining the treasurer of the organization, as the case may be, and attested to by its
agreement. president.

While Article 257 cited by the Solicitor General directs the automatic conduct of a Moreover, section 4 of Rule II, Book V of the Implementing Rules requires that the
certification election in an unorganized establishment, it also requires that the application should be signed by at least twenty percent (20%) of the employees in
petition for certification election must be filed by a legitimate labor organization. the appropriate bargaining unit and be accompanied by a sworn statement of the
12
applicant union that there is no certified bargaining agent or, where there is an
existing collective bargaining agreement duly submitted to the DOLE, that the (1) The requirement that the application for registration must be signed by at least
application is filed during the last sixty (60) days of the agreement. 20% of the employees in the appropriate bargaining unit;

But when an unregistered union becomes a branch, local or chapter of a federation, 2) The submission of officers' addresses, principal address of the labor organization,
some of the aforementioned requirements for registration are no longer required. the minutes of organizational meetings and the list of the workers who participated
The provisions governing union affiliation are found in Rule II, Section 3, Book V of in such meetings;
the Implementing Rules, the relevant portions of which are cited below:
3) The submission of the minutes of the adoption or ratification of the constitution
Sec. 3. Union affiliation; direct membership with national union. — An affiliate of a and by the laws and the list of the members who participated in it.
labor federation or national union may be a local or chapter thereof or an
independently registered union. In the case at bar, the constitution and by-laws and list of officers submitted in the
BLR, while attested to by the chapter's president, were not certified under oath by
a) The labor federation or national union concerned shall issue a charter certificate the secretary.
indicating the creation or establishment of a local or chapter, copy of which shall be
submitted to the Bureau of Labor Relations within thirty (30) days from issuance of A local or chapter therefore becomes a legitimate labor organization only upon
such charter certificate. submission of the following to the BLR:

b) An independently registered union shall be considered an affiliate of a labor 1) A charter certificate, within 30 days from its issuance by the labor federation or
federation or national union after submission to the Bureau of the contract or national union, and
agreement of affiliation within thirty (30) days after its execution.
2) The constitution and by-laws, a statement on the set of officers, and the books of
xxx xxx xxx accounts all of which are certified under oath by the secretary or treasurer, as the
case may be, of such local or chapter, and attested to by its president.
e) The local or chapter of a labor federation or national union shall have and maintain
a constitution and by laws, set of officers and books and accounts. For reporting Absent compliance with these mandatory requirements, the local or chapter does
purposes, the procedure governing the reporting of independently registered unions, not become a legitimate labor organization.
federations or national unions shall be observed.
In the case at bar, the failure of the secretary of PDEU-Kilusan to certify the required
Paragraph (a) refers to the local or chapter of a federation which did not undergo documents under oath is fatal to its acquisition of a legitimate status.
the rudiments of registration while paragraph (b) refers to an independently
registered union which affiliated with a federation. Implicit in the foregoing On June 19, 1990, or just three days after the organizational meeting, Kilusan filed
differentiation is the fact that a local or chapter need not be independently registered. a petition for certification election (Records, pages 2 and 3) accompanied by a copy
By force of law (in this case, Article 212[h]); such local or chapter becomes a each of the charter certificate, constitution and by-laws and minutes of the
legitimate labor organization upon compliance with the aforementioned provisions organizational meeting. Had the local union filed an application for registration, the
of Section 3. petition for certification election could not have been immediately filed. The applicant
union must firstly comply with the "20% signature" requirement and all the other
Thus, several requirements that are otherwise required for union registration are requisites enumerated in Article 234. Moreover, since under Article 235 the BLR
omitted, to wit: shall act on any application for registration within thirty (30) days from its filing, the
13
likelihood is remote that, assuming the union complied with all the requirements, the constitution and by-laws were adopted only on July 7, 1993, hence, there was no
application would be approved on the same day it was filed. bases for the supposed election of officers on June 27, 1993 because as of this
date, there existed no positions to which the officers could be validly elected;

2) Voting was not conducted by secret ballot in violation of Article 241, section (c) of
the Labor Code;

3) The Constitution and by Laws submitted in support of its petition were not properly
acknowledged and notarized.
ii. Action on Application and Denial of Registration
• Articles 242 and 243, Labor Code. Petitioner filed a Petition seeking the cancellation of the Union's registration on the
grounds of fraud and falsification, Motion was likewise filed by petitioner with the
Progressive Development Corporation – Pizza Hut v. Laguesma, G.R. No. Med-Arbiter requesting suspension of proceedings in the certification election case
115077, April 18, 1997. until after the prejudicial question of the Union's legal personality is determined in
the proceedings for cancellation of registration. However, in an Order. Med-Arbiter
Nagkakaisang Lakas ng Manggagawa (NLM)-Katipunan (respondent Union) filed a Rasidali C. Abdullah directed the holding of a certification election among petitioner's
petition for certification election with the Department of Labor (National Capital rank and file employees.
Region) in behalf of the rank and file employees of the Progressive Development
Corporation (Pizza Hut). MED-ARBITER
The Order explained Sumasaklaw sa Manggagawa ng Pizza Hut is a legitimate labor
Petitioner filed verified Motion to Dismiss the petition alleging fraud, falsification and organization in contemplation of law and shall remain as such until its very charter
misrepresentation in the respondent. Union's registration making it void and invalid. certificate is canceled or otherwise revoked by competent authority. The alleged
The motion specifically alleged that: a) respondent Union's registration was misrepresentation, fraud and false statement in connection with the issuance of the
tainted with false, forged, double or multiple signatures of those who allegedly charter certificate are collateral issues which could be properly ventilated in the
took part in the ratification of the respondent Union's constitution and by-laws cancellation proceedings.
and in the election of its officers that there were two sets of supposed
attendees to the alleged organizational meeting that was alleged to have taken DOLE USEC
place on June 26, 1993; that the alleged chapter is claimed to have been Denied appeal
supported by 318 members when in fact the persons who actually signed their
names were much less; and b) while the application for registration of the ISSUE:
charter was supposed to have been approved in the organizational meeting Whether or not DOLE USEC committed GAD in affirming Med-Arbiter decision
held on June 27, 1993, the charter certification issued by the federation considering that: (1) respondent Union's legal personality was squarely put in
KATIPUNAN was dated June 26, 1993 or one (1) day prior to the formation of issue; (2) allegations of fraud and falsification, supported by documentary
the chapter, thus, there were serious falsities in the dates of the issuance of the evidence were made; and (3) a petition to cancel respondent Union's
charter certification and the organization meeting of the alleged chapter. registration is pending with the regional office of the Department of Labor and
Employment
Other violations alleged in supplement to MTD included:
1.)Election of its officers was held on June 27, 1993; however, it appears from the HELD:
documents submitted by respondent union to the BIR-DOLE that the Union's YES.
14
In the public respondent's assailed Resolution dated December 29, 1993, the documents, a labor organization should be denied recognition as a legitimate labor
suggestion is made that once a labor organization has filed the necessary organization. And if a certificate of recognition has been issued, the propriety of the
documents and papers and the same have been certified under oath and attested labor organization's registration could be assailed directly through cancellation of
to, said organization necessarily becomes clothed with the character of a legitimate registration proceedings in accordance with Articles 238 and 239 of the Labor Code,
labor organization. or indirectly, by challenging its petition for the issuance of an order for certification
election.
In the first place, the public respondent's views as expressed in his December 29,
1993 Resolution miss the entire point behind the nature and purpose of proceedings These measures are necessary — and may be undertaken simultaneously — if the
leading to the recognition of unions as legitimate labor organizations. Article 234 of spirit behind the Labor Code's requirements for registration are to be given flesh and
the Labor Code provides: blood. Registration requirements specifically afford a measure of protection to
unsuspecting employees who may be lured into joining unscrupulous or fly-by-night
Art. 234. Requirements of registration. — Any applicant labor organization, unions whose sole purpose is to control union funds or use the labor organization
association or group of unions or workers shall acquire legal personality and shall for illegitimate ends. 12 Such requirements are a valid exercise of the police power,
be entitled to the rights and privileges granted by law to legitimate labor because the activities in which labor organizations, associations and unions of
organizations upon issuance of the certificate of registration based on the following workers are engaged directly affect the public interest and should be protected.
requirements:
Furthermore, the Labor Code itself grants the Bureau of Labor Relations a period of
(a) Fifty pesos (P50.00) registration fee; thirty (30) days within which to review all applications for registration. Article 235
provides:
(b) The names of its officers, their addresses, the principal address of the labor
organization, the minutes of the organizational meetings and the list of the workers Art. 235. Action on application. — The Bureau shall act on all applications for
who participated in such meetings; registration within thirty (30) days from filing.

(c) The names of all its members comprising at least twenty percent (20%) of all the All requisite documents and papers shall be certified under oath by the secretary or
employees in the bargaining unit where it seeks to operate; the treasurer of the organization, as the case may be, and attested to by its
president.
(d) If the applicant union has been in existence for one or more years, copies of its
annual financial reports; and The thirty-day period in the aforecited provision ensures that any action taken by the
Bureau of Labor Relations is made in consonance with the mandate of the Labor
(e) Four (4) copies of the constitution and by-laws of the applicant union, minutes of Code, which, it bears emphasis, specifically requires that the basis for the issuance
its adoption or ratification, and the list of the members who participated in it. of a certificate of registration should be compliance with the requirements for
recognition under Article 234. Since, obviously, recognition of a labor union or labor
A more than cursory reading of the aforecited provisions clearly indicates that the organization is not merely a ministerial function, the question now arises as to
requirements embodied therein are intended as preventive measures against the whether or not the public respondent committed grave abuse of discretion in
commission of fraud. After a labor organization has filed the necessary papers and affirming the Med-Arbiter's order in spite of the fact that the question of the Union's
documents for registration, it becomes mandatory for the Bureau of Labor Relations legitimacy was squarely put in issue and that the allegations of fraud and falsification
to check if the requirements under Article 234 have been sedulously complied with. were adequately supported by documentary evidence.
If its application for registration is vitiated by falsification and serious irregularities,
especially those appearing on the face of the application and the supporting
15
It cannot be denied that the grounds invoked by petitioner for the cancellation of iii. Effect of Registration, Non-registration, and
respondent Union's registration fall under paragraph (a) and (c) of Article 239 of the Acquisition of Legal Personality
Labor Code, to wit: • Articles 240 and 241, Labor Code.

(a) Misrepresentation, false statement or fraud in connection with the adoption or San Miguel Corporation (Mandaue Packaging Products Plants) v. Mandaue
ratification of the constitution and by-laws or amendments thereto, the minutes of Packing Products Plants – San Miguel Packaging Products – San Miguel
ratification, the list of members who took part in the ratification of the constitution Corporation Monthlies Rank-and-File Union – FFW (MPPP-SMPP-SMAMRFU-
and by-laws or amendments thereto, the minutes of ratification, the list of members FFW), G.R. No. 152356, August 16, 2005.
who took part in the ratification;
FACTS
xxx xxx xxx Respondent, identifying itself as an affiliate of Federation of Free Workers (FFW),
filed a petition for certification election with the DOLE Regional Office No. VII. In the
(c) Misrepresentation, false statements or fraud in connection with the election of petition, respondent stated that it sought to be certified and to represent the
officers, minutes of the election of officers, the list of voters, or failure to submit these permanent rank-and-file monthly paid employees of the petitioner.2 The following
documents together with the list of the newly elected-appointed officers and their documents were attached to the petition: (1) a Charter Certificate issued by FFW on
postal addresses within thirty (30) days from election. 5 June 1998 certifying that respondent as of that date was duly certified as a local
or chapter of FFW; (2) a copy of the constitution of respondent prepared by its
xxx xxx xxx Secretary, Noel T. Bathan and attested by its President, Wilfred V. Sagun; (3) a list
of respondent’s officers and their respective addresses, again prepared by Bathan
The grounds ventilated in cancellation proceedings in accordance with Article 239 and attested by Sagun; (4) a certification signifying that respondent had just been
of the Labor Code constitute a grave challenge to the right of respondent Union to organized and no amount had yet been collected from its members, signed by
ask for certification election. The Med-Arbiter should have looked into the merits of respondent’s treasurer Chita D. Rodriguez and attested by Sagun; and (5) a list of
the petition for cancellation before issuing an order calling for certification election. all the rank-and-file monthly paid employees of the Mandaue Packaging Products
Registration based on false and fraudulent statements and documents confer no Plants and Mandaue Glass Plant prepared by Bathan and attested by Sagun.
legitimacy upon a labor organization irregularly recognized, which, at best, holds on
to a mere scrap of paper. Under such circumstances, the labor organization, not Petitioner filed a motion to dismiss the petition for certification election on the sole
being a legitimate labor organization, acquires no rights, particularly the right to ask ground that herein respondent is not listed or included in the roster of legitimate
for certification election in a bargaining unit. labor organizations based on the certification issued by the Officer-In-Charge,
Regional Director of the DOLE Regional Office No. VII.
it would have been more prudent for the Med-Arbiter and public respondent to have
granted petitioner's request for the suspension of proceedings in the certification Respondent submitted to the Bureau of Labor Relations the same documents earlier
election case, until the issue of the legality of the Union's registration shall have been attached to its petition for certification. The accompanying letter, signed by
resolved. Failure of the Med-Arbiter and public respondent to heed the request respondent’s president Sagun, stated that such documents were submitted in
constituted a grave abuse of discretion. compliance with the requirements for the creation of a local/chapter pursuant to the
NOTE: case was REMANDED to the Med-Arbiter to resolve with reasonable Labor Code and its Implementing Rules; and it was hoped that the submissions
dispatch petitioner's petition for cancellation of respondent Union's would facilitate the listing of respondent under the roster of legitimate labor
registration organizations. the Chief of Labor Relations Division of DOLE Regional Office No.
VII issued a Certificate of Creation of Local/Chapter certifying that from 30 July 1998,

16
respondent has acquired legal personality as a labor organization/worker’s RULING:
association, it having submitted all the required documents. YES
At the onset, the arguments raised by petitioner on this point are plainly erroneous.
Respondent instead filed a Position Paper wherein it asserted that it had complied Petitioner cites the case of Toyota Motor Philippines v. Toyota Motor Philippines
with all the necessary requirements for the conduct of a certification election, and Corporation Labor Union,38 and the purported holding therein that "[if] it is true that
that the ground relied upon in the Motion to Dismiss was a mere technicality at the time of the filing of the petition, the said registration certificate has not been
approved yet, then, petitioner lacks the legal personality to file the petition."
Petitioner filed a Comment, wherein it reiterated that respondent was not a legitimate However, an examination of the case actually reveals that the cited portion was lifted
labor organization at the time of the filing of the petition. Petitioner also propounded from one of the antecedent rulings of the Med-Arbiter in that case which had not
that contrary to respondent’s objectives of establishing an organization representing even been affirmed or reinstated by the Court on review.40 Moreover, such
rank-and-file employees, two of respondent’s officers, namely Vice-President pronouncement made prior to the enactment of Department Order No. 9 squarely
Emannuel L. Rosell and Secretary Bathan, were actually supervisory employees. In contradicts Section 3, Rule VI thereof, which provides that legal personality of the
support of this allegation, petitioner attached various documents evidencing the local/chapter is vested upon the submission of the complete documentary
designation of these two officers in supervisory roles, as well as their exercise of requirements.
various supervisory functions.9 Petitioner cited Article 245 of the Labor Code, which
provides that supervisory employees shall not be eligible for membership in a labor It is also worth noting that petitioner union in Toyota was an independent labor union,
organization of the rank-and-file employees. and not a local/chapter, and under Department Order No. 9, independent labor
unions, unlike local/chapters, acquire legal personality only upon issuance of the
Petitioner’s petition to cancel union registration was denied and CA denied their certificate of registration by the Bureau or Regional Office.
appeal. The decision became final and executory. Despite this Med-Arbiter Manit
issued an Order dismissing respondent’s petition for certification election. The sole In regular order, it is the federation or national union, already in possession of legal
ground relied upon for the dismissal was the Med-Arbiter’s Opinion that as of the personality, which initiates the creation of the local/chapter. It issues a charter
date of filing of the petition on 15 June 1998, respondent did not have the legal certificate indicating the creation or establishment of the local/chapter. It then
personality to file the said petition for certification election. submits this charter certificate, along with the names of the local/chapter’s officers,
constitution and by-laws to the Regional Office or Bureau. It is the submission of
DOLE USEC these documents, certified under oath by the Secretary or Treasurer of the
Respondent appealed to DOLE. DOLE USEC Rosalinda Dimapilis-Baldoz reversed local/chapter and attested by the President, which vests legal personality in the
ruling that respondent acquired legal personality as early as 15 June 1998, the date local/chapter, which is then free to file on its own a petition for certification election.
it submitted the required documents, citing Section 3, Rule VI of the New Rules
Implementing the Labor Code (Implementing Rules) which deems that a In this case, the federation in question, the FFW, did not submit any of these
local/chapter acquires legal personality from the date of filing of the complete documentary requirements to the Regional Office or Bureau. It did however issue a
documentary requirements as mandated in the Implementing Rules. The DOLE also charter certificate to the putative local/chapter (herein respondent). Respondent
ruled that the contention that two of respondent’s officers were actually supervisors then submitted the charter certificate along with the other documentary requirements
can be threshed out in the pre-election conferences where the list of qualified voters to the Regional Office, but not for the specific purpose of creating the local/chapter,
is to be determined. but for filing the petition for certification election.

ISSUE: It could be properly said that at the exact moment respondent was filing the petition
Whether or not the union acquired legal personality for certification, it did not yet possess any legal personality, since the requisites for
acquisition of legal personality under Section 3, Rule VI of Department Order No. 9
17
had not yet been complied with. It could also be discerned that the intention of the Article IV establishes the requisites for membership in the local/chapter.
Labor Code and its Implementing Rules that only those labor organizations that have Articles V and VI name the various officers and what their respective functions are.
acquired legal personality are capacitated to file petitions for certification elections. The procedure for election of these officers, including the necessary vote
Such is the general rule. requirements, is provided for in Article IX, while Article XV delineates the procedure
for the impeachment of these officers.
Yet there are peculiar circumstances in this case that allow the Court to rule that Article VII establishes the standing committees of the local/chapter and how their
respondent acquired the requisite legal personality at the same time it filed the members are appointed.
petition for certification election. In doing so, the Court acknowledges that the strict Article VIII lays down the rules for meetings of the union, including the notice and
letter of the procedural rule was not complied with. However, labor laws are quorum requirements thereof.
generally construed liberally in favor of labor, especially if doing so affirms the Article X enumerates with particularity the rules for union dues, special
constitutionally guaranteed right to self-organization. assessments, fines, and other payments.
Article XII provides the general rule for quorum in meetings of the Board of Directors
True enough, there was no attempt made by the national federation, or the and of the members of the local/chapter, and cites the applicability of the Robert’s
local/chapter for that matter, to submit the enumerated documentary requirements Rules of Order43 in its meetings.
to the Regional Office or Bureau for the specific purpose of creating the Article XVI governs and institutes the requisites for the amendment of the
local/chapter. However, these same documents were submitted by the local/chapter constitution.
to the Regional Office as attachments to its petition for certification election. Under
Section 3, Rule VI of Department Order No. 9, it is the submission of these same The Court likewise sees no impediment in deeming respondent as having acquired
documents to the Regional Office or Bureau that operates to vest legal personality legal personality as of 15 June 1998, the fact that it was the local/chapter itself, and
on the local/chapter. not the FFW, which submitted the documents required under Section 1, Rule VI of
Department Order No. 9.In the ordinary course, it should have been FFW, and not
Thus, in order to ascertain when respondent acquired legal personality, we only respondent, which should have submitted the subject documents to the Regional
need to determine on what date the Regional Office or Bureau received the complete Office. Nonetheless, there is no good reason to deny legal personality or defer its
documentary requirements enumerated under Section 1, Rule VI of Department conferral to the local/chapter if it is evident at the onset that the federation or national
Order No. 9. There is no doubt that on 15 June 1998, or the date respondent filed union itself has already through its own means established the local/chapter.
its petition for certification election, attached thereto were respondent’s constitution,
the names and addresses of its officers, and the charter certificate issued by the
national union FFW. The first two of these documents were duly certified under oath Tropical Hut Employees’ Union-CGW v. Tropical Hut Food Market, Inc., G.R.
by respondent’s secretary Bathan and attested to by president Sagun Nos. L-43495-99, January 20, 1990

It may be noted though that respondent never submitted a separate by-laws, nor FACTS
does it appear that respondent ever intended to prepare a set thereof. Section 1(c), On January 2, 1968, the rank and file workers of the Tropical Hut Food Market
Rule VI, Book V of Department Order No. 9 provides that the submission of both a Incorporated, organized a local union called the Tropical Hut Employees
constitution and a set of by-laws is required, or at least an indication that the Union(THEU), elected their officers, adopted their constitution and by-laws and
local/chapter is adopting the constitution and by-laws of the federation or national immediately sought affiliation with the National Association of Trade Unions (NATU).
union. The NATU accepted the THEU application for affiliation. Following such affiliation
with NATU, Registration Certificate was issued by the Department of Labor in the
An examination of respondent’s constitution reveals it sufficiently comprehensive in name of the Tropical Hut Employees Union — NATU. It appears, however, that
establishing the necessary rules for its operation. NATU itself as a labor federation, was not registered with the Department of Labor
18
We, the undersigned, hereby designate the NATIONAL Association of Trade
After several negotiations were conducted between THEU-NATU, represented by Unions, of which the TROPICAL HUT EMPLOYEES UNION is an affiliate as sole
its local president and the national officers of the NATU, particularly Ignacio Lacsina, collective bargaining agent in all matters relating to salary rates, hours of work and
President, Pacifico Rosal, Executive Vice-President and Marcelino Lontok, Jr., Vice other terms and conditions of employment in the Tropical Hut Food Market, Inc. and
President, and respondent Tropical Hut Food Market, Incorporated, thru its we hereby authorize the said company to deduct the amount of Four (P 4.00) Pesos
President and General Manager, Cesar Azcona, Sr., a Collective Bargaining each every month as our monthly dues and to deliver the amount to the Treasurer
Agreement was concluded between the parties on April 1, 1968, the term of which of the Union or his duly authorized representatives.
expired on March 31, 1971.
On May 21, 1971, respondent company and THEU-NATU entered into a new
The CBA included the following stipulations: Collective Bargaining Agreement which ended on March 31, 1974. This new CBA
incorporated the previous union-shop security clause and the attached check-off
Article I authorization form.

Coverage and Effectivity Sometime in July, 1973, Arturo Dilag, incumbent President of THEU-NATU, was
appointed by the respondent company as Assistant Unit Manager. On July 24, 1973,
Sec. 1. The COMPANY recognizes the UNION as the sole and exclusive collective he wrote the general membership of his union that for reason of his present position,
bargaining agent for all its workers and employees in all matters concerning wages, he was resigning as President of the THEU-NATU effective that date. As a
hours of work, and other terms and conditions of employment. consequence thereof, his Vice-President, Jose Encinas, assumed and discharged
the duties of the presidency of the THEU-NATU.
xxx xxx xxx
On December 19,1973, NATU received a letter dated December 15, 1973, jointly
Article III signed by the incumbent officers of the local union informing the NATU that THEU
was disaffiliating from the NATU federation. On December 20, 1973, the Secretary
Union Membership and Union Check-off of the THEU, Nemesio Barro, made an announcement in an open letter to the
general membership of the THEU, concerning the latter's disaffiliation from the
Sec. 1 —. . . Employees who are already members of the UNION at the time of the NATU and its affiliation with the Confederation of General Workers (CGW)
signing of this Agreement or who become so thereafter shall be required to maintain
their membership therein as a condition of continued employment. On January 9, 1974, Pacifico Rosal, President of the Confederation of General
Workers (CGW), wrote a letter in behalf of complainant THEU-CGW to the
xxx xxx xxx respondent company demanding the remittance of the union dues collected by the
Tropical Hut Food Mart, Incorporated to the THEU-CGW, but this was refused by
Sec. 3—Any employee who is expelled from the UNION for joining another the respondent company.
federation or forming another union, or who fails or refuses to maintain his
membership therein as required, . . . shall, upon written request of the UNION be The then so-called THEU-CGW held its annual election of officers, with Jose
discharged by the COMPANY. (Rollo, pp. 667-670) Encinas elected as President. On January 3, 1974, Encinas, in his capacity as
THEU-CGW President, informed the respondent company of the result of the
And attached to the Agreement as Appendix "A" is a check-off Authorization Form, elections. On January 9, 1974, Pacifico Rosal, President of the Confederation of
the terms of which are as follows: General Workers (CGW), wrote a letter in behalf of complainant THEU-CGWto the
respondent company demanding the remittance of the union dues collected by the
19
Tropical Hut Food Mart, Incorporated to the THEU-CGW, but this was refused by
the respondent company. All employees enjoy the right to self organization and to form and join labor
A request made by the NATU federation to the respondent company to dismiss him organizations of their own choosing for the purpose of collective bargaining and to
(Encinas) in view of his violation of Section 3 of Article III of the Collective Bargaining engage in concerted activities for their mutual aid or protection. This is a
Agreement. fundamental right of labor that derives its existence from the Constitution. In
The respondent company applied for clearance with the Secretary of Labor to interpreting the protection to labor and social justice provisions of the Constitution
dismiss the other officers and members of THEU-CGW. The company also and the labor laws or rules or regulations, We have always adopted the liberal
suspended them effective that day. approach which favors the exercise of labor rights.
NATU received a letter jointly signed by the incumbent officers of the local union
informing the NATU that THEU was disaffiliating from the NATU federation. The Further, there is no merit in the contention of the respondents that the act of
employees were dismissed because, as respondent company contended, they disaffiliation violated the union security clause of the CBA and that their dismissal
violated the union security clause. as a consequence thereof is valid. A perusal of the collective bargaining agreements
shows that the THEU-NATU, and not the NATU federation, was recognized as the
Not properly discussed in full text of the case but certain employees were dismissed sole and exclusive collective bargaining agent for all its workers and employees in
due to the disaffiliation from NATU supposedly due to CBA violation all matters concerning wages, hours of work and other terms and conditions of
employment Although NATU was designated as the sole bargaining agent in the
LABOR ARBITER check-off authorization form attached to the CBA, this simply means it was acting
Ordered reinstatement of dismissed employees only for and in behalf of its affiliate. The NATU possessed the status of an agent
while the local union remained the basic principal union which entered into contract
NLRC with the respondent company.
Granted appeal of private respondent, ordered the respondent company, under pain
of being cited for contempt for failure to do so, to give the individual complainants a When the THEU disaffiliated from its mother federation, the former did not lose its
second chance by reemploying them upon their voluntary reaffirmation of legal personality as the bargaining union under the CBA. Moreover, the union
membership and loyalty to the Tropical Hut Employees Union-NATU and the security clause embodied in the agreements cannot be used to justify the dismissals
National Association of Trade Unions in the event it hires additional personnel. meted to petitioners since it is not applicable to the circumstances obtaining in this
case. The CBA imposes dismissal only in case an employee is expelled from the
DOLE SEC union for joining another federation or for forming another union or who fails or
Affirmed NLRC refuses to maintain membership therein. The case at bar does not involve the
withdrawal of merely some employees from the union but of the whole THEU itself
from its federation. Clearly, since there is no violation of the union security provision
ISSUES: in the CBA, there was no sufficient ground to terminate the employment of
Whether THEU-CGW’s disaffiliation was valid and whether they lost legal petitioners. Petition granted
personality by disaffiliating with NATU

HELD: iv. Rights of Legitimate Labor Organization


YES. The right of a local union to disaffiliate from its mother federation is well-settled. • Article 251, Labor Code.
A local union, being a separate and voluntary association, is free to serve the interest
of all its members including the freedom to disaffiliate when circumstances warrant. Baptista v. Villanueva, G.R. No. 194709, July 31, 2013.
This right is consistent with the constitutional guarantee of freedom of association.
20
FACTS RPNEU’s officers informed their company of the expulsion of petitioners and the 12
Petitioners were former union members of Radio Philippines Network Employees others from the union and requested the management to serve them notices of
Union (RPNEU), a legitimate labor organization and the sole and exclusive termination from employment in compliance with their CBA’s union security clause.
bargaining agent of the rank and file employees of Radio Philippines Network (RPN), RPN HRD Manager, Lourdes Angeles, informed petitioners and the 12 others of the
a government-sequestered corporation involved in commercial radio and television termination of their employment effective March 20, 2006, enforcing Article II,
broadcasting affairs Section 216 also known as the union security clause of their current CBA.

Respondents were the union’s elected officers and members Petitioners filed three complaints for Unfair Labor Practice questioning legality of
their expulsion
On suspicion of union mismanagement, petitioners, together with some other union
members, filed a complaint for impeachment of their union president, Reynato
Siozon, before the executive board of RPN, which was eventually abandoned. They LABOR ARBITER
later re-lodged the impeachment complaint, this time, against all the union officers Favored petitioners, adjudged the respondents guilty of ULP pursuant to Article 249
and members of RPNEU before the Department of Labor and Employment (DOLE). (a) and (b) of the Labor Code. The LA clarified that only the union officers of RPNEU
They likewise filed various petitions for audit covering the period from 2000 to 2004 could be held responsible for ULP, so they exonerated six (6) of the original
defendants who were mere union members. The LA also ordered the reinstatement
Two written complaints were filed against petitioners and several others for alleged of petitioners as bonafide members of RPNEU.
violation of the union’s Constitution and By-Laws.Months later a different group of
union members filed a third complaint against petitioners and 12 others, before the NLRC
Chairman of RPNEU’s Committee on Grievance and Investigation citing as grounds Set aside LA decision. NLRC found that petitioners filed a suit calling for the
the "commission of an act which violates RPNEU Constitution and By-Laws, impeachment of the officers and members of the Executive Board of RPNEU without
specifically: first resorting to internal remedies available under its own Constitution and By-Laws.
Article IX, Section 2.2 for joining or forming a union outside the sixty (60) days period The NLRC likewise decreed that the LA’s order of reinstatement was improper
Article IX, Section 2.5 for urging or advocating that a member start an action in any because the legality of the membership expulsion was not raised in the proceedings
court of justice or external investigative body against the Union or its officer without and, hence, beyond the jurisdiction of the LA
first exhausting all internal remedies open to him or available in accordance with the
CBL CA
Affirmed NLRC. CA stated that the termination of employment by virtue of a union
Petitioners and their group, through an exchange of communications with the security clause was recognized in our jurisdiction. It explained that the said practice
Committee, denied the charges imputed against them and contested the procedure fortified the union and averted disunity in the bargaining unit within the duration of
adopted by the Committee in its investigation. The Committee submitted their the CBA. The CA declared that petitioners were accorded due process before they
recommendation of expulsion from the union to RPNEU’s Board of were removed from office. In fact, petitioners were given the opportunity to explain
Directors.RPNEU’s Board of Directors affirmed the recommendation of expulsion of their case and they actually availed of said opportunity by submitting letters
petitioners and the 12 others from union membership in a Board Resolution. containing their arguments

Petitioners with the 12 others wrote to RPNEU’s President and Board of Directors ISSUE:
that their expulsion from the union was an ultra vires act because the Committee Whether the union members are guilty of ULP
failed to observe the basic elements of due process because they were not given
the chance to physically confront and examine their complainants. In a letter, RULING:
21
NO to the committee on Grievance and Investigation. The Committee shall hear any
The primary concept of ULP is embodied in Article 247 of the Labor Code, which charge and subsequently, forward its finding and recommendation to the BOD. The
provides: BOD has the power to approve or nullify the recommendation of the Committee on
Grievance and Investigation based on the merit of the appeal.
Article 247. Concept of unfair labor practice and procedure for prosecution thereof.– was correctly applied under the circumstances.
–Unfair labor practices violate the constitutional right of workers and employees to
self-organization, are inimical to the legitimate interests of both labor and It is well-settled that workers’ and employers’ organizations shall have the right to
management, including their right to bargain collectively and otherwise deal with draw up their constitutions and rules to elect their representatives in full freedom, to
each other in an atmosphere of freedom and mutual respect, disrupt industrial peace organize their administration and activities and to formulate their programs.In this
and hinder the promotion of healthy and stable labor-management relations. case, RPNEU’s Constitution and By-Laws expressly mandate that before a party is
allowed to seek the intervention of the court, it is a pre-condition that he should have
In essence, ULP relates to the commission of acts that transgress the workers’ right availed of all the internal remedies within the organization. Petitioners were found to
to organize. As specified in Articles 248 and 249 of the Labor Code, the prohibited have violated the provisions of the union’s Constitution and By-Laws when they filed
acts must necessarily relate to the workers' right to self-organization and to the petitions for impeachment against their union officers and for audit before the DOLE
observance of a CBA. Absent the said vital elements, the acts complained, although without first exhausting all internal remedies available within their organization. This
seemingly unjust, would not constitute ULP act is a ground for expulsion from union membership. Thus, petitioners’ expulsion
from the union was not a deliberate attempt to curtail or restrict their right to organize,
ART. 249. UNFAIR LABOR PRACTICES OF LABOR ORGANIZATIONS.- It shall but was triggered by the commission of an act, expressly sanctioned by Section 2.5
be unfair labor practice for a labor organization, its officers, agents or of Article IX of the union’s Constitution and By-Laws.
representatives:
Petitioners failed to discharge the burden required to prove the charge of ULP
(a) To restrain or coerce employees in the exercise of their rights to self- against the respondents. Aside from their self-serving allegations, petitioners were
organization. However, a labor organization shall have the right to prescribe its own not able to establish how they were restrained or coerced by their union in a way
rules with respect to the acquisition or retention of membership: that curtailed their right to self-organization.petitioners had the burden of adducing
substantial evidence to support its allegations of ULP, which burden they failed to
(b) To cause or attempt to cause an employer to discriminate against an employee, discharge. In fact, both the NLRC and the CA found that petitioners were unable to
including discrimination against an employee with respect to whom membership in prove their charge of ULP against the respondents
such organization has been denied or to terminate an employee on any ground other
than the usual terms and conditions under which membership or continuation of
membership is made available to other members;

Based on RPNEU’s Constitution and By-Laws, the charges against petitioners were Acedera v. International Container Terminal Services, Inc. (ICTSI), G.R. No.
not mere internal squabbles, but violations that demand proper investigation 146073, January 13, 2003.
because, if proven, would constitute grounds for their expulsion from the union. As FACTS:
such, Article X, Investigation Procedures and Appeal Process of RPNEU’s Jerry Acedera, Et. Al. are employees of herein private respondent International
Constitution and By-Laws, which reads – Container Terminal Services, Inc. (ICTSI) and are officers/members of Associated
Port Checkers & Workers Union-International Container Terminal Services, Inc.
SECTION 1. Charge against any member or officer of the Union shall be submitted Local Chapter (APCWU-ICTSI), a labor organization duly registered as a local
to the Board of Directors (BOD) in writing, which shall refer the same, if necessary, affiliate of the Associated Port Checkers & Workers Union (APCWU).
22
CA’s Ruling:
On September 28, 1990, ICTSI entered into its first Collective Bargaining Agreement ● dismissed APCWU’s petition on the following grounds: failure to allege
(CBA) with APCWU with a term of five years effective until September 28, 1995. The when its motion for reconsideration of the NLRC decision was filed, failure
CBA was renegotiated and thereafter renewed through a second CBA that took to attach the necessary appendices to the petition, and failure to file its
effect on September 29, 1995, effective for another five years. Thus, in accordance motion for extension to file its petition within the reglementary period.
with the above-quoted provision of the CBA, the employees’ work week was reduced ● Records shows that petitioners, claiming to be employees of ICTSI, are
to five days or a total of 250 days a year. ICTSI, however, continued using the 304- already well represented by its employees union, APCWU.
day divisor in computing the wages of the employees.
ISSUE:
Regional Tripartite Wage and Productivity Board (RTWPB) in the NCR decreed a Whether or not Acedera et al. have no legal right to intervene in the case as their
P17.00 daily wage increase for all workers and employees receiving P125.00 per intervention was a superfluity -YES.
day or lower in the National Capital Region. 7 The then president of APCWU,
together with some union members, thus requested the ICTSI’s Human Resource HELD:
Department/Personnel Manager to compute the actual monthly increase in the Petitioner’s contention is based on Rule 19 of the 1997 Rules of Civil Procedure,
employees’ wages by multiplying the RTWPB mandated increase by 365 days and Section 1. They stress that they have complied with the requisites for intervention
dividing the product by 12 months. ICTSI stopped using 304 days as divisor and because (1) they are the ones who stand to gain or lose by the direct legal operation
started using 365 days to determine the daily wage. and effect of any judgment that may be rendered in this case, (2) no undue delay or
prejudice would result from their intervention since their Complaint-in-Intervention
Later on, ICTSI went on a retrenchment program and laid off its on-call employees. with Motion for Intervention was filed while the Labor Arbiter was still hearing the
This prompted the APCWU-ICTSI to file a notice of strike which included as cause case and before any decision thereon was rendered, and (3) it was not possible for
of action not only the retrenchment of the employees but also ICTSI’s use of 365 them to file a separate case as they would be guilty of forum shopping because the
days as divisor in the computation of wages. The dispute respecting the only forum available for them was the Labor Arbiter.
retrenchment was resolved by a compromise settlement while that respecting the
computation of wages was referred to the Labor Arbiter. Acedera et al. filed a Motion However, SC ruled that Petitioners failed to consider, in addition to the rule on
to Intervene. intervention, the rule on representation, thusly:
Sec. 3. Representatives as parties. — Where the action is allowed to be prosecuted
LA’s Ruling: or defended by a representative or someone acting in a fiduciary capacity, the
● The correct divisor in computing the daily wage and other labor standard beneficiary shall be included in the title of the case and shall be deemed to be the
benefits of the employees of ICTSI who are members of the complainant real party in interest. A representative may be a trustee of an express trust, a
Union as well as the other employees similarly situated is 250 days. guardian, an executor or administrator, or a party authorized by law or these Rules.
● ICTS is ordered to pay the employees concerned the differentials A labor union is one such party authorized to represent its members under Article
representing the underpayment of said salaries and other benefits. 242(a) of the Labor Code which provides that a union may act as the representative
● Denied petitioners-appellants’ Complaint-in-Intervention with Motion for of its members for the purpose of collective bargaining. This authority includes the
Intervention upon a finding that they are already well represented by power to represent its members for the purpose of enforcing the provisions of the
APCWU CBA. That APCWU acted in a representative capacity "for and on behalf of its Union
members and other employees similarly situated," the title of the case filed by it at
NLRC’s Ruling: the Labor Arbiter’s Office so expressly states.
● Reversed the decision of the LA and dismissed APCWU’s complaint.

23
While a party acting in a representative capacity, such as a union, may be petition for certification election may be refiled as soon as the said requirements are
permitted to intervene in a case, ordinarily, a person whose interests are met.
already represented will not be permitted to do the same except when there is
a suggestion of fraud or collusion or that the representative will not act in Med-Arbiter Manit’s Ruling:
good faith for the protection of all interests represented by him. ● Usec Laguesma, granted the second petition for certification election of
private respondent IBM.
To reiterate, for a member of a class to be permitted to intervene in a representative
action, fraud or collusion or lack of good faith on the part of the representative must SMFI’s contention upon appeal:
be proven. It must be based on facts borne on record. Mere assertions, as what Med-Arbiter erred in directing the conduct of certification election considering that
petitioners-appellants proffer, do not suffice. the local or chapter of IBM at SMFI is still not a legitimate labor organization with a
right to be certified as the exclusive bargaining agent in petitioner's establishment
San Miguel Foods, Inc. – Cebu B-MEG Feed Plant v. Lauesma, G.R. No. 116172, based on two grounds: (1) the authenticity and due execution of the Charter
October 10, 1996. Certificate submitted by IBM in favor of its local at SMFI cannot yet be ascertained
as it is still not known who is the legitimate and authorized representative of the IBM
FACTS: Federation who may validly issue said Charter Certificate; and (2) a group of workers
On September 24, 1993, a petition for certification election among the monthly-paid or a local union shall acquire legal personality only upon the issuance of a Certificate
employees of the San Miguel Food, Inc.-Cebu B-Meg Feeds Plant was filed by labor of Registration by the Bureau of Labor Relations under Article 234 of the Labor
federation Ilaw at Buklod ng Mangagawa (IBM) before Med-Arbiter Achilles V. Manit, Code, which IBM at SMFI did not possess.
alleging, that it is a legitimate labor organization duly registered with DOLE. SMFI-
Cebu B-Meg Feeds Plant (SMFI), is a business entity duly organized and existing Secretary of Labor and Employment’s Ruling:
under the laws of the Philippines which employs roughly 75 monthly paid employees, ● Denied the appeal
almost all of whom support the present petition. It was submitted in the petition that
there has been no certification election conducted in SMFI to determine the sole and ISSUE:
exclusive bargaining agent thereat for the past 2 years and that the proposed Whether or not SMFI is not a legitimate labor organization on the basis of Article 234
bargaining unit, which is SMFI's monthly paid employees, is an unorganized one. It of the Labor Code, any labor organization shall acquire legal personality only upon
was also stated therein that IBM has already complied with the mandatory the issuance of the Certificate of Registration by the Bureau of Labor Relations -
requirements for the creation of its local or affiliate in SMFI's establishment. NO.

SMFI filed a Motion to Dismiss on the ground that a similar petition remains pending HELD:
between the same parties for the same cause of action before Med-Arbiter Achilles Article 212(h) of the Labor Code defines a legitimate labor organization as "any labor
V. Manit. The earlier petition, pertains to the the issuance of an order by the Med- organization duly registered with the Department of Labor and Employment, and
Arbiter allowing the conduct for a certification election in SMFI's establishment. includes any branch or local thereof ."

IBM filed its Opposition to SMFI's Motion to Dismiss contending, case referred to by It is important to determine whether or not a particular labor organization is legitimate
SMFI had already been resolved by Med-Arbiter Manit in his Resolution and Order since legitimate labor organizations have exclusive rights under the law which
wherein IBM's first petition for certification election was denied mainly due to IBM's cannot be exercised by non-legitimate unions, one of which is the right to be certified
failure to comply with certain mandatory requirements of the law. Thus, IBM argues as the exclusive representative of all the employees in an appropriate collective
that there having been no similar petition pending before Med-Arbiter Manit, another bargaining unit for purposes of collective bargaining. These rights are found under
Article 242.
24
sole and exclusive bargaining agent. This democratic decision deserve utmost
Moreover,, a labor organizations attains the status of legitimacy only upon the respect. Again, it bears stressing that labor legislation seeks in the main to protect
issuance in its name of a Certificate of Registration by the Bureau of Labor Relations the interest of the members of the working class. It should never be used to subvert
pursuant to Articles 234 and 235 of the Labor Code. The procedure is not the only their will.
way by which a labor union may become legitimate, however. When an unregistered
union becomes a branch, local or chapter of a federation, some of the b. Cancellation of Union Certificate of Registration
aforementioned requirements for registration are no longer required. Section 3, Rule • Articles 245; 246; 247; 248; 250(j), last paragraph; 252; and 256, Labor Code.
II, Book V of the Implementing Rules of the Labor Code governs the procedure for
union affiliation. Article 245: [238] Cancellation of Registration.186 The certificate of registration of
any legitimate labor organization, whether national or local, may be cancelled by the
In upholding the legitimate status of IBM at SMFI as mentioned by the SolGen, the Bureau, after due hearing, only on the grounds specified in Article 239 hereof.
contention of the respondent that unless and until the issue on who is the legitimate
national president, of the Ilaw at Buklod ng Mangagawa is resolved, the petitioner Article 246: [238-A] Effect of a Petition for Cancellation of Registration.188 A
cannot claim that is has a valid charter certificate necessary for it to acquire legal petition for cancellation of union registration shall not suspend the proceedings for
personality is untenable. We wish to stress that the resolution of the said issue will certification election nor shall it prevent the filing of a petition for certification election.
not in any way affect the validity of the charter certificate issued by the IBM in favor
of the local union. It must be borne in mind that the said charter certificate was issued In case of cancellation, nothing herein shall restrict the right of the union to seek just
by the IBM in its capacity as a labor organization, a juridical entity which has a and equitable remedies in the appropriate courts.
separate and distinct legal personality from its members. When as in this case, there
is no showing that the Federation acting as a separate entity is questioning the Article 247: [239] Grounds for Cancellation of Union Registration.189 The following
legality of the issuance of the said charter certificate, the legality of the issuance of may constitute grounds for cancellation of union registration:
the same in favor of the local union is presumed. This, notwithstanding the alleged
controversy on the leadership of the federation. (a) Misrepresentation, false statement or fraud in connection with the adoption or
ratification of the constitution and by-laws or amendments thereto, the minutes of
In affirming the comment of the SolGen, SC ruled that respondent's Comment to the ratification, and the list of members who took part in the ratification;
petition indicates that in the election of officers held to determine the representatives
of IBM, the faction of Mr. Meron lost to the group of Mr. Edilberto Galvez, and the (b) Misrepresentation, false statements or fraud in connection with the election of
latter was acknowledged as the duly elected IBM National President. Thus, the officers, minutes of the election of officers, and the list of voters;
authority of Mr. Galvez to sign the charter certificate of IBM at SMFI, as President of
(c) Voluntary dissolution by the members.
the IBM Federation, can no longer be successfully questioned. A punctilious
examination of the records presents no evidence to the contrary and petitioner,
Article 248: [239-A] Voluntary Cancellation of Registration.190 The registration of
instead of squarely refuting this point, skirted the issue by insisting that the mere
a legitimate labor organization may be cancelled by the organization itself: Provided,
presence of two contending factions in the IBM prevents the issuance of a valid and
That at least two-thirds of its general membership votes, in a meeting duly called for
authentic charter certificate in favor of IBM at SMFI.
that purpose to dissolve the organization: Provided, further, That an application to
cancel registration is thereafter submitted by the board of the organization, attested
Lastly,, the certification election sought to be stopped by petitioner is fait accompli.
to by the president thereof.
The monthly paid rank-and-file employees of SMFI have already articulated their
choice as to who their collective bargaining agent should be. In the certification
election held on August 20, 1994, the SMFI workers chose IBM at SMFI to be their
25
Article 252: [242-A] Reportorial Requirements.194 The following are documents
required to be submitted to the Bureau by the legitimate labor organization Takata’s contention:
concerned: - In the May 1, 2009 organizational meeting of respondent, only 68 attendees
signed the attendance sheet, and which number comprised only 17% of the
total number of the 396 regular rank- and-file employees which respondent
193 As amended by Sec. 17 of R.A. No. 6715 (1989).194 As amended by Sec. 7 of
sought to represent, and hence, respondent failed to comply with the 20%
R.A. No. 9481 (2007). minimum membership requirement;
- The document “Pangalan ng mga Kasapi ng Unyon” bore no signatures of
(a) Its constitution and by-laws, or amendments thereto, the minutes of ratification, the alleged 119 union members;
and the list of members who took part in the ratification of the constitution and by- - The employees were not given sufficient information on the documents they
laws within thirty (30) days from adoption or ratification of the constitution and by- signed;
laws or amendments thereto; - The document “Sama-Samang Pahayag ng Pagsapi” was not submitted at
the time of the filing of respondent's application for union registration; that
the 119 union members were actually only 117;
(b) Its list of officers, minutes of the election of officers, and list of voters within thirty
- The total number of petitioner's employees as of May 1, 2009 was 470, and
(30) days from election; not 396 as respondent claimed.

(c) Its annual financial report within thirty (30) days after the close of every fiscal SALAMAT’s contention:
year; and - Denied the charge and claimed that the 119 union members were more than
the 20% requirement for union registration.
(d) Its list of members at least once a year or whenever required by the Bureau. - The document “Sama-Samang Pahayag ng Pagsapi sa Unyon” which it
presented in its petition for certification election supported their claim of 119
Failure to comply with the above requirements shall not be a ground for cancellation members.
- Takata was estopped from assailing its legal personality as it agreed to a
of union registration but shall subject the erring officers or members to suspension,
certification election and actively participated in the pre-election conference
expulsion from membership, or any appropriate penalty. of the certification election proceedings.
- The union members were informed of the contents of the documents they
Article 256: [245-A] Effect of Inclusion as Members of Employees Outside the signed and that the 68 attendees to the organizational meeting constituted
Bargaining Unit.198 The inclusion as union members of employees outside the more than 50% of the total union membership, hence, a quorum existed for
bargaining unit shall not be a ground for the cancellation of the registration of the the conduct of the said meeting.
union. Said employees are automatically deemed removed from the list of DOLE:
● Granted petition for cancellation of respondent's certificate of registration.
membership of said union.
Thus, the Union Certificate of Registration is revoked and/or cancelled and
SALAMAT is delisted from the roll of legitimate labor organization of this
Takata (Philippines) Corporation v. Bureau of Labor Relations, G.R. No. office.
192676, June 4, 2014. ● 68 employees who attended the organizational meeting was obviously less
than 20% of the total number of 396 regular rank-and-file employees which
FACTS: respondent sought to represent, hence, short of the union registration
On July 7, 2009, Takata Corp filed with DOLE a petition for cancellation of Certificate requirement;
of Union Registration of Samahang Lakas Manggagawa ng Takata (SALAMAT) on ● attendance sheet which contained the signatures and names of the union
the ground that was guilty of misrepresentation, false statements, and fraud with members totalling to 68 contradicted the list of names stated in the
respect to the number of those who participated in their organizational meeting, the document denominated as “Pangalan ng mga Kasapi ng Unyon.”
adoption and ratification of its Constitution and by-laws, and in the election of its
officers.
26
● The document “Sama-Samang Pahayag ng Pagsapi” was not attached to membership in the union and not to the list of workers who participated in the
the application for registration as it was only submitted in the petition for organizational meeting.
certification election filed by respondent at a later date.
● The proceedings in the cancellation of registration and certification elections
Indeed, Article 234 (b) and (c) provide for separate requirements, which must be
are two different and entirely separate and independent proceedings which
were not dependent on each other. submitted for the union's registration, and which respondent did submit. Here, the
total number of employees in the bargaining unit was 396, and 20% of which was
Bureau of Labor Relations’s Ruling: about 79. Respondent submitted a document entitled “Pangalan ng Mga Kasapi ng
● Reversed the decision of DOLE and ordered that SALAMAT shall remain in Unyon” showing the names of 119 employees as union members, thus respondent
the roster of labor organizations. sufficiently complied even beyond the 20% minimum membership requirement.
● Takata failed to prove that respondent deliberately and maliciously Respondent also submitted the attendance sheet of the organizational meeting
misrepresented the number of rank-and-file employees. It pointed out
which contained the names and signatures of the 68 union members who attended
petitioner's basis for the alleged non-compliance with the minimum
membership requirement for registration was the attendance of 68 members the meeting. Considering that there are 119 union members which are more than
to the May 1, 2009 organizational meeting supposedly comprising only 17% 20% of all the employees of the bargaining unit, and since the law does not provide
of the total 396 regular rank-and-file employees. for the required number of members to attend the organizational meeting, the 68
● The list of employees who participated in the organizational meeting was a attendees which comprised at least the majority of the 119 union members would
separate and distinct requirement from the list of the names of members already constitute a quorum for the meeting to proceed and to validly ratify the
comprising at least 20% of the employees in the bargaining unit; and that Constitution and By-laws of the union. Thus, there’s, no basis for petitioner to
there was no requirement for signatures opposite the names of the union
contend that grounds exist for the cancellation of respondent's union registration.
members; and there was no evidence showing that the employees assailed
their inclusion in the list of union members. For fraud and misrepresentation to be grounds for cancellation of union registration
under Article 239 of the Labor Code, the nature of the fraud and misrepresentation
CA: Denied the petition and affirmed the decision of the BLR. must be grave and compelling enough to vitiate the consent of a majority of union
members.
ISSUE:
Whether or not SALAMAT’s Certificate of Union Registration should be cancelled -
NO. The Heritage Hotel Manila v. National Union of Workers in the Hotel,
Restaurant and Allied Industries – Hertiage Hotel Manila Supervisors Chapter
HELD: (NUWHRAINHHMSC), G.R. No. 178296, January 12, 2011.
Petitioner's allegation of misrepresentation and fraud is based on its claim that
during the organizational meeting on May 1, 2009, only 68 employees attended, FACTS:
while respondent claimed that it has 119 members as shown in the document On October 11, 1995, respondent filed with the DOLE a petition for certification
denominated as “Pangalan ng mga Kasapi ng Unyon;” hence, respondent election. The Med-Arbiter granted the petition on February 14, 1996 and ordered the
misrepresented on the 20% requirement of the law as to its membership. holding of a certification election. On appeal, the DOLE Secretary, in a Resolution
dated August 15, 1996, affirmed the Med-Arbiter’s order and remanded the case to
SC ruled that It does not appear in Article 234 (b) of the Labor Code that the the Med-Arbiter for the holding of a preelection conference on February 26, 1997.
attendees in the organizational meeting must comprise 20% of the employees in the Petitioner filed a motion for reconsideration, but it was denied on September 23,
bargaining unit. In fact, even the IRR of the Labor Code does not so provide. It is 1996.
only under Article 234 (c) that requires the names of all its members comprising at
least twenty percent (20%) of all the employees in the bargaining unit where it seeks The preelection conference was not held as initially scheduled; Petitioner moved for
to operate. Clearly, the 20% minimum requirement pertains to the employees’ the dismissal on the petition due to alleged repeated non-appearance of respondent.
27
The latter agreed to suspend proceedings until further notice. The preelection 1997, 1998, and 1999, its updated list of officers, and its list of members for
conference resumed on January 29, 2000. Later on, Petitioner discovered that the years 1995, 1996, 1997, 1998, and 1999; (d) the petition is already moot
respondent had failed to submit to the Bureau of Labor Relations (BLR) its annual and academic, considering that the certification election had already been
financial report for several years and the list of its members since it filed its held, and the members had manifested their will to be represented by
registration papers in 1995. Thus, petitioner filed a Petition for Cancellation of respondent.
Registration of respondent, on the ground of the non-submission of the said - Citing National Union of Bank Employees v. Minister of Labor, et al. and
documents. Petitioner prayed that respondent’s Certificate of Creation of Samahan ng Manggagawa sa Pacific Plastic v. Hon. Laguesma, the Med-
Local/Chapter be cancelled and its name be deleted from the list of legitimate labor Arbiter held that the pendency of a petition for cancellation of registration is
organizations. It further requested the suspension of the certification election not a bar to the holding of a certification election.
proceedings.
DOLE: Dismissed the appeal and the Regional Director’s reasoning:
Nevertheless, the certification election pushed through on June 23, 2000. ● While finding that respondent had indeed failed to file financial reports and
Respondent emerged as the winner. Thus, petitioner filed a Protest with Motion to the list of its members for several years, he, nonetheless, denied the
Defer Certification of Election Results and Winner. petition, ratiocinating that freedom of association and the employees’ right
to self-organization are more substantive considerations.
Petitioner’s contention: ● The fact that respondent won the certification election and that it had already
- The certification election held on June 23, 2000 was an exercise in futility been certified as the exclusive bargaining agent of the supervisory
because, once respondent’s registration is cancelled, it would no longer be employees. In view of the foregoing, Regional Director Maraan—while
entitled to be certified as the exclusive bargaining agent of the supervisory emphasizing that the non-compliance with the law is not viewed with favor—
employees. considered the belated submission of the annual financial reports and the
- Some of respondent’s members were not qualified to join the union because list of members as sufficient compliance thereof and considered them as
they were either confidential employees or managerial employees. having been submitted on time.
- Prayed that the certification of the election results and winner be deferred
until the petition for cancellation shall have been resolved, and that Bureau of Labor Relations: Director Cacdac’s inhibited himself from the case
respondent’s members who held confidential or managerial positions be being the previous counsel of respondent. Thus, DOLE Secretary Sto. Tomas took
excluded from the supervisors’ bargaining unit. cognizance of the appeal by dismissing the same holding that the constitutionally
guaranteed freedom of association and right of workers to self-organization
Respondent’s contention: outweighed respondent’s noncompliance with the statutory requirements to maintain
- The petition was filed primarily to delay the conduct of the certification its status as a legitimate labor organization.
election, the respondent’s certification as the exclusive bargaining
representative of the supervisory employees, and the commencement of CA’s Ruling: Denied the petition.
bargaining negotiations.
- Prayed for the dismissal of the petition for the following reasons: (a) ISSUE:
petitioner is estopped from questioning respondent’s status as a legitimate Whether or not failure to comply with the statutory requirement, filing financial
labor organization as it had already recognized respondent as such during reports and the list of its members, sufficient ground for the cancellation of
the preelection conferences; (b) petitioner is not the party-in-interest, as the registration of the respondent as a labor union -NO.
union members are the ones who would be disadvantaged by the non-
submission of financial reports; (c) it has already complied with the HELD:
reportorial requirements, having submitted its financial statements for 1996,
28
Petitioner also insists that respondent’s registration as a legitimate labor union Mariwasa Siam Ceramics, Inc. v. The Secretary of the Department of Labor and
should be cancelled. Petitioner posits that once it is determined that a ground Employment, G.R. No. 183317, December 21, 2009.
enumerated in Article 239 of the Labor Code is present, cancellation of registration
should follow; it becomes the ministerial duty of the Regional Director to cancel the FACTS:
registration of the labor organization, hence, the use of the word "shall." Petitioner On May 4, 2005, respondent Samahan Ng Mga Manggagawa Sa Mariwasa Siam
points out that the Regional Director has admitted in its decision that respondent Ceramics, Inc. (SMMSC-Independent) was issued a Certificate of Registration as a
failed to submit the required documents for a number of years; therefore, legitimate labor organization by DOLE.
cancellation of its registration should have followed as a matter of course.
Mariwasa Siam Ceramics, Inc. filed a Petition for Cancellation of Union Registration
SC ruled that the non-compliance should not be a ground for the cancellation. against respondent, claiming that the latter violated Article 234 of the Labor Code
Articles 238 and 239 of the Labor Code provide that failure to file financial reports for not complying with the 20% requirement, and that it committed massive fraud
and the list of its members are grounds for the cancellation of Union Organization. and misrepresentation in violation of Article 239 of the same code. The petitioner
However, consideration must be taken of the fundamental rights guaranteed by insists that respondent failed to comply with the 20% union membership requirement
Article XIII, Section 3 of the Constitution, i.e., the rights of all workers to self- for its registration as a legitimate labor organization because of the disaffiliation from
organization, collective bargaining and negotiations, and peaceful concerted the total number of union members of 102 employees who executed affidavits
activities. Labor authorities should bear in mind that registration confers upon a recanting their union membership.
union the status of legitimacy and the concomitant right and privileges granted by
law to a legitimate labor organization, particularly the right to participate in or ask for Regional Director of DOLE: issued an Order granting the petition, revoking the
certification election in a bargaining unit. Thus, the cancellation of a certificate of registration of respondent, and delisting it from the roster of active labor unions.
registration is the equivalent of snuffing out the life of a labor organization. Bureau of Labor Relations: granted the appeal. Reversed decision of Regional
Director, SMMSC-Independent remains in the roster of legitimate labor
Moreover, submission of the required documents is the duty of the officers of the organizations.
union. It would be unreasonable for this Office to order the cancellation of the union CA: denied the petition.
and penalize the entire union membership on the basis of the negligence of its
officers.As aptly ruled by respondent Bureau of Labor Relations Director Noriel: "The ISSUES:
rights of workers to self-organization finds general and specific constitutional 1. Whether or not SMMSC-Independent failed to comply with 20% requirement
guarantees. x x x Such constitutional guarantees should not be lightly taken much -NO.
less nullified. A healthy respect for the freedom of association demands that acts 2. whether or not the withdrawal of 31 union members from NATU affected the
imputable to officers or members be not easily visited with capital punishments petition for certification election insofar as the 30% requirement is
against the association itself." concerned -NO.

At any rate, we note that on 19 May 2000, appellee had submitted its financial HELD:
statement for the years 1996-1999. With this submission, appellee has substantially 1. Even assuming the veracity of the affidavits of recantation, the legitimacy of
complied with its duty to submit its financial report for the said period. To rule respondent as a labor organization must be affirmed. While it is true that the
differently would be to preclude the union, after having failed to meet its periodic withdrawal of support may be considered as a resignation from the union,
obligations promptly, from taking appropriate measures to correct its omissions. For the fact remains that at the time of the union's application for registration,
the record, we do not view with favor appellee’s late submission. Punctuality on the the affiants were members of respondent and they comprised more than the
part of the union and its officers could have prevented this petition. required 20% membership for purposes of registration as a labor union.
Article 234 of the Labor Code merely requires a 20% minimum membership
29
during the application for union registration. It does not mandate that a union The Heritage Hotel Manila v. Pinag-isang Galing at Lakas ng mga Manggagawa
must maintain the 20% minimum membership requirement all throughout its sa Heritage Manila (PIGLASHERITAGE), G.R. No. 177024, October 30, 2009.
existence.
FACTS:
2. It appears undisputably that the 31 union members had withdrawn their The Heritage Hotel Employees Union (HHE) was formed in 2000 by certain rank and
support to the petition before the filing of said petition. It would be otherwise file employees of herein petitioner Heritage Hotel Manila, to which the Department
if the withdrawal was made after the filing of the petition for it would then be of Labor and Employment-National Capital Region issued a certificate of
presumed that the withdrawal was not free and voluntary. The presumption registration. HHE filed a petition for certification election which petitioner opposed
would arise that the withdrawal was procured through duress, coercion or on the ground that HHE misrepresented itself to be an independent union, when in
for valuable consideration. In other words, the distinction must be that fact it was a local chapter of the National Union of Workers in Hotel and Restaurant
withdrawals made before the filing of the petition are presumed voluntary and Allied Industries (NUWHRAIN). It was also alleged that such omitted disclosure
unless there is convincing proof to the contrary, whereas withdrawals made was intentional because petitioner’s supervisors union was already affiliated with it.
after the filing of the petition are deemed involuntary. Petitioner also filed a petition to cancel the union’s registration certificate. The Med-
Arbiter nevertheless granted HHE’s petition for certification election.
The reason for such distinction is that if the withdrawal or retraction is made
before the filing of the petition, the names of employees supporting the Petitioner appealed to the Secretary of Labor but it was denied as well as its motion
petition are supposed to be held secret to the opposite party. Logically, any for reconsideration, prompting petitioner to file a petition for certiorari with the Court
such withdrawal or retraction shows voluntariness in the absence of proof of Appeals. The CA issued a writ of injunction against the holding of HHE’s
to the contrary. Moreover, it becomes apparent that such employees had certification election until the petition for cancellation of its registration shall have
not given consent to the filing of the petition, hence the subscription been resolved with finality.
requirement has not been met.
In 2003, another union, herein respondent Pinag-Isang Galing at Lakas ng mga
When the withdrawal or retraction is made after the petition is filed, the Manggagawa sa Heritage Manila (PIGLAS), was formed by certain rank and file
employees who are supporting the petition become known to the opposite employees of petitioner at a meeting. PIGLAS was issued its registration certificate
party since their names are attached to the petition at the time of filing. by the DOLE-NCR in 2004. HHE later on adopted a resolution for its dissolution and
Therefore, it would not be unexpected that the opposite party would use foul then filed a petition for cancellation of its union registration.
means for the subject employees to withdraw their support.
PIGLAS filed a petition for certification election which petitioner opposed alleging
The affidavits of recantation were executed after the identities of the union that the new union’s officers and members were also those who comprised the old
members became public, i.e., after the union filed a petition for certification union. Petitioner likewise alleged that PIGLAS was formed to circumvent the CA’s
election on May 23, 2005, since the names of the members were attached injunction earlier issued. The Med-Arbiter nevertheless granted the petition for
to the petition. The purported withdrawal of support for the registration of certification election.
the union was made after the documents were submitted to the DOLE,
Region IV-A. The logical conclusion, therefore, following jurisprudence, is Petitioner filed a petition to cancel PIGLAS’ registration, claiming that the documents
that the employees were not totally free from the employer's pressure, and submitted with the union’s application for registration bore the following false
so the voluntariness of the employees' execution of the affidavits becomes information:
suspect. ● The List of Members showed that the PIGLAS union had 100 union
members;

30
● The Organizational Minutes said that 90 employees attended the meeting proved, the labor union acquires none of the rights accorded to registered
on December 10, 2003; organizations.
● The Attendance Sheet of the meeting of December 10, 2003 bore the
signature of 127 members who ratified the union’s Constitution and By- The discrepancies can be explained. While it appears that in the minutes of the
Laws; and December 10, 2003 organizational meeting, only 90 employees responded to the
● The Signature Sheet bore 128 signatures of those who attended that roll call at the beginning, it cannot be assumed that such number could not grow to
meeting. 128 as reflected on the signature sheet for attendance. The meeting lasted 12 hours
from 11:00am to 11:00pm. There is no evidence that the meeting hall was locked
Petitioner alleged that the misrepresentation was evidenced by the discrepancy in up to exclude late attendees. As to the fact that only 127 members ratified the
the number of union members appearing in the application and the list as well as union’s constitution and by-laws when 128 signed the attendance sheet, it cannot
the number of signatories to the attendance and signature sheets. The minutes be assumed that all those who attended approved of such. Any member had the
reported only 90 employees attended the meeting. Petitioner also alleged that 33 right to hold out and refrain from ratifying those documents or to simply ignore the
members of PIGLAS were members of HHE, which is in violation of the policy process. The Labor Code and its implementing rules do not require that the number
against dual unionism. of members appearing on the documents in question should completely dovetail.
For as long as the documents and signatures are shown to be genuine and regular
DOLE-NCR: denied petitioner’s petition to cancel PIGLAS’ registration because the and the constitution and by-laws democratically ratified, the union is deemed to have
discrepancies in the number of members in the application’s supporting documents complied with registration requirements.
were not material and did not constitute misrepresentation. The dual unionism is
also not a ground for canceling registration, since the members of HHE simply The discrepancy in the list of members (showing only 100 members) and the
exercised their right to self-organization and freedom of association when they signature and attendance sheets (showing 127 or 128 members) submitted is
joined PIGLAS. The Bureau of Labor Relations affirmed the DOLE-NCR, by immaterial. A comparison of the documents shows that except for six members, the
reasoning that PIGLAS’ organization meeting lasted for 12 hours. Thus, it was names found in the list are also in the attendance and signature sheets. PIGLAS
possible for the number of attendees to have increased as the meeting progressed. more than complied with the 20% requirement since only 50 employees out of 250
Besides, the union only needed 50 members of the total of 250 employees in the employees in the bargaining unit were required to unionize.
bargaining unit to comply with the 20% membership requirement. Petitioner filed a
petition for certiorari with the Court of Appeals but it was denied as well as Labor laws are liberally construed in favor of labor especially if doing so would affirm
petitioner’s motion for reconsideration. Hence, this petition for review under Rule 45. its constitutionally guaranteed right to self-organization. PIGLAS union’s supporting
documents reveal the unmistakable yearning of petitioner company’s rank and file
ISSUES: employees to organize. This yearning should not be frustrated by inconsequential
1. Whether or not the union made fatal misrepresentation in its application for union technicalities.
registration –NO. 2. NO. The right of any person to join an organization also includes the right to leave
2. Whether or not dual unionism is a ground for canceling a union’s registration – that organization and join another one.HHE union ceased to exist, its certificate of
NO. registration being already cancelled.

HELD: S.S. Ventures International, Inc. v. S.S. Ventures Labor Union (SSVLU), G.R.
1. Petitioner has no evidence of the alleged misrepresentation. The discrepancies No. 161690, July 23, 2008.
alone cannot be taken as indication that PIGLAS misrepresented the information FACTS:
Petitioner S.S. Ventures International, Inc. (Ventures), a PEZA- registered export
contained in these documents. Charges of fraud and misrepresentation should be
firm with principal place of business at Phase I-PEZA- Bataan Export Zone,
clearly established by evidence and surrounding circumstances because once it is
31
Mariveles, Bataan, is in the business of manufacturing sports shoes. Respondent application for registration and the supporting documents, such as the adoption or
S.S. Ventures Labor Union (Union) is a labor organization registered with the DOLE. ratification of the constitution and by-laws or amendments thereto and the minutes
of ratification of the constitution or by-laws, among other documents.
March 21, 2000, the Union filed with DOLE-Region III a petition for certification The evidence presented by Ventures consist mostly of separate hand-written
election in behalf of the rank-and-file employees statements of 82 employees who alleged that they were unwilling or harassed
signatories to the attendance sheet of the organizational meeting. However these
evidence was presented seven months after the union filed its petition for
August 21, 2000, Ventures filed a Petition to cancel the Union’s certificate of cancellation of registration. Hence these statements partake of the nature of
registration alleging that the Union deliberately and maliciously included the names withdrawal of union membership executed after the Union’s filing of a petition for
of more or less 82 former employees no longer connected with Ventures in its list of certification election on March 21, 2000. We have said that the employees’
members who attended the organizational meeting and in the adoption/ratification withdrawal from a labor union made before the filing of the petition for certification
of its constitution and by-laws; that No organizational meeting and ratification election is presumed voluntary, while withdrawal after the filing of such petition is
actually took place; and the Union’s application for registration was not supported considered to be involuntary and does not affect the same. Now then, if a withdrawal
by at least 20% of the rank-and-file employees of Ventures. from union membership done after a petition for certification election has been filed
does not vitiate such petition, it is but logical to assume that such withdrawal cannot
work to nullify the registration of the union. The Court is inclined to agree with the
Regional Director of DOLE- Region III favored Ventures and resolved to Cancel the CA that the BLR did not abuse its discretion nor gravely err when it concluded that
Certificate of the union. On appeal, the BLR Director granted the Union’s appeal and the affidavits of retraction of the 82 members had no evidentiary weight.
reversing the decision of RD. Ventures went to the CA. The CA dismissed Ventures’
petition as well as the MR. Hence, this petition for review
The registration or the recognition of a labor union after it has submitted the
corresponding papers is not ministerial on the part of the BLR. It becomes
ISSUE: mandatory for the BLR to check if the requirements under Art. 234 of the Labor Code
Whether the registration of the Union must be cancelled. have been sedulously complied with. If the union’s application is infected by
falsification and like serious irregularities, especially those appearing on the face of
the application and its attachments, a union should be denied recognition as a
RULING: legitimate labor organization. The issuance to the Union of Certificate of
NO. The right to form, join, or assist a union is specifically protected by Art. XIII, Registration, in the case at bar, necessarily implies that its application for registration
Section 3 of the Constitution and such right, according to Art. III, Sec. 8 of the and the supporting documents thereof are prima facie free from any vitiating
Constitution and Art. 246 of the Labor Code, shall not be abridged. Once registered irregularities.
with the DOLE, a union is considered a legitimate labor organization endowed with
the right and privileges granted by law to such organization. While a certificate of The relevance of the 82 individuals’ active participation in the Union’s organizational
registration confers a union with legitimacy with the concomitant right to participate meeting and the signing ceremonies thereafter comes in only for purposes of
in or ask for certification election in a bargaining unit, the registration may be determining whether or not the Union, even without the 82, would still meet what Art.
canceled or the union may be decertified as the bargaining unit, in which case the 234(c) of the Labor Code requires to be submitted, requiring that the union applicant
union is divested of the status of a legitimate labor organization. Among the grounds must file the names of all its members comprising at least twenty percent (20%) of
all the employees in the bargaining unit where it seeks to operate.
for cancellation is the commission of any of the acts enumerated in Art. 239(a) of
the Labor Code, such as fraud and misrepresentation in connection with the
adoption or ratification of the union’s constitution and like documents. The Court, In its union records on file with this Bureau, respondent union submitted the names
has in previous cases, said that to decertify a union, it is not enough to show that of 542 members. This number easily complied with the 20% requirement, be it 1,928
or 2,202 employees in the establishment. Even subtracting the 82 employees from
the union includes ineligible employees in its membership. It must also be shown
542 leaves 460 union members, still within 440 or 20% of the maximum total of 2,202
that there was misrepresentation, false statement, or fraud in connection with the rank-and-file employees of the employer Venture.
32
Whatever misgivings the petitioner may have with regard to the 82 dismissed APC filed a Motion for Reconsideration/Appeal regarding this Decision of the DOLE-
employees is better addressed in the inclusion-exclusion proceedings during a pre- NCR. In a Resolution dated 18 July 2001, the Bureau of Labor Relations (BLR)
election conference. The issue surrounding the involvement of the 82 employees is denied the appeal, affirming the rationale of the DOLE-NCR.4
a matter of membership or voter eligibility. It is not a ground to cancel union
registration. APC then immediately filed a Petition for Certiorari with the Court of Appeals,
imputing grave abuse of discretion on the part of the BLR in denying its appeal.
However, the petition was dismissed outright by the Court of Appeals on the ground
For fraud and misrepresentation to be grounds for cancellation of union registration that APC had "failed to avail of the remedy of a prior Motion for Reconsideration"
under Article 239, the nature of the fraud and misrepresentation must be grave and before the filing of the certiorari petition, which step, it stressed, is a "condition sine
compelling enough to vitiate the consent of a majority of union members. qua non to the filing of a petition for certiorari."5

WHEREFORE, the petition is DENIED APC filed a Motion for Reconsideration dated 5 February 2002, but this too was
denied by the Court of Appeals in a Resolution dated 13 September 2002. This time,
the appellate court ruled that the Motion for Reconsideration was "totally defective,"
Air Philippines Corporation v. Bureau of Labor relations, G.R. No. 155395, for failing to contain the proof of service or registry return receipts to the
June 22, 2006. respondents. The Court of Appeals even noted that the Affidavit of Service attached
to the Motion for Reconsideration "failed to indicate the registry return receipts of the
registered mails to the respondents."6

FACTS: Hence, the present petition.

The case initially centered on the union registration of respondent Air Philippines APC argues that its petition before the Court of Appeals involved mere questions of
Flight Attendants Association (APFLAA). APFLAA filed on 17 March 1999 a petition law, among which is whether APFLAA’s union registration may be cancelled
for certification election as the collective bargaining representative of the flight considering that the union is allegedly composed of a mixture of supervisory and
attendants of APC. After the Med-Arbiter rendered a ruling ordering the holding of a rank-and-file employees. It is posited that questions of law may be raised directly in
certification election, such election was held on 5 August 1999, with majority of the a petition for certiorari without need of a prior motion for reconsideration.
votes cast in favor of APFLAA.
However, it is clear from the petition filed by APC before the Court of Appeals that
On 25 November 1999, APC filed a Petition for De-Certification and Cancellation of the issues involved do not consist of questions of law only. It is insisted therein that
Union Registration against APFLAA with the DOLE. APC alleged that APFLAA could employees holding the position of Lead Cabin Attendants are supervisory
not be registered as a labor organization, as its composition consisted of "a mixture employees and hence disallowed from joining a union of rank-and-file employees.8
of supervisory and rank-and-file flight attendants." Particularly, APC alleged that On the other hand, APFLAA countered before the DOLE-NCR and the BLR that only
flight attendants holding the position of "Lead Cabin Attendant," which according to rank-and-file flight attendants comprised its membership.9 Thus, the very question
it is supervisory in character, were among those who comprised APFLAA. of whether Lead Cabin Attendants are indeed supervisory employees appears to be
factual in nature, the proper resolution of which necessitates a factual determination
On 18 July 2001, the DOLE-National Capital Region (NCR) Regional Director Alex of the actual duties of Lead Cabin Attendants. Indeed, APC made reference therein
E. Maraan rendered a Decision dismissing the petition. The DOLE-NCR held that to such documents as an employee’s manual in support of its argument, 10
Article 245 of the Labor Code, which states that supervisory employees are not documents that would evidently require factual evaluation before accorded proper
eligible for membership in labor organizations of rank-and-file employees, does not evidentiary value.
provide a ground for cancellation of union registration, which is instead governed by
Article 239 of the Labor Code.3 There is admittedly some leeway for the Court of Appeals if it was so minded to give
due course to APC’s petition, notwithstanding the failure to file a motion for
reconsideration. Yet ultimately, the determination of whether or not to admit a

33
petition attended with such defect falls within the sound discretion of the Court of qualified to become a legitimate labor organization by reason of its mixed
Appeals. composition of rank-and-file and supervisory employees; and that APFLAA
committed misrepresentation by making it appear that its composition was
Should the Court of Appeals decide, as it did, to dismiss the petition outright on such composed purely of rank-and-file employees. Such misrepresentation (if it can be
ground, it would commit no reversible error of law nor any grave abuse of discretion, called as such) as alleged by APC, is not conformable to Article 239 (a) and (c) of
considering that the rule requiring the filing of a motion for reconsideration before the Labor Code. Indeed, it appears from the record that APC instead devoted the
resorting to the special civil action of certiorari is well entrenched in jurisprudence. bulk of its arguments in establishing that supervisory employees comprised part of
the membership of APFLAA, a ground which is not sufficient to cause the
It also does not escape the attention of the Court that the Motion for Reconsideration cancellation of union registration. And this is of course all under the assumption that
filed by APC before the Court of Appeals was itself fatally defective, allowing the Lead Cabin Attendants are indeed supervisory employees, a claim consistently
appellate court to deny the same without having to evaluate its substantial denied by APFLAA and which was not confirmed by either the DOLE-NCR or the
arguments. The action of the appellate court relative to APC’s missteps is consistent BLR.
with procedural rules.
There may be remedies available to enforce the proscription set forth in Article 245
Still, the Court has deigned to give a close look at the substantial arguments raised of the Labor Code on supervisory employees joining the union of rank-and-file
in APC’s petition before the Court of Appeals. employees. But consistent with jurisprudence, the rule under Article 245 barring
supervisory employees from joining the union of rank-and-file employees is not a
ISSUE: Whether or not the APFLAA could be registered ground for cancellation of union registration. Accordingly, we see no error on the
part of the DOLE-NCR and the BLR in having dismissed APC’s petition, and thus
HELD: Yes. no cause to compel the Court of Appeals to disregard APC’s procedural errors and
The DOLE-NCR Regional Director, in dismissing the petition for cancellation, cited accept the petition for certiorari.
our minute resolution in SPI Technologies Incorporated v. DOLE wherein the Court WHEREFORE, the petition is DENIED. Costs against petitioner.
observed that Article 245 of the Labor Code, the legal basis for the petition for SO ORDERED.
cancellation, merely prescribed the requirements for eligibility in joining a union and
did not prescribe the grounds for cancellation of union registration. 13 Since the filing c. Reportorial Requirements
of this petition, the Court has had occasion to rule, in Tagaytay Highlands • Article 252, Labor Code.
International Golf Club v. Tagaytay Highlands Employees Union-PGTWO,14 that
"[t]he inclusion in a union of disqualified employees is not among the grounds for
6. Foreign Activities
cancellation, unless such inclusion is due to misrepresentation, false statement or
fraud under the circumstances enumerated in Sections (a) and (c) of Article 239 15 of • Articles 284; 285; and 286, Labor Code.
the Labor Code."
7. Union-Member Relations
Clearly then, for the purpose of de-certifying a union, it is not enough to establish • Articles 250 and 260(a), Labor Code.
that the rank-and-file union includes ineligible employees in its membership.
Pursuant to Article 239 (a) and (c) of the Labor Code, it must be shown that there University of Santo Tomas v. Samahang Manggagawa ng UST (SM-UST), G.R.
was misrepresentation, false statement or fraud in connection with the adoption or No. 169940, September 14, 2009.
ratification of the constitution and by-laws or amendments thereto, the minutes of
ratification, or in connection with the election of officers, minutes of the election of Petitioner: University of Santo Tomas
officers, the list of voters, or failure to submit these documents together with the list Respondent: Samahang Manggagawa ng UST (SM-UST)
of the newly elected-appointed officers and their postal addresses to the BLR.17
FACTS:
In its Petition for De-certification and Cancellation of Union Registration, APC did
not impute on APFLAA such misrepresentation of the character necessitated under
Article 239 (a) and (c) of the Labor Code. APC merely argued that APFLAA was not
34
● Respondent Union was the authorized bargaining agent of the non- ● On the other hand, Union reduced its demands for the first year from P8,000.00
academic/non-teaching rank-and-file daily- and monthly-paid employees (about monthly salary increase per employee to P7,000.00, and from P75,000.00
619) of Petitioner UST. signing bonus to P60,000.00 for each employee, but UST insisted on its final
● October 2001: During formal negotiations for a new collective bargaining offer.
agreement (CBA) for the academic year 2001 through 2006, UST submitted its ● As a result, Union declared a deadlock and filed a notice of strike with the
"2001-2006 CBA Proposals.” National Conciliation and Mediation Board -National Capital Region (NCMB-
● December 2001: UST submitted its final offer on the economic proposals. The NCR).
proposals of Union and UST are compared below: ● Conciliation and mediation proved to be futile, such that in January 2002,
majority of Union’s members voted to stage a strike.
UNION UST
● However, the DOLE Secretary (SOLE) timely assumed jurisdiction over the
dispute, and the parties were summoned and heard on their respective claims,
A. ACADEMIC YEAR and were required to submit their respective position papers.
2001-2002
● May 31, 2002: the SOLE issued an Order:
1. Salary increase per P800.00 P1,000.00 - Given all the foregoing, we cannot follow the Union's formula and in effect
month disregard the School's two other bargaining units; to do so is a distortion of
economic reality that will not bring about long term industrial peace. We
2. Signing bonus P10,000. P10,000.0 cannot simply adopt the School's proposal in light of the parties' bargaining
00 0
history, particularly the pattern of increases in the last cycle.
3. Additional Christmas P2,000.0 P2,000.00 - Considering all these, we believe the following to be a fair and reasonable
bonus 0 resolution of the wage issue.
1st Year - P1,000.00/month
B. ACADEMIC YEAR 2nd Year - P2,000.00/month
2002-2003
3rd Year - P2,200.00/month
1. Salary increase per P1,500.0 P1,700.00 - These increases, at a three-year total of P68,337,600, are less than the
month 0 three (3)- year increases in the last CBA cycle to accommodate the School's
proven lack of capacity to afford a higher increase, but are still substantial
2. Additional Christmas P2,000.0 P2,000.00 enough to accommodate the workers' needs while taking into account the
bonus 0
symmetry that must be maintained with the wages of the other bargaining
3. Salary restructuring P6 million P6.19 units.
million - On a straight line aggregate of P5,200.00, the non-academic personnel will
receive P498.48 less than an Instructor I (member of the faculty union) who
C. ACADEMIC YEAR received an aggregate of P5,698.48, thus maintaining the gap between the
2003-2004
teaching and non-teaching personnel. The salary difference will as well be
1. Salary increase per P1,700.0 P2,000.00 maintained over the three (3)-year period of the CBA.
month 0 - An RFI employee (member of the union's bargaining unit) will receive a
monthly salary of P21,695.95 while an Instructor I (faculty union member)
2. Additional Christmas P2,000.0 P2,000.00 will have a salary of P22,948.00; while an RF5-5/A (member of the union's
bonus 0
bargaining unit) will receive a salary of P23,462.97 compared to an Asst.
Prof. 1 (faculty) who will receive P29,250.96.
35
- From a total cost of salary increases for the first year at P7,428,000, these 70% of the UST incremental tuition fee proceeds for AY 2001-2002.
costs will escalate to P22,284,000 in the second year, and to P38,625,000 Considering the fringe benefits being extended to UST employees, it is safe
at the third year. Given these figures, the amounts available for distribution to assume that the fringe benefits being extended to UST employees, it is
and the member of groups sharing these amounts, these increases are by safe to assume that the funds for such benefits need to be sourced from
no means minimal. UST's other revenues. xxx We examined UST audited financial statements
- A review of the past bargaining history of the parties shows that the School from 1997 to 2001 and found that UST's "other incomes" come from parking
as a matter of course grants a signing bonus. This ranged from P8,000.00 fees, rent income and interest income. It, likewise, derives income from
during the first three (3) years of the last CBA to P10,000.00 during the school operations. xxx
remaining two (2) years of the re-negotiated term. In this instance, the - Even if the other benefits of the faculty members were to be charged from
School's offer of P10,000.00 signing bonus is already reasonable the remaining balance of the Accumulated Excess of Revenues Over
considering that the School could have taken the position that no signing Expenses, there would still be sufficient amount to fund the other benefits
bonus is due on compulsory arbitration in line with the ruling in Meralco v. of the non-teaching personnel. xxx
Quisumbing et al., G.R. No. 127598, 27 January 1999. - However, while We subscribe to UST's position on "salary distortion", Our
● Union filed a motion for reconsideration but it was denied by the SOLE. earlier findings support UST's contention that the UST has substantial
● Thus, Union filed an original petition for certiorari with the Court of Appeals, accumulated income and thus, We deem it proper to award an increase, not
claiming that the awards made by the SOLE are not supported by the evidence in salary, to prevent any salary distortion, but in signing bonus. The arbitral
on record and are contrary to law and jurisprudence. award of P10,000 signing bonus per employee awarded by public Union is
● The CA ruled: hereby increased to P18,000.00.
- Based on UST Chief Accountant Antonio J. Dayag's Certification, the tuition ● UST filed an MR, which was denied. Hence, this petition.
fee increment for the SY 2001-2002 amounted to P101,036,330.37. From
this amount, the tuition fee adjustment amounting to P2,785,143.00 was ISSUES:
deducted leaving a net tuition fee increment of P98,251,189.36.
- Pursuant to Section 5 (2) RA 6728, 70% of P98,251,187.36 or 1. WON the CA gravely abused its discretion when it ruled that the members
P68,775,831.15 is the amount UST has to allocate for salaries, wages, of Union did not voluntarily and knowingly accept the arbitral award of the
allowances and other benefits of its 2,290 employees, categorized as SOLE. - NO
follows: 2. WON the CA has completely ignored the clear mandate and intention of RA
619 non-teaching personnel represented by UNION; 6728 otherwise known as the Government Assistance to Students and
1,452 faculty members represented by UST-Faculty Union (UST- Teachers in Private Education Act. - The SC need not rule on this issue.
FU); and 3. WON the CA gravely abused its discretion when it increased the signing
219 academic/administrative officials. The last group of employees bonus from P10,000.00 to P18,000.00. - YES
is excluded from the coverage of the two bargaining units.
- Based on the SOLE’s arbitral award for the first year (AY 2001-2002), HELD:
UNION would get P15,475,000.00 or 22.5% of P68,775,831.00 (or from the
70% of the tuition fee increment for AY 2001-2002) by approximating UST's 1. NO, the CA did not. UST alleges that, as of December 11, 2002, 526 regular
expense on the increment of salaries/wages, allowances and benefits of the non-academic employees - out of a total of 619 members - have decided to
non-teaching personnel. xxx unconditionally abide by the May 31, 2002 Order of SOLE. UST claims that it
- The allocation for salary increases, 13th month pay, signing bonus and began paying the wage adjustment and other benefits pursuant to the SOLE
Christmas bonus for UST's teaching and non-teaching employees, as well Order; and that to date, 572 out of the 619 members have been paid. UST
as the school officials, amount to P68.475 million. This represents almost argues that by their acceptance of the award and the resulting payments made
36
to them, the said Union members have ratified its offer and thus rendered moot - Union argues that RA 6728 does not provide that the increase or
the case before the CA. improvement of the salaries and fringe benefits of the employees should be
- Union echoes the CA ruling that they are not precluded from receiving exclusively funded from the income of UST which is derived from the
additional benefits that the courts may award later on, bearing in mind that increase in tuition fees. In fact, the statute has no application with respect
– “the employer and the employee do not stand on the same footing. to the manner of disposition of the other incomes (as distinguished from
Considering the country's prevailing economic conditions, the employee income derived from tuition fee increases) of UST, nor does it preclude or
oftentimes finds himself in no position to resist money proffered, thus, his exempt the latter from using its other income or part thereof to fund the cost
case becomes one of adherence and not of choice. of increases or improvements in the salaries and benefits of its employees.
- Union’s members were merely constrained to accept payment at the time. - The SC finds it unnecessary to rule on this matter. These fringe benefits are
Christmas was then just around the corner, and the Union members were included in the DOLE Secretary's award - an award which UST seeks to
in no position to resist the temptation to accept much-needed cash for use affirm in toto; this being so, it cannot now argue otherwise. Since it abides
during the most auspicious occasion of the year. Time and again, we have by the DOLE Secretary's award, which it finds "fair and equitable," it must
held that necessitous men are not, truly speaking, free men; but to answer raise the said amount through sources other than incremental tuition fee
a present emergency, will submit to any terms that the crafty may impose proceeds.
upon them. 3. YES, the CA did. UST contends that the CA’s award of additional signing bonus
- Besides, as individual components of a Union possessed of a distinct and is contrary to the nature and principle behind the grant of such benefit, which is
separate corporate personality, Union’s members should realize that in one given as a matter of discretion and cannot be demanded by right, a
joining the organization, they have surrendered a portion of their individual consideration paid for the goodwill that existed in the negotiations, which
freedom for the benefit of all the other members; they submit to the will of culminate in the signing of a CBA. UST claims that since this condition is absent
the majority of the members in order that they may derive the advantages in the parties' case, it was erroneous to have rewarded UNION with an increased
to be gained from the concerted action of all. Since the will of the members signing bonus.
is personified by its board of directors or trustees, the decisions it makes - Union argues that since no strike or any untoward incident occurred,
should accordingly bind them. Precisely, a labor Union exists in whole or in goodwill between the parties remained, which entitles UNION's members to
part for the purpose of collective bargaining or of dealing with employers receive their signing bonus. Besides, UNION asserts that since UST did not
concerning terms and conditions of employment. What the individual appeal the DOLE Secretary's award, it may not now asserts that since UST
employee may not do alone, as for example obtain more favorable terms did not appeal the DOLE Secretary's award, it may not now argue against
and conditions of work, the labor organization, through persuasive and its grant, the issue remaining being the propriety of the awarded amount;
coercive power gained as a group, can accomplish better. that is, whether or not it was proper for the CA to have raised it from
2. The SC need not rule on this issue. UST argues that the CA erred in ordering it P10,000.00 to P18,000.00.
to source part of its judgment award from the school's other income, claiming - A signing bonus is a grant motivated by the goodwill generated when a CBA
that RA 6728 does not compel or require schools to allocate more than 70% of is successfully negotiated and signed between the employer and the Union.
the incremental tuition fee increase for the salaries and benefits of its Here, no CBA was successfully negotiated by the parties. It is only because
employees. Citing an authority in education law (Sarmiento III), it stresses that: UST prays for this Court to affirm in toto the SOLE’s Order that we shall
"...if schools use any part of the 20% reserved for the upgrading of school allow an award of signing bonus. There would have been no other basis to
facilities to supplement the salaries of their academic and non-academic grant it if UST had not so prayed. We shall take it as a manifestation of
personnel, they would not only be violating the students' constitutional right to UST's liberality, which we cannot now allow it to withdraw. A bonus is a
quality education through "improvement and modernization" but also committing gratuity or act of liberality of the giver; when UST filed the instant petition
a serious infraction of the mandatory provisions of RA 6728." seeking the affirmance of the SOLE’s Order in its entirety, assailing only the

37
increased amount of the signing bonus awarded, it is considered to have - The general assembly was attended by members of the USTFU and, as
unqualifiedly agreed to grant the original award to UNION's members. admitted by the appellants, also by "non-USTFU members who are
members in good standing of the UST Academic Community Collective
WHEREFORE, the petition is PARTIALLY GRANTED. The signing bonus of Bargaining Unit"
EIGHTEEN THOUSAND PESOS (P18,000.00) per member of respondent - On this occasion, appellants were elected as USTFU's new set of officers
Samahang Manggagawa ng U.S.T. as awarded by the Court of Appeals is by acclamation and clapping of hands
REDUCED to TEN THOUSAND PESOS (P10,000.00). All other findings and - Upon a motion of one Atty. Lopez, admittedly not a member of USTFU, that
dispositions made by the Court of Appeals in its January 31, 2005 Decision and the USTFU Constitution and By-Laws (CBL) and "the rules of the election
September 23, 2005 Resolution in CA-G.R. SP No. 72965 are AFFIRMED. be suspended and that the election be held [on] that day"
● Respondents filed the instant petition seeking injunctive reliefs and the
UST Faculty Union (USTFU) v. Bitonio, G.R. No. 131235, November 16, 1999. nullification of the results
● Subsequently, petitioners and UST allegedly entered into another CBA covering
Petitioners: UST Faculty Union (Ustfu), Gil Y. Gamilla, Corazon Qui, Norma the period from 01 June 1996 to 31 May 2001
Calaguas, Irma Potenciano, Luz De Guzman, Remedios Garcia, Rene Arnejo, ● The med-arbiter issued a TRO against petitioners
Editha Ocampo, Cesar Reyes, Celso Nierra, Gliceria Baldres, Ma. Lourdes Medina, ● Public respondent, director of the BLR agreed with the med-arbiter that the Oct.
Hidelita Gabo, Mafel Ysrael, Laura Abara, Natividad Santos, Ferdinand Limos, 4 election was void
Carmelita Espina, Zenaida Famorca, Philip Aguinaldo, Benedicta Alava and Leoncio
Casal ISSUES:
Respondents: Dir. Benedicto Ernesto R. Bitonio Jr. of The Bureau of Labor 1. WON the October 4 election was valid. – NO
Relations, Med-Arbiter Tomas F. Falconitin of the National Capital Region, 2. WoN the suspension of the CBL was valid. – NO
Department of Labor And Employment (DOLE), Eduardo J. Mariño Jr., Ma. Melvyn
Alamis, Norma Collantes, Urbano Alabagia, Ronaldo Asuncion, Zenaida Burgos, HELD:
Anthony Cura, Fulvio M. Guerrero, Myrna Hilario, Teresita Meer, Fernando Pedrosa, 1. The election was not valid
Nilda Redoblado, Rene Sison, Evelyn Tirol and Rosie Alcantara ● The right to vote in the union is enjoyed only by union members
- A union election should be distinguished from a certification election, which
FACTS: is the process of determining, through secret ballot, the sole and exclusive
● Private Respondents are duly elected officers of the UST Faculty Union bargaining agent of the employees in the appropriate bargaining unit, for
(USTFU) purposes of collective bargaining
● The secretary general of USTFU posted a notice addressed to all USTFU - Specifically, the purpose of a certification election is to ascertain whether or
members announcing a general assembly to be held on 05 October 1996 not a majority of the employees wish to be represented by a labor
- Among others, the general assembly was called to elect USTFU's next set organization and, in the affirmative case, by which particular labor
of officers. organization
● On 02 October 1996, the secretary general of UST, upon the request of the - In a certification election, all employees belonging to the appropriate
various UST faculty club presidents, issued notices allowing all faculty members bargaining unit can vote
to hold a convocation on 04 October 1996 - A union member who likewise belongs to the appropriate bargaining unit is
- Denominated as a general faculty assembly, the convocation was supposed entitled to vote in said election. However, the reverse is not always true; an
to discuss the "state of the unratified UST-USTFU CBA" and "status and employee belonging to the appropriate bargaining unit but who is not a
election of USTFU officers" member of the union cannot vote in the union election, unless otherwise
● The convocation was held as scheduled (Oct. 4, 1996) authorized by the constitution and bylaws of the union.
38
- In this case, non-members participated in the assembly and in the voting Petitioners: Heirs of Teodolo M. Cruz, Mary Concepcion and Edgardo Cruz
● The election did not follow the procedures imposed by the CBL (counsel of the Union, not the 49 Union members – petitioners in a separate case
- The union's CBL is the fundamental law that governs the relationship consolidated with this one)
King Hongs: Court of Industrial Relations (CIR), Santiago Rice Mill and King Hong
between and among the members of the union. It is where the rights,
and Company
duties and obligations, powers, functions and authority of the officers as well
as the members are defined FACTS:
- First, the assembly was not called by the USTFU. It was merely a ● June 21, 1952: The Santiago Labor Union, composed of workers of the Santiago
convocation of faculty clubs, It was not convened in accordance with the Rice Mill, a business enterprise engaged in the buying and milling of palay at
provision on general membership meetings. It was in fact a gathering that Santiago, Isabela, and owned and operated by King Hong Co., Inc. (King Hong),
was called and participated in by management and non-union members filed before the CIR a petition for overtime pay, premium pay for night, Sunday
- Second, there was no commission on elections to oversee the election, as and holiday work, and for reinstatement of workers illegally laid off.
mandated by Sections 1 and 2 of Article IX of the USTFU's CBL ● The total sum claimed by the workers, P100,816.36 for overtime pay,
- Third, the purported election was not done by secret balloting, in violation of P19,350.00 for premium pay and P3,360.00 for differential pay under the
Section 6, Article IX of the USTFU's CBL, as well as Article 241 (c) of the Minimum Wage Law - amounted to P123,526.36.
Labor Code ● September 19, 1958: The CIR, thru Judge Emiliano G. Tabigne, dismissed the
2. The suspension of the CBL was not valid petition of the union for lack of merit and want of jurisdiction.
● Petitioners contend that the October 4, 1996 assembly "suspended" the union's ● Upon a motion for reconsideration, the CIR en banc, by a split decision of 3-2
CBL. They aver that the suspension and the election that followed were in vote, reversed the decision of the trial judge.
accordance with their "constituent and residual powers as members of the ● The Supreme Court affirmed the CIR en banc decision and remanded the
collective bargaining unit to choose their representatives for purposes of records for enforcement by the CIR.
collective bargaining." ● The Chief Examiner filed with the CIR his Partial Report of December 14, 1962,
● The general faculty assembly was not the proper forum to conduct the election wherein the judgment award in favor of the workers was determined and
of USTFU officers. Not all who attended the assembly were members of the computed as totaling PHP423,756.74. Both the union members and King Hong
union; some, apparently, were even disqualified from becoming union members, opposed the report.
since they represented management ● Judge Tabigne, as the trial judge, ordered a hearing thereon on December 22,
● The person who moved for the suspension of USTFU's CBL was not a member 1962, as a condition precedent to execution of the judgment. Such Report was
of USTFU. Allowing a non-union member to initiate the suspension of a union's submitted for resolution and approval at the hearing of December 22, 1962, but
CBL, and non-union members to participate in a union election on the premise the records before us fail to show that the trial judge ever acted on or approved
that the union's CBL had been suspended in the meantime, is incompatible with the Report.
the freedom of association and protection of the right to organize ● Before and after the submittal of the Chief Examiner's Report of December 14,
● The grievances of the petitioners could have been brought up and resolved in 1962, the union pressed for execution of the final judgment in favor of its
accordance with the procedure laid down by the union's CBL claimants-members.
- It filed, furthermore, on December 20, 1962, an Urgent Motion for
WHEREFORE, the Petition is hereby DISMISSED and the assailed Resolutions Preliminary Attachment, in view of the disposition by King Hong of its trucks
AFFIRMED. Costs against petitioners. and automotive equipment and by virtue of the fact admitted by King Hong
that it had stopped operations preparatory to liquidation, by reason of the
Heirs of Teodolo M. Cruz v. Court of Industrial Relations, G.R. No. L-23331, alien nationality of most of its stockholders, under the provisions of Republic
December 27, 1969. Act No. 3018 nationalizing the rice and corn industry.

39
- In another motion of December 4, 1962, the union had asked that the CIR appears from the transcript of the conference, King Hong and the directors of
at least order King Hong to put up a bond of P500,000.00 to answer for the the union decided to settle the case amicably with the payment by King Hong of
payment of the judgment or to deposit said amount in Court. the same amount of P110,000.00 which was deposited with the Court's
● The 49 union members-petitioners assert that these motions were left hanging disbursing officer immediately upon the signing of the settlement which will be
until the union filed a mandamus petition with this Court (not the present case). prepared by King Hong through its counsel.
● The trial judge issued and released on April 15, 1963 the Deposit Order dated ● Petitioner Natividad Magalpo, a director of the union, together with petitioners
March 30, 1963, ordering King Hong, within 10 days from its finality, to deposit Lydia Bulos and Paciencia Batoon, both union members-claimants, filed on
in Court P100,000.00 and to file a surety bond of equal amount, "to guarantee November 5, 1963, their verified "Manifestation and Objection with Ex- Parte
the payment of whatever amount (a) due petitioner (union) and its members Urgent Motion", relating what transpired at the conference, charging the union
concerned after this Court shall have finally decided the obligation of herein King president, Maylem, with bad faith in that he never previously advised the union
Hongs under the judgment." This Order was affirmed by the CIR en banc, in its representatives that the conference of October 31, 1963 was to discuss a
Resolution denying King Hong's motion for reconsideration. compromise settlement nor that the Supreme Court’s resolution dismissing King
● King Hong sought a review of the Deposit Order. The Supreme Court, in its Hong’s appeal from the Deposit Order had already become final, and asking the
Resolution of September 20, 1963, dismissed for lack of merit the petition for trial judge to shelve the proposed settlement until King Hong shall have
review, and the dismissal became final on October 24, 1963. complied with the said deposit order.
● Earlier, on June 25, 1963: King Hong had offered the Union the maximum ● These petitioners further filed on November 4, 1963 an urgent Ex-parte Motion
amount of P110,000.00 in full settlement of its obligations to the members- for the issuance of a writ of execution for the enforcement of the Deposit Order
claimants of the Union under the judgment, but the union rejected the offer and against King Hong, and asked the trial judge to act on their two urgent motions
counter-offered the minimum amount of P200,000.00. upon receipt thereof.
● The union meanwhile filed to no avail a series of urgent motions on May 8, July ● Both urgent motions were totally ignored by both the trial judge as well as by
1, August 29 and September 6, 1963 for approval of the Chief Examiner's Partial King Hong, despite due notice on the latter.
Report of December 14, 1962 and for enforcement, through a writ of execution ● The request of the union president, Maylem, at the October 31, 1963 conference
or contempt proceedings, of the Deposit Order. that the trial judge have the union counsel present during the proposed signing
● Upon the finality of the Supreme Court’s Resolution dismissing King Hong’s of the settlement agreement set for November 8, 1963, as expressly noted by
petition for a review of the Deposit Order, the union again filed on October 29, the trial judge, was likewise ignored.
1963 still another Urgent Motion, advising the trial judge of the SC's action ● Notwithstanding that notice of the conference set for November 8, 1963 at 2:30
rejecting King Hong’s appeal and invoking the CIR’s ministerial duty of enforcing p.m. was served on November 5, 1963 on the union counsel, petitioner Mary
its said Order. The trial judge took no action on this latest Urgent Motion of the Concepcion, the scheduled conference was never held.
union. ● Unexplainedly, Maylem, the union president and nine other members of the
● Instead, an unscheduled conference was called and held on October 31, 1963 union's board of directors (out of 13 board members) had earlier executed a
in the chambers of the trial judge, and attended by representatives of King Hong, "Settlement" on November 8, without the knowledge, advice, and conformity of
including their counsels of record, on one hand and Segundino S. Maylem, the union counsel, with King Hong's attorney-in-fact, who was duly assisted by
president of the union and eight directors of the union, on the other. Four of King Hong’s two counsels, who likewise executed the "Settlement." In this
these nine union representatives, including the union president himself, had no "Settlement", the said union officials claiming to act “with the authorization of the
claims or awards whatever under the judgment. Said union officials were not Board of Directors and its members," in consideration of the sum of
assisted by counsel, as petitioner Mary Concepcion, counsel of record of the P110,000.00, or one-fourth of the estimated P423,756.74-judgment liability of
union, was not present, not having been notified of the conference. At this King Hong, “waived and quitclaimed…any and all claims it (the union) may have
conference, King Hong made again the same offer to settle and quitclaim the against King Hong as well as the claim of each and every one of the members
judgment in favor of the union members for P110,000.00. But this time, as of the…union against the…firm.” The union further "warranted" in said
40
“Settlement" "that aside from the petitioner (union) itself and the members P110,000.00 settlement paid by [King Hong] and this in itself is a ratification on
thereof, there are no other persons who have any interest over the judgment their part of said settlement."
debt and that if it should happen that other persons shall make a claim against ● Nothing appears in the record, however, as to whether and in what manner King
[King Hong] and/or said judgment debt, that [King Hong], nevertheless, shall no Hong Court determined the authenticity of the signatures.
longer be liable therefor." ● King Hong further filed on December 18, 1963 a motion for reconsideration of
● The “Settlement" was immediately submitted to the trial judge who forthwith on the trial judge's Order approving payment of P33,000.00 to the petitioners-
the same day, November 8, 1963, issued his Order, approving the same, and attorneys by way of attorneys' fees.
entered into CIR’s records at 1:45 p.m. of the same day. ● On August 1, 1964, and August 4, 1964, they were served with copies of the
● Petitioners-lawyers Mary Concepcion et al. upon learning of the "Settlement" CIR’s en banc Resolution dated March 9, 1964, penned by the trial judge,
and King Hong’s deposit with the CIR of the sum of P110,000.00 in pursuance “finding no sufficient justifications to set aside, disturb or modify the Order issued
thereof filed in the afternoon of November 8, 1963 a motion for withdrawal of the in these cases on November 8 and 9, 1963" and denying all three motions for
sum of P33,000.00 equivalent to their 30% contingent fee, without prejudice to reconsideration. Judges Amando C. Bugayong and Ansberto F. Paredes
such action as they may take for enforcing their lien to its full extent. The trial concurred under date of July 29, 1964 with the Resolution, while Judge Arsenio
judge granted such motion in its Order of November 9, 1963. Martinez took no part. No statement of the material allegations of, and issues
● In due course, petitioners-lawyers moved for reconsideration and setting aside raised in, the pertinent pleadings set out in detail hereinabove nor reasons for
of the trial judge's Order of November 8, 1963 approving the "Settlement" and the conclusion of insufficient justification reached by the majority resolution are
prayed the CIR en banc to reinstate the judgment against King Hong and to given therein.
enforce the Deposit Order ● Then Presiding Judge Jose S. Bautista dissented. "Taking into account the
● Petitioners Magalpo, Bulos and Batoon, likewise moved CIR en banc to precipitate approval of settlement over the objection of some union members
reconsider and set aside the trial judge's approval of the "Settlement", in concerned and without hearing them, on the strength simply of the manifestation
disregard of their objection and pending motions of November 5, 1963 to shelve of the petitioner's Board of Directors that it had authority to compromise when
the proposed settlement and to enforce the Deposit Order. previously said union members concerned had already manifested in Annex "E"
● December 26, 1963: They were joined in their plea for reconsideration by 47 that there was no such authority," he voted "that the case be restored to the
other union members-claimants, Co-petitioners at bar. status quo as of October 30, 1963, but the payment already made to the union
● King Hong filed its opposition to the motions for reconsideration, questioning the members be considered as partial payments on account, subject to final
personality and interest of petitioners-movants Magalpo and her 2 other co- liquidation and adjustment; that an order of execution of the judgment in cases
movants and asserting that they were bound by the "Settlement" entered into by Nos. G.R. L-21758 and L-21759 of the Supreme Court be issued (upholding the
their union's board of directors. It alleged that it had deposited with the CIR the Order of March 30, 1963 for deposit of P200,000.00 in cash and surety bond)
sum of P110,000.00 stipulated in the “Settlement" on the same day of its be issued and that the Hearing Officer shall resume the hearing of the
approval by the trial judge. It filed with the CIR on November 21, 1963 a letter Examiner's Report."
of ratification dated November 10, 1963 addressed to the trial judge and ● Hence, the appeals of petitioners.
purportedly signed by some 79 union members-claimants confirming and ● The Santiago Labor Union filed its Answer “putting its weight behind the prayers
accepting the settlement executed by the union board. of the petitioners." The Answer reveals that the union members, feeling
● Petitioners list 21 of these signatures as questionable, asserting that they are at betrayed, had disauthorized and removed from office Maylem, the union
variance with other corresponding signatures in the Payroll dated November 8, president and his board of directors who had executed the "Settlement" with
1963 submitted to the CIR on November 21, 1963, such that "either one or the King Hong and disclaimed the documents of ratification that they had signed at
other signature is a forgery." the behest of Maylem.
● King Hong counters that there is "absolutely no truth to the claim" and that the
signers of the ratification letter “have all received their individual shares of the
41
ISSUE: WON the Supreme Court can sanction the CIR’s majority resolution to satisfy their judgment credit, while the report of King Hong court's
upholding the trial judge's approval of the union board's settlement for P110,000.00 examiner showed that King Hong had sufficient assets and considering that
of the estimated P423,756.74 - judgment liability of King Hong in favor of the their partial judgment credit, as estimated by King Hong court's examiner,
individual union member. amounted to more than P400,000.00.
● The trial judge's rush approval of the settlement disregarded the grave adverse
HELD: NO. consequences thereof to the union members. The settlement, as prepared by
● The precipitate approval of the purported settlement under the circumstances King Hong’s counsel, provided for a union warranty that aside from the union
goes against the grain of fundamental considerations of justice, equity and due itself and the members thereof, "there are no other persons who have any
process. interest over the judgment debt and that if it should happen that other persons
● Petitioners were not accorded due process of law, when, for reasons shall make a claim against King Hong and/or said judgment debt, that King
unexplained in the record, the conference set for November 8, 1963 at 2:30 p.m. Hong, nevertheless, shall no longer be liable therefor." Such warranty was
to take up formally the proposed settlement was cancelled and never held. against the very facts of record, which showed that as early as June 21, 1963,
● The lack of due deliberation and caution in the trial judge's instant approval of petitioners-counsels had duly recorded their attorneys' lien of "30% of whatever
the settlement is seen from the stipulations therein that the union thereby waived amount may finally be awarded in favor of the petitioner." Thus, technically,
and quitclaimed any and all claims which it may have against King Hong, as well since the award in favor of the union members amounted to more than
as the claim of each and every one of the members of the union against King P400,000.00 the settlement for P110,000.00 would conceivably just about cover
Hong, when precisely the authority of the union board members to enter into the 30% attorneys' fees payable to the petitioners-counsels under the contract,
any such compromise or settlement was under express challenge by petitioner if they were so minded to enforce it and bad faith on the union's part were shown,
Magalpo, a board member herself in her Objection and Urgent Motion to shelve with the union members left holding an empty bag. Such onerous terms of the
the settlement filed on November 5, 1963, which the trial judge completely settlement could not then properly be approved by the trial judge as "not contrary
disregarded. to law, morals and public policy."
- Petitioner Magalpo further made serious charges that Maylem, the union ● All these underscore the failure of due process when petitioners were deprived
president, had misled the board members into attending the unscheduled of the formal conference on the proposed settlement scheduled for November
conference held on October 31, 1963 before the trial judge, and had 8, 1963 and of their right to be assisted by the union counsel as expressly
deliberately concealed from them the fact of entry on October 24, 1963 of requested, so that a fair hearing could be accorded petitioners and an
the Order of the Supreme Court upholding the Deposit Order and the effect opportunity afforded them to air their serious charges of bad faith and lack of
thereof of making mandatory upon the trial judge, in accordance with the authority against the union leadership.
terms of his own order, the issuance of a writ for execution or enforcement ● Another vital reason for striking down the settlement is the lack of any express
to compel King Hong to so deposit P100,000.00 in cash and an equal or specific authority of the president and majority of the union board of directors
amount in surety bond to guarantee satisfaction of the union members' to execute the same and scale down the estimated P423,756.74 - judgment
judgment against King Hong. liability of King Hong in favor of the individuals union members to P110,000.00.
- In point of fact, the union's own Urgent Motion of October 29, 1963, On the contrary, petitioner board member Magalpo timely challenged the
emphasizing that the CIR no longer had any excuse for not complying with authority of the union board to execute any such settlement, expressly informing
the deposit order, as well as petitioner Magalpo et al's Urgent ex-parte the trial judge that the union had specifically appointed an entity in Manila, the
motion of November 4, 1963 to the same effect were pending before the "CREAM, Inc.", formerly Credit Research and Intelligence, as its attorney-in-fact
trial judge, unresolved and unacted upon. and "exclusive authorized representative for the evaluation, adjustment and
- Petitioners Magalpo et al. had reason, therefore, to assail the proposed liquidation of its claim against King Hong." Forty-seven other union members-
settlement for P110,000.00 as unconscionable, when at the very least the claimants joined petitioner Magalpo in their denunciation of the union board's
union members could be assured of P200,000.00 under the deposit order unauthorized action, and in their plea for reconsideration with the CIR. Forty-
42
nine union members-claimants entitled to the bulk of the judgment award have "one is the degree of dependence of the individual employee on the union
filed this appeal from the adverse rulings of the Court below. These union organization; the other, a corollary of the first, is the comprehensive power
members have repudiated the former union president, Maylem and his board of vested in the union with respect to the individual." The union may be considered
directors, for having betrayed the union members, and the new union but the agent of its members for the purpose of securing for them fair and just
leadership, in its Answer filed with the Court, has joined petitioners in their prayer wages and good working conditions and is subject to the obligation of giving the
for redress, categorically asserting that the union records do not show any grant members as its principals all information relevant to union and labor matters
by the members to the former union board under Maylem to "negotiate the claim entrusted to it.
or subsequent ratification of the settlement for P110,000.00" which is ● King Hong never filed with the CIR any denial or responsive pleading traversing
“unthinkable and ridiculous." Under such circumstances, the letter of ratification the factual allegations in petitioner Magalpo's Manifestation and Objection. Such
of the settlement purportedly signed by some 79 members, many of whose failure on the part of King Hong constitutes an implied admission of the material
signatures thereon are denounced as forgeries and which ratification was not averments. King Hong’s justification now that it did not file any responsive
authenticated in the proceedings below and has been expressly disowned by pleading or denial because Magalpo and her co-petitioners had no personality
petitioners herein, cannot be given any legal significance or effect. to file their pleadings as they were not parties to the cases in the CIR is of no
● The judgment award was for the payment of overtime, premium and differential avail, for they were actually the awardees and beneficiaries under the judgment
pay to the individual union members as claimants and premium and differential against King Hong and the union was but their agent.
pay to the individual union members as claimants and for the reinstatement of ● The cases of Jesalva et al. vs. Bautista, and Diomela et al. vs. CIR cited by King
the individual union members who testified and proved their having been illegally Hong clearly have no application. In Jesalva, there was no question that the
laid-off, which represent a personal material interest directly in favor of the union had acted with the authority of the union membership. No deceit or
individual union members, as against the lack of material interest on the part of concealment or misrepresentation tainted the settlement. Neither was the
the union as such, the union's lack of authority to execute the settlement, in the amount of the settlement denounced as unconscionable. In Diomela, there was
absence of express or specific authorization by the union members, becomes no question of the authority of the union president to withdraw the unfair labor
patent. The authority of the union as such, to execute a settlement of the practice charge, as the three employees directly affected had co-signed the
judgment award in favor of the individual union members, cannot be presumed withdrawal motion with him.
but must be expressly granted. ● We therefore sustain the minority opinion of then Presiding Judge Bautista of
● In La Campana Food Products, Inc. v. Employees Ass'n. vs. Court of Industrial the CIR.
Relations, et al., the Supreme Court ruled that the union's loss of interest in the ● It is directed that an order for the enforcement of the Deposit Order, be forthwith
case was no ground for dismissing the case, since “the labor union as a body in issued, and that hearings on the Chief Examiner's Report of December 14, 1962
reality has not so great a material interest in the controversy as would prejudice be resumed immediately and without interruption so that the amounts due under
it in the event of dismissal. It is the twenty-one (21) members for whose benefit the judgment to the individual union members may be finally determined without
the ULP case was prosecuted who stand to take tremendous losses" and suffer further delay.
injustice. ● It is unfortunate that pending these proceedings, no application for preliminary
● Just as this Court has stricken down unjust exploitation of laborers by injunction restraining King Hong from disposing of its assets was made, since
oppressive employers, so will it strike down their unfair treatment by their own King Hong had stopped operations in 1962 preparatory to liquidation, by virtue
unworthy leaders. The Constitution enjoins the State to afford protection to of the provisions of Republic Act No. 3018 nationalizing the rice and corn
labor. Fair dealing is equally demanded of unions as well as of employers in industry. King Hong's stockholders are, however, charged with notice of King
their dealings with employees. The union has been evolved as an organization Hong's liability by virtue of the pendency of these appeals, and should any
of collective strength for the protection of labor against the unjust exactions of liquidating dividends have been distributed and paid to them in the meantime,
capital, but equally important is the requirement of fair dealing between the they shall stand liable for the satisfaction of the union workers' judgment against
union and its members, which is fiduciary in nature, and arises out of two factors: King Hong to the extent of such dividends respectively paid to and received by
43
them. Similarly, any outstanding unpaid subscriptions or balances of 1977. Within the last 60 days of the CBA, upon written authority of at least 30%
subscriptions to King Hong's capital stock, estimated at P20,000.00 shall be of the employees in the company, including the petitioners, the Federation of
subject to garnishment and execution in satisfaction of the judgment. Unions of Rizal (FUR) filed a petition for certification election with MOLE. The
● As to the contingent 30% attorneys' fees of petitioners-lawyers, the Court deems petition was opposed by the PAFLU with whom the Amigo Employees Union
it proper at this stage, to direct in the exercise of its authority to control the was at that time affiliated. The same employees who had signed the petition
amount of such fees, that petitioners-lawyers may collect their stipulated filed by FUR signed a joint resolution disaffiliating from PAFLU.
contingent 30% attorneys' fees to the extent that additional amounts may be ● Dolores Villar, representing herself to be the authorized representative of the
realized on the union workers' judgment up to the sum of P150,000.00, including Amigo Employees Union, filed a petition for certification election in the
the initial payment of P110,000.00, (on which they have already collected their Company. The Amigo Employees Union-PAFLU intervened and moved for the
corresponding fee), such that any further amounts collected beyond dismissal of the petition for certification election filed by Villar, on the ground,
P150,000.00 shall no longer be subject to said contingent fee. among others that Villar had no legal personality to sign the petition since she
was not an officer of the union nor is there factual or legal basis for her claim
WHEREFORE, the respondent Court's Orders of November 8, 1963 and March 9, that she was the authorized representative of the local union.
1964 are hereby declared null and void and set aside. The respondent court is ● Med-Arbiter dismissed the petition filed by Villar, which dismissal is still pending
directed to proceed immediately with the execution of the judgment rendered by it appeal before BLR. Amigo Employees Union-PAFLU called a special meeting
against respondent firm in Cases Nos. 709-V and V-1 as affirmed by this Court's of its general membership. A Resolution was thereby unanimously approved
decision of August 31, 1962, in accordance with the directives set forth in the next which called for the investigation by the PAFLU national president, of all of the
preceding paragraph, which is incorporated by reference as an integral portion of petitioners and one Felipe Manlapao, for continuously maligning the union
the dispositive part of this decision. With costs against private respondent in both spreading false propaganda that the union officers were merely appointees of
cases herein decided. the management; and for causing divisiveness in the union. PAFLU formed a
Trial Committee to investigate the local union's charges against the petitioners
a. Admission and Discipline of Members for acts of disloyalty.
• Articles 250(a) and (e); 260(a), and 292(c), Labor Code. ● PAFLU and the Company concluded a new CBA which also reincorporated the
same provisions of the existing CBA, including the union security clause. PAFLU
Villar v. Inciong, G.R. Nos. 50283-84, April 20, 1983. President rendered a decision finding the petitioners guilty of the charges.
PAFLU demanded the Company to terminate the employment of the petitioners
Petitioners: Dolores Villar, Romeo Pequito, Dionisio Ramos, Benigno Mamaraldo, pursuant to the security clause of the CBA. Acting on PAFLU's demand, the
Orlando Acosta, Recitacion Bernus, Anselma Andan, Rolando De Guzman and Rita Company informed PAFLU that it will first secure the necessary clearances to
Llagas terminate petitioners. PAFLU requested the Company to put petitioners under
Respondents: The Hon. Amado G. Inciong, as Deputy Minister of The Ministry of preventive suspension pending the application for said clearances to terminate
Labor, Amigo Manufacturing Incorporated and Philippine Association of Free Labor the petitioners. The Company filed the request for clearance to terminate the
Unions (PAFLU) petitioners before DOLE which was granted. DOLE Secretary Inciong denied
the appeal, hence, this petition for review.
FACTS:
● Petitioners were members of the Amigo Employees Union-PAFLU, a duly ISSUE: WON the DOLE Secretary erred in affirming the grant of clearance of
registered labor organization which, was the existing bargaining agent of the termination of petitioners.
employees in private respondent Amigo Manufacturing, Inc. (Company).
● The Company and the Amigo Employees Union-PAFLU had a CBA governing HELD: NO.
their labor relations, which agreement was then about to expire on February 28,
44
● It is true that disaffiliation from a labor union is not open to legal objection. It is Francisco Salunga (Salunga) had been an employee of San Miguel Brewery, Inc
implicit in the freedom of association ordained by the Constitution. But the Court since 1948. And Salunga was also a member of a Union in San Miguel since 1953.
has laid down the ruling that a closed shop is a valid form of union security, and
such provision in a CBA is not a restriction of the right of freedom of association San Miguel Agrees that as a condition of employment of those employees who are
guaranteed by the Constitution. members of the UNION on the date of the signing of this agreement, or may join the
● In the case at bench, the Company and the Amigo Employees Union-PAFLU UNION during the effectivity of this agreement that if such employees retracts its
entered into a CBA with a union security clause which is a reiteration of the old membership from the union, he/she shall be automatically removed as an employee
CBA. The quoted stipulation for closed-shop is clear and unequivocal. in San Miguel.
Petitioners’ theory that their expulsion was not valid upon the grounds is
untenable. PAFLU had the authority to investigate petitioners on the charges On August 18, 1961 Salunga tendered his resignation from the Union, which
filed by their co-employees in the local union and after finding them guilty as accepted it on August 26, 1961, and transmitted it to the Company on August 29,
charged, to expel them from the roll of membership of the Amigo Employees 1961, with a request for the immediate implementation of section 3, Article II.
Union-PAFLU is clear under the constitution of the PAFLU to which the local
San Miguel informed Salunga that his aforementioned resignation would result in
union was affiliated. And pursuant to the security clause of the new CBA,
the termination of his employment, in view of Section 3. AS such Salunga wrote to
reiterating the same clause in the old CBA, PAFLU was justified in applying said
the Union a letter withdrawing or revoking his resignation and advising the Union to
security clause.
continue deducting his monthly union dues. Salunga also furnished a copy of this
● Recognized and salutary is the principle that when a labor union affiliates with a
communication to the Company. San Miguel in turn, notified the Union of the receipt
mother union, it becomes bound by the laws and regulations of the parent
of said copy and that San
organization. It is undisputable that oppositors were members of the Amigo
Employees Union at the time that said union affiliated with PAFLU; hence, Miguel shall not take any action on this case and shall consider Salunga still a
oppositors are bound by the laws and regulations of PAFLU. member of the union and continue deducting his union dues,
● Inherent in every labor union, or any organization for that matter, is the right of
self-preservation. When members of a labor union seek the disintegration and On September 8, 1961, the Union told the San Miguel that Salunga’s membership
destruction of the very union to which they belong; they thereby forfeit their rights could not be reinstated and insisted on his separation from the service.San Miguel
to remain as members of the union which they seek to destroy. Prudence and told The Union that they asked Salunga if he realized that by resigning from the
equity, as well as the dictates of law and justice, therefore, compelling mandate Union he would in effect be forfeiting his position in the company. When he
the adoption by the labor union of such corrective and remedial measures, in answered in the negative, San Miguel showed him a copy of our Collective
keeping with its laws and regulations, for its preservation and continued Bargaining Agreement and called his attention to Sec. 3, Art. II. Salunga told San
existence; lest by its folly and inaction, the labor union crumble and fall. Miguel that he did not realize that he would be losing his job if he were to resign from
the Union.
WHEREFORE, IN VIEW OF ALL THE FOREGOING, the Order appealed from
affirming the joint decision of the OIC of Regional Office No. 4 in RO4-Case No. T- San Miguel clarified that it did not at any time ask or urge him to withdraw his
IV-3549-T and RO4 Case No. RD-4-4088-77-T granting clearance to terminate resignation; neither that it is asking or insisting that the Union readmit him into the
petitioners as well as dismissing their complaint with application for preliminary Union. San Miguel asserted that informing him of the consequences of his
injunction, is hereby AFFIRMED. No costs. resignation from the Union, was the only humane thing to do under the
circumstances. Nevertheless, if notwithstanding the foregoing clarification the Union
Salunga v. Court of Industrial Relations, G.R. No. L-22456, September 27, 1967. still consider him as having actually resigned from the organization, and The Union
Facts: insist that San Miguel dismiss him from the service in accordance with Sec. 3, Article
II of the CBA, Then San Miguel will have no alternative but to do so.
45
Salunga appealed to his complaint to PAFLU National Convention but his dismissal requisite qualifications, with more reason may the law and the courts
was maintained. exercise the coercive power when the employee involved is a long standing
union member, who, owing to provocations of union officers, was impelled
a complaint for unfair labor practice, against the Union, its president, respondent to tender his resignation, which he forthwith withdrew or revoked. Surely, he
John de Castillo, respondent Cipriano Cid, as PAFLU president, the Company, and may, at least, invoke the rights of those who seek admission for the first
its aforementioned Vice-President Miguel Noel with the CIR. time, and can not arbitrarily he denied readmission.

CIR: ruled for Salunga and held that San Miguel, Miguel Noel, National Brewery &
Allied Industrial Labor Union of the Philippines (PAFLU), John de Castillo, and
Cipriano Cid, are hereby declared guilty of unfair labor practices as charged, and 2. S.C ruled that there was no Unfair Lanor practice The Company was
ordered to cease and desist from further committing such unfair labor practice acts reluctant — if not unwilling — to discharge the petitioner. When the Union
complained. first informed the Company of petitioner's resignation and urged
implementation of section 3 of the bargaining contract, the Company
On appeal, CIR en banc was reversed. advised petitioner of the provision thereof, thereby intimating that he had to
withdraw his resignation in order to keep his employment. Besides, the
Issue/s: Company notified the Union that it (the Company) would not take any action
on the case and would consider the petitioner, "still a member" of the Union.
Whether or not Salunga cannot be re-admitted
When the latter, thereafter, insisted on petitioner's discharge, the Company
Whether or not San Miguel was guilty of unfair labor practice. still demurred and explained it was not taking sides and that its stand was
prompted merely by "humane" considerations, springing from the belief that
Ruling: petitioner had resigned from the Union without realizing its effect upon his
employment. And, as the Union reiterated its demand, the Company notified
1. He may be re-admitted as a member, although he voiced out concerns and petitioner that it had no other alternative but to terminate his employment,
grievances against The Union, because it was stated in the constitution and and dismissed him from the service, although with "regret".
by-laws of the Union.

Although as a general rule, A state may not compel ordinary voluntary Bugay v. Kapisanan ng mga Manggagawa sa Manila Railroad Company, G.R.
associations to admit thereto any given individual, because membership No. L-13093, February 28, 1962.
therein may be accorded or withheld as a matter of privilege; the rule is
qualified in respect of labor unions holding a monopoly in the supply of labor, FACTS:
either in a given locality, or as regards a particular employer with which it Bugay, auditor of the Kapisanan Ng Mga Manggagawa Sa Manila Railroad
has a closed-shop agreement. Company (Kapisanan) and a payroll clerk of Manila Railroad, was requested by the
secretary-treasurer of the company to deliver certain documents belonging to the
Consequently, it is well settled that such unions are not entitled to arbitrarily union and in compliance therewith he delivered them without consulting the officers
exclude qualified applicants for membership, and a closed-shop provision of the union. Making use of these documents, the management of the company filed
would not justify the employer in discharging, or a union in insisting upon with the City Fiscal of Manila against Vicente K. Olazo, president of the union a
the discharge of, an employee whom the union thus refuses to admit to charge for falsification of commercial document (such was dismissed by the city
membership, without any reasonable ground therefor. Needless to say, if fiscal).
said unions may be compelled to admit new members, who have the
46
Subsequently, charges for disloyalty and conduct unbecoming a union member Kapisanan board of directors shows that all of the votes, except those of the
were preferred against Bugay, which caused his explusion. Bugay filed a charge for Hondagua Chapters and Engineering Manila Yard Chapter were not validly cast.
ULP against the union before the CIR which, after due hearing, rendered decision
holding that Bugay’s expulsion was illegal it appearing that the same has not been b. Election of Officers
approved by the majority of the chapters of the union as required by its constitution • Article 250(c); (f); and (k), Labor Code.
and by-laws. Bugay did not have sufficient opportunity to defend himself.
Montaño v. Verceles, G.R. No. 168583, July 26, 2010.
Issue: Whether or not Bugay expulsion as a union member is proper.
FACTS:
RULING:
No. Bugay’s affiliation with the Kapisanan was never terminated. That being the Atty. Montaño worked as legal assistant of FFW Legal Center on October 1, 1994.
case, Bugay is entitled to all the rights and obligations appertaining to every member Subsequently, he joined the union of rank‐ and‐ file employees, the FFW Staff
of the Kapisanan Union actions, whether favorable or otherwise, must be taken by Association, and eventually became the employees' union president in July 1997. In
the chapters within a period of ten days from the time they receive the resolution. November 1998, he was likewise designated officer‐ in‐ charge of FFW Legal
Even under the assumption that the proceedings against Bugay were not irregular, Center.
the resolution in question never had any valid effect on his union membership.
Considering that he has been unduly and discriminatorily deprived of such rights During the 21st National Convention and Election of National Officers of FFW, Atty.
and obligations, the Court finds, and so holds, that the Kapisanan, by their act and Montaño was nominated and elected for the position of National Vice‐ President
conduct, have engaged in and are engaging in ULP. despite the finding of FFW COMELEC that Atty. Montaño is not qualified to run for
It should be observed that the main basis of Bugay's action is his claim that because the position because Section 76 of Article XIX of the FFW Constitution and By‐
of the ULP committed by the officers of defendant union as found by the CIR and Laws prohibits federation employees from sitting in its Governing Board and strong
the SC, he has suffered moral damages. It is true that the decisions both of the CIR opposition and protest of respondent Atty. Ernesto C. Verceles (Atty. Verceles), a
and SC do not contain any statement that the charges preferred by the officers of delegate to the convention and president of University of the East Employees'
the union against him which resulted in his expulsion were "trumped up" or Association (UEEA‐ FFW) which is an affiliate union of FFW.
fabricated, or that said officers acted maliciously or in bad faith, but the fact remains
that the two courts have found that his expulsion was illegal because of the On May 28, 2001, through a letter to the Chairman of FFW COMELEC, Atty.
irregularities committed in his investigation. In effect, it was found that not only has Verceles reiterated his protest over Atty. Montaño's candidacy which he manifested
he not been given an opportunity to defend himself but his expulsion was not during the plenary session before the holding of the election in the Convention. On
submitted to the different chapters of the union as required by its constitution and June 18, 2001, Atty. Verceles sent a follow‐ up letter to the President of FFW
by-laws. requesting for immediate action on his protest.
Issue: WON Bugay was afforded due process.
SC: No. In the investigation held and in the board meeting where the committee’s On July 13, 2001, Atty. Verceles, as President of UEEA‐ FFW and officer of the
report recommending Bugay’s expulsion was approved, Bugay was not present (he Governing Board of FFW, filed before the BLR a petition13 for the nullification of the
was in Lucena). The committee assigned to summon him failed to serve notice upon election of Atty. Montaño as FFW National Vice‐ President.
him because he was in Lucena. However, proceedings still continued anent the
absence of Bugay. Atty. Montaño filed his Comment with Motion to Dismiss on the grounds that the
Besides, the contention that majority of the chapters voted in favor of Bugay's Regional Director of the Department of Labor and Employment (DOLE) and not the
expulsion is not borne by the evidence. An examination of the chapters to the BLR has jurisdiction over the case.

47
DECISION OF LOWER FFW COMELEC: Montano not qualified. Ruling:

Bureau of Labor Relations (BLR): 1.YES. The BLR has jurisdiction over intra‐ union disputes involving a federation.
Montano qualified. it upheld its jurisdiction over the intra‐ union dispute case.
Section 226 of the Labor Code28 clearly provides that the BLR and the Regional
BLR likewise affirmed that Atty. Verceles' legal personality to institute the action as Directors of DOLE have concurrent jurisdiction over inter‐ union and intra‐ union
president of an affiliate union of FFW. disputes. Such disputes include the conduct or nullification of election of union and
workers' association officers. There is, thus, no doubt as to the BLR's jurisdiction
BLR ruled that there were no grounds to hold Atty. Montaño unqualified to run for over the instant dispute involving member‐ unions of a federation arising from
National Vice‐ President of FFW. disagreement over the provisions of the federation's constitution and by‐ laws.

BLR denied motion for reconsiderarion. Rule XVI lays down the decentralized intra‐ union dispute settlement mechanism.
Section 1 states that any complaint in this regard ‘shall be filed in the Regional Office
CA: Reversed BLR, montano not qualified. where the union is domiciled.' The concept of domicile in labor relations regulation
Atty. Montaño did not possess the qualification requirement under paragraph (d) of is equivalent to the place where the union seeks to operate or has established a
Section 26 that candidates must be an officer or member of a legitimate labor geographical presence for purposes of collective bargaining or for dealing with
organization. According to the CA, since Atty. Montaño, as legal assistant employed employers concerning terms and conditions of employment.
by FFW, is considered as confidential employee, consequently, he is ineligible to
join FFW Staff Association, the rank‐ and‐ file union of FFW. * CA (motion for The matter of venue becomes problematic when the intra‐ union dispute involves a
reconsideration): denied. federation, because the geographical presence of a federation may encompass
more than one administrative region. Pursuant to its authority under Article 226, this
ISSUE/S: Bureau exercises original jurisdiction over intra‐ union disputes involving
1.Whether or not the CA was correct in upholding the jurisdiction of the BLR; federations. It is well‐ settled that FFW, having local unions all over the country,
operates in more than one administrative region. Therefore, this Bureau maintains
2.Whether or not was correct in not declaring as premature the petition in view of original and exclusive jurisdiction over disputes arising from any violation of or
the pending protest before FFW COMELEC disagreement over any provision of its constitution and by‐ laws.

3. Whether or not the CA was correct in not finding that the petition violated the rule 2.YES. The petition to annul Atty. Montaño's election as VP was not prematurely
on non‐ forum shopping filed.

4. Whether or not the CA was correct in not dismissing the case for being moot in It is true that under the Implementing Rules, redress must first be sought within the
view of the appointment of Atty. Verceles as NLRC Commissioner; organization itself in accordance with its constitution and by‐ laws. However, this
requirement is not absolute but yields to exception under varying circumstances. the
5. Whether or not Montano is qualified to run as National Vice ‐ President FFW COMELEC failed to timely act thereon. Thus, Atty. Verceles had no other
recourse but to take the next available remedy to protect the interest of the union he
6. Whether or not the CA was correct in granting the petition to annul Montano's represents as well as the whole federation, especially so that Atty. Montaño,
election as FFW National Vice‐ President on the ground that FFW Staff Association immediately after being proclaimed, already assumed and started to perform the
is not a legitimate labor organization. duties of the position. Consequently, Atty. Verceles properly sought redress from
the BLR so that the right to due process will not be violated.
48
promoted to the position of Department Head, thereby disqualifying him for union
3.Montano is estopped from raising this issue since he only raised this during this membership. Said incident, led to a strike spearheaded by Lacanilao group,
motion for reconsideration with the CA. The allegation regarding certification against respondents. Another group however, led by petitioners staged a strike inside the
forum shopping was belatedly raised. It is settled that new issues cannot be raised company premises. After 4 days the strike was settled. On May 10, 1986 an
for the first time on appeal or on motion for reconsideration. agreement was entered into by the representatives of the management, Lacanilao
group and the Tancinco group the relevant terms of which states that all monthly
4. The CA is correct. There is necessity to resolve the case despite the issues having employees shall be united under one union, the ITM Month Employees Association
become moot. As manifested by Atty. Verceles, Atty. Montaño ran and won as FFW (ITM-MEA) to be affiliated with ANGLO. The management of ITM recognizes
National President after his challenged term as FFW National Vice‐ President had ANGLO as the sole and exclusive bargaining agent of all the monthly-paid
expired. It must be stated at this juncture that the legitimacy of Atty. Montaño's employees;
leadership as National President is beyond our jurisdiction and is not in issue in the However, during the pre-election conference attended by MOLE officers, ANGLO
instant case. The only issue for our resolution is petitioner's qualification to run as through its National Secretary, made a unilateral ruling excluding some 56
FFW National Vice‐ President during the May 26‐ 27, 2001 elections. We find it employees consisting of the Manila office employees, members of Iglesia ni Kristo,
necessary and imperative to resolve this issue not only to prevent further repetition non-time card employees, drivers of Mrs. Salazar and the cooperative employees of
but also to clear any doubtful interpretation and application of the provisions of FFW Mrs. Salazar. The election of officers was conducted, the 56 employees in question
Constitution & By‐ laws in order to ensure credible future elections in the interest participated but their votes were segregated without being counted. Lacanilao's
and welfare of affiliate unions of FFW. group won. Lacanilao garnered 119 votes with a margin of 3 votes over Tancinco
prompting petitioners to make a protest.
5. NO, the decision of FFW COMELEC is final and should have been given BLR ruled holding the exclusion of the 56 employees as arbitrary, whimsical, and
credence. wanting in legal basis but set aside the challenged order on the ground that 51 ** of
56 challenged voters were not yet union members at the time of the election per
FFW COMELEC, undeniably, has sufficient authority to adopt its own interpretation April 24, 1986 list submitted before the Bureau.
of the explicit provisions of the federation's constitution and by‐ laws and unless it
is shown to have committed grave abuse of discretion, its decision and ruling will ISSUE:
not be interfered with. The FFW Constitution and By‐ laws are clear that no member WON the 56 employees have the right to vote even though some of them are not
of the Governing Board shall at the same time perform functions of the rank‐ and‐ included in the list of union members submitted to the Bureau. YES
file staff. The BLR erred in disregarding this clear provision. The FFW COMELEC's
ruling which considered Atty. Montaño's candidacy in violation of the FFW RATIO: Submission of the employees names with the BLR as qualified members of
Constitution is therefore correct. the union is not a condition sine qua non to enable said members to vote in the
election of union's officers. It finds no support in fact and in law. Per public
6. NO. the CA's declaration of the illegitimate status of FFW Staff Association is respondent's findings, the April 24, 1986 list consists of 158 union members only
proscribed by law, owing to the preclusion of collateral attack. wherein 51 of the 56 challenged voters' names do not appear.
It is true that under article 242(c) of the Labor Code, as amended, only members of
the union can participate in the election of union officers. The question however of
Tancinco v. Ferrer-Calleja, G.R. No. 78131, January 20, 1988. eligibility to vote may be determined through the use of the applicable payroll period
FACTS: and employee's status during the applicable payroll period. The payroll of the month
Private respondents are the prime organizers of ITM-MEA. While said respondents next preceding the labor dispute in case of regular employees and the payroll period
were preparing to file a petition for direct certification of the Union as the sole and at or near the peak of operations in case of employees in seasonal industries.
exclusive bargaining agent of ITM's bargaining unit, the union's Vice- President, was
49
It can also be shown that their act of joining the election by casting their votes is a And the Union officers asserted asserted Non-ratification of the constitution and by-
clear manifestation of their intention to join the union. They must therefore be laws of the Union and the non-segregation of the Union funds occurred before they
considered ipso facto members. Said employees having exercised their right to became officers and that they have already been correcting the same
unionism by joining ITM-MEA their decision is paramount. Their names could not
have been included in the list of employee submitted on April 24, 1986 to the Bureau Med-Arbiter Cabibihan ordered the holding of a referendum, to be conducted
of Labor for the agreement to join the union was entered into only on May 10, 1986. under the supervision of BLR
Indeed the election was supervised by the Department of Labor where said 56
members were allowed to vote. Private respondents never challenged their right to Petitioners appealed the order to respondent Trajano of BLR the disallowed
vote then. expenditures of P1,278.00 were made in good faith and not used for the personal
benefit of herein union officers but, instead, contributed to the benefit of the
members they were elected in 1980 only and, therefore, they could not be made
Kapisanan ng Manggagawang Pinagyakap v. Trajano, G.R. No. 62306, January responsible for the omissions of their predecessors who failed to turn over union
21, 1985. records for the questioned period there would be a general election on Oct. 4, 1982,
at which time, both the election and the desired referendum could be undertaken to
FACTS: determine the membership at minimum expense they prayed that resolution on the
issue be held in abeyance
On June 30, 1981, a written request for accounts examination of the financial status
of KMP Labor Union, the existing labor union at Franklin Baker Company in San Respondents claimed that Med-Arbiter erred in calling a referendum to decide issue;
Pablo City, was filed by private respondent Silvestre and 13 other employees, who the appropriate action should be the expulsion of union officers
are members of the union
Trajano dismissed both appeals and affirmed in toto the order of Med-Arbiter
Acting on said request, Union Account Examiner Vicedo of the MOLE conducted the
investigation and thereafter submitted a report
ISSUE:
Based on the revelations, private respondents filed with the Regional Office QC, Whether or not union officers were guilty of the alleged acts imputed against them
MOLE, a petition for the expulsion of the union officers thus expulsion was proper

They committed gross violation of the Labor Code, specifically pars. (a), (b), (g), (h), Ruling: The Supreme Court ruled that were not guilty of the alleged acts
(j), and (k) of Article 242; and, the constitution and by-laws of the union, Sections 6
and 7 If herein union officers (also petitioners) were guilty of the alleged acts imputed
against them, said public respondent pursuant to Article 242 of the New Labor Code
The Union Officers Denied imputation and argued that the disallowed expenditures and in the light of Our ruling in Duyag vs. Inciong, 98 SCRA 522, should have meted
were made in good faith; that the same conduced the benefit of members out the appropriate penalty on them, i.e., to expel them from the Union, as prayed
for, and not call for a referendum to decide the issue;
That They are willing to reimburse the same from their own personal funds and They
should not be held accountable for the non-production of books of accounts of the The alleged falsification and misrepresentation of herein union officers were not
Union for years 1977, 1978, and 1979 because they were not the officers then and supported by substantial evidence. The fact that they disbursed the amount of
not one of the former officers of the Union had turned over to them the records P1,278.00 from Union funds and later on was disallowed for failure to attach
supporting papers thereon did not of itself constitute falsification and/or
50
misrepresentation. The expenditures appeared to have been made in good faith and transfer to the employees the title to a tract of land in Caloocan, Rizal. The parcel of
the amount spent for the purpose mentioned in the report, if concurred in or accepted land was eventually registered in the name of the Union.
by the members, are reasonable; and
The Union, through Atty. Benjamin C. Pineda, filed an urgent motion with the Ministry
The repudiation of both private respondents to the highly sensitive position of auditor of Labor and Employment (MOLE) requesting for authority to sell and dispose of the
at the October 4, 1982 election, is a convincing manifestation and demonstration of property. Union President Amado Lopez, in a letter, informed J.C. Espinas and
the union membership's faith in the herein officers' leadership on one hand and a Associates that the general membership of the said Union had authorized a 20%
clear condonation of an act they had allegedly committed. contingent fee for the law firm based on whatever amount would be awarded the
Union.
By and large, the holding of the referendum in question has become moot and Atty. Jose C. Espinas, (the original counsel) established the award of 897 workers'
academic. This is in line with Our ruling in Pascual vs. Provincial Board of Nueva claim. When Atty. Pineda appeared for the Union in these cases, still an associate
Ecija, 106 Phil. 471, which We quote: of the law firm, his appearance carried the firm name B.C. Pineda and Associates,"
giving the impression that he was the principal lawyer in these cases.
The Court should never remove a public officer for acts done prior to his present
term of office. To do otherwise would be to deprive the people of their right to elect Atty. Pineda joined the law firm of Atty. Espinas in 1965 when these cases were
their officers. When the people have elected a man to office, it must be assumed pending resolution. He always held office in the firm's place at Puyat Building, except
that they did this with knowledge of his life and character, and that they disregarded in 1966 to 1967 when he transferred to the Lakas ng Manggagawa Offices. During
or forgave misfaults or misconduct, if he had been guilty of any. It is not for the court, this one-year stint at the latter office, Atty. Pineda continued handling the case with
by reason of such faults or misconduct to practically overrule the will of the people. the arrangement that he would report the developments to the Espinas firm. When
he rejoined the law firm in 1968, he continued working on these cases and using the
Puyat Building office as his address in the pleadings.
c. Major Policy Matter
• Article 250(d), Labor Code. When Atty. Pineda rejoined the Espinas firm in 1968, he did not reveal to his partners
Art. 250. Procedure in collective bargaining. The following procedures shall be (he was made the most senior partner) that he had a retainer's contract. He stayed
observed in collective bargaining with the law firm until 1974 and still did not divulge the 1967 retainer's contract. Only
the officers of the Union knew of the contract.
xxx
The alleged retainer's contract between Atty. Pineda and the Union appears
d. During the conciliation proceedings in the Board, the parties are prohibited from anomalous and even illegal as well as unethical considering that-
doing any act which may disrupt or impede the early settlement of the disputes;
1. The contract was executed only between Atty. Pineda and the officers of the
Halili v. Court of Industrial Relations, G.R. No. L-24864, April 30, 1985. Union chosen by about 125 members only. It was not a contract with the general
membership.
Facts: 2. The contingent fee of 30% for those who were still working with Halili Transit and
The cases involve disputes regarding claims for overtime of more than five hundred the 45% fee for those who were no longer working worked to the prejudice of the
bus drivers and conductors of Halili Transit. Litigation initially commenced with the latter group who should and were entitled to more benefits. Thus, too, when the
filing of a complaint for overtime with the CIR. The disputes were eventually settled alleged retainer's contract was executed in 1967, the Halili Transit had already
when the contending parties reached an Agreement where the Administratrix would stopped operations in Metro Manila. By then, Atty. Pineda knew that all the workers

51
would be out of work which would mean that the 45% contingent fee would apply to authorizing, among others, the affiliation of PEU with NUWHRAIN, and the
all. direct membership of its individual members thereto.
3. The contract which retroactively took effect on January 1, 1966, was executed ● On the same day, the said act was submitted to the general membership,
when Atty. Espinas was still handling the appeal of Halili Transit in the main case and was duly ratified by 223 PEU members. Beginning January 1, 2009,
before the Supreme Court. PEU-NUWHRAIN sought to increase the union dues/agency fees from one
4. When Atty. Pineda filed his motion for approval of his attorney's lien with Arbiter percent (1 % ) to two percent (2%) of the rank and file employees’ monthly
Valenzuela on February 8, 1983, he did not attach the retainer's contract. salaries, brought about by PEU’s affiliation with NUWHRAIN, which
5. The retainer's contract was not even notarized. supposedly requires its affiliates to remit to it two percent (2%) of their
monthly salaries.
A prospective buyer, the Manila Memorial Park Cemetery, Inc. objected in view of ● The non-PEU members objected to the assessment of increased agency
PD 1529 which requires no less than an order from a court of competent jurisdiction fees arguing that: (a) the new CBA is unenforceable since no written CBA
as authority to sell property in trust. has been formally signed and executed by PEU-NUWHRAIN and the Hotel;
(b) the 2% agency fee is exorbitant and unreasonable; and (c) PEU-
Atty. Pineda, without authority from the Supreme Court but relying on the earlier NUWHRAIN failed to comply with the mandatory requirements for such
authority given him by the Ministry of Labor, filed another urgent motion, praying that increase.
the Union be authorized to sell the lot. The sale was finally consummated, resulting ● The OSEC upheld PEU-NUWHRAIN's right to collect agency fees from the
in the execution of an escrow agreement. non-PEU members but only 1% and denied its bid to increase the agency
fees to two percent (2%) for failure to show that its general membership
When Atty. Jose C. Espinas (herein movant and alleged original counsel for the approved the same.
Union) learned of the sale and apportionment of the proceeds from past Union ● On MR, OSEC issued an Order partially granting PEU-NUWHRAIN's motion
president Amado Lopez, he requested Labor Arbiter Raymundo Valenzuela to allow for reconsideration, and declaring it entitled to collect two percent (2%)
him to look into the records of the case. The latter, however, told him that the records agency fees from the non-PEU members.
of the case were missing. Thereupon, Atty. Espinas requested Director Pascual ● The CA set aside the OSEC's March 6, 2012 Order, and reinstated the June
Reyes of the NLRC to locate the records. 2, 2010 Decision. It ruled that PEU-NUWHRAIN failed to prove compliance
with the requisites for a valid check-off since the October 28, 2008 minutes
Issue: do not show that the increase in union dues was duly approved by its
general membership. It also found the July 1, 2010 GMR suspicious
RULING: considering that it surfaced only after PEU received the OSEC's June 2,
2010 Decision disallowing the collection of increased agency fees.
d. Union Funds
• Articles 228(b); 250; and 289, Labor Code. ISSUE: Whether or not PEU-NUWHRAIN has right to collect the increased agency
fees. YES.
Peninsula Employees Union (PEU) v. Esquivel, G.R. No. 218454, December 1,
2016. HELD:
● The recognized collective bargaining union which successfully negotiated
FACTS: the CBA with the employer is given the right to collect a reasonable fee
● On December 13, 2007, Peninsula Employees Union’ (PEU) Board of called “agency fee” from non-union members who are employees of the
Directors passed Local Board Resolution No. 12, series of 20078 appropriate bargaining unit, in an amount equivalent to the dues and other

52
fees paid by union members, in case they accept the benefits under the becomes inconsequential. Jurisprudence states that the express consent of
CBA. the employee to any deduction in his compensation is required to be
● While the collection of agency fees is recognized by Article 259 (formerly obtained in accordance with the steps outlined by the law, which must be
Article 248) of the Labor Code, as amended, the legal basis of the union’s followed to the letter; however, PEU-NUWHRAIN failed to comply.
right to agency fees is neither contractual nor statutory, but quasi- ● Thus, the CA correctly ruled that there is no legal basis to impose union
contractual, deriving from the established principle that non-union dues and agency fees more than that allowed in the expired CBA, .e., at
employees may not unjustly enrich themselves by benefiting from one percent (1 %) of the employee’s monthly basic salary.
employment conditions negotiated by the bargaining union. In the present
case, PEU-NUWHRAIN’s right to collect agency fees is not disputed. Kaisahan at Kapatiran ng mga Manggagawa at Kawani sa MWC-East Zone
● Case law interpreting Article 250 (n) and ( o ) of the Labor Code mandates Union v. Manila Water Company, Inc., G.R. no. 174179, November 16, 2011.
the submission of three (3) documentary requisites in order to justify a valid
levy of increased union dues. FACTS:
● These are: (a) an authorization by a written resolution of the majority of all
the members at the general membership meeting duly called for the ● The Union is the duly-recognized bargaining agent of the rank-and-file
purpose; (b) the secretary’s record of the minutes of the meeting, which shall employees of the respondent Manila Water Company, Inc. (Company) while
include the list of all members present, the votes cast, the purpose of the Borela is the Union President. Metropolitan Waterworks and Sewerage
special assessment or fees and the recipient of such assessment or fees; System (MWSS) entered into a Concession Agreement (Agreement) with
and (c) individual written authorizations for check-off duly signed by the the Company to privatize the operations of the MWSS.
employees concemed. ● Article 6.1.3 of the Agreement provides that the Concessionaire shall grant
● In the present case, however, PEU-NUWHRAIN failed to show compliance its employees benefits no less favorable than those granted to MWSS
with the foregoing requirements. It attempted to remedy the “inadvertent employees at the time of their separation from MWSS such as amelioration
omission” of the matter of the approval of the deduction of two percent (2%) allowance (AA) and the cost-of-living allowance (COLA).
union dues from the monthly basic salary of each union member. ● The payment of the AA and the COLA was discontinued pursuant to
● While the matter of implementing the two percent (2%) union dues was Republic Act No. 6758, otherwise known as the Salary Standardization Law,
taken up during the PEU-NUWHRAIN’s 8th General Membership Meeting which integrated the allowances into the standardized salary. Nonetheless,
on October 28, 2008, there was no sufficient showing that the same had the Union demanded from the Company the payment of the AA and the
been duly deliberated and approved. COLA during the renegotiation of the parties Collective Bargaining
● The minutes of the Assembly itself belie PEU-NUWHRAIN’s claim that the Agreement (CBA).
increase in union dues and the corresponding check-off were duly approved ● The company subsequently agreed for the payment of the AA and COLA.
since it merely stated that “the [two percent (2%)] Union dues will have to However, did not pay said benefits because COA disapproved their
be implemented,” meaning, it would still require the submission of such payment because the company had no funds to cover this benefit.
matter to the Assembly for deliberation and approval. ● A complaint against the Company for payment of the AA, COLA, moral and
● Having failed to establish due deliberation and approval of the increase in exemplary damages, legal interest, and attorney's fees before the National
union dues from one percent ( 1 % ) to two percent (2% ), as well as the Labor Relations Commission (NLRC).
deduction of the two percent (2%) union dues during PEU-NUWHRAIN’s ● LA ruled in favor of the petitioners and ordered the payment of their AA and
8th General Membership Meeting on October 28, 2008, there was nothing COLA, six percent (6%) interest of the total amount awarded, and ten
to confirm, affirm, or ratify through the July 1, 2010 GMR. percent (10%) attorney's fees. NLRC confirmed.
● Corollarily, no individual check-off authorizations can proceed therefrom, ● The Company pointed out that the award of ten percent (10%) attorney's
and the submission of the November 2008 check-off authorizations fees to the petitioners is already provided for in their Memorandum of
53
Agreement (MOA) which mandated that attorney's fees shall be deducted ● As a consequence, the union members represented by the Union were
from the AA and CBA receivables. compelled to litigate and incur legal expenses.On these bases, we find no
● The petitioners argued that the MOA only covered the payment of their difficulty in upholding the NLRC's award of ten percent (10%) attorney's
share in the contracted attorney's fees, but did not include the attorney's fees.
fees awarded by the NLRC. ● The attorney's fees contracted under the MOA do not refer to the amount of
● The CA modified the assailed NLRC rulings by deleting the order for attorney's fees awarded by the NLRC; the MOA provision on attorney's fees
respondent MWCI to pay attorney's fees equivalent to 10% of the total does not have any bearing at all to the attorney's fees awarded by the NLRC
judgment awards. under Article 111 of the Labor Code.
● The Company's argument that the attorney's fees are unconscionable as
ISSUE: Whether or not the NLRC gravely abused its discretion in awarding ten they represent 20% of the amount due or about P21.4 million is more
percent (10%) attorney's fees to the petitioners. NO. apparent than real. Since the attorney's fees awarded by the LA pertained
to the Union's members as indemnity for damages, it was totally within their
HELD: right to waive the amount and give it to their counsel as part of their
● There are two commonly accepted concepts of attorney's fees. In its contingent fee agreement.
ordinary concept, an attorney's fee is the reasonable compensation paid ● Beyond the limit fixed by Article 111 of the Labor Code, such as between
to a lawyer by his client for the legal services the former renders; the lawyer and the client, the attorney's fees may exceed ten percent (10%)
compensation is paid for the cost and/or results of legal services per on the basis of quantum meruit, as in the present case. The SC reversed
agreement or as may be assessed. In its extraordinary concept, CA decision and reinstated LA’s award.
attorney's fees are deemed indemnity for damages ordered by the court to
be paid by the losing party to the winning party. Mariño v. Gamilla, G.R. No. 149763, July 7, 2009.
● Article 111 of the Labor Code, as amended, contemplates the extraordinary
concept of attorney's fees and that Article 111 is an exception to the FACTS:
declared policy of strict construction in the award of attorney's fees. ● Petitioners were among the executive officers and directors (collectively
Although an express finding of facts and law is still necessary to prove the called the Mariño Group) of the University of Sto. Tomas Faculty Union
merit of the award, there need not be any showing that the employer acted (USTFU), a labor union duly organized and registered under the laws of the
maliciously or in bad faith when it withheld the wages. Republic of the Philippines and the bargaining representative of the faculty
● Settled is the rule that in actions for recovery of wages, or where an members of the University of Santo Tomas (UST). Respondents were UST
employee was forced to litigate and, thus, incur expenses to protect his professors and USTFU members.
rights and interests, a monetary award by way of attorney's fees is justifiable ● The 1986 CBA between UST and USTFU expired on 31 May 1988.
under Article 111 of the Labor Code; Section 8, Rule VIII, Book III of its Thereafter, bargaining negotiations ensued between UST and the Mariño
Implementing Rules; and paragraph 7, Article 2208 of the Civil Code. The Group, which represented USTFU. They were not able to reach an
award of attorney's fees is proper, and there need not be any showing that agreement and a bargaining deadlock was declared.
the employer acted maliciously or in bad faith when it withheld the wages. ● On 1992, UST and USTFU executed a (MOA), whereby UST faculty
There need only be a showing that the lawful wages were not paid members belonging to the collective bargaining unit were granted additional
accordingly. economic benefits for the fourth and fifth years of the 1988-1993 CBA,
● In the present case, we find it undisputed that the union members are specifically, the period from 1 June 1992 up to 31 May 1993.
entitled to their AA benefits and that these benefits were not paid by the
Company. That the Company had no funds is not a defense as this was not MEMORANDUM OF AGREEMENT
an insuperable cause that was cited and properly invoked.
54
1.0. The University hereby grants additional benefits to Faculty Members belonging election of the Gamilla Group as USTFU officers. Affirmed on appeal by the
to the collective bargaining unit xxx, which additional benefits shall amount in the (BLR).
aggregate to P42,000,000.00[.]
 ● Respondents filed before the Med-Arbiter, DOLE-NCR, a complaint against
7.0.It is clearly understood and agreed upon that the the Mariño Group, as well as the Philippine Foundation for the Advancement
aggregate sum of P42 million is chargeable against the share of the faculty members of the Teaching Profession, Inc., Security Bank Corporation, and Bank of
in the incremental proceeds of tuition fees collected and still to be collected; xxx and the Philippine Islands claiming that they were the legitimate USTFU officers,
incremental proceeds are, by law and pertinent (DECS) regulations, required to be having been elected on 4 October 1996. They prayed for an order directing
allotted for the payment of salaries, wages, allowances and other benefits of the Mariño Group to cease and desist from using the name of USTFU and
teaching and non-teaching personnel for the UNIVERSITY. from performing acts for and on behalf of the USTFU and the rest of the
members of the collective bargaining unit.
● On 12
purportedly signified their consent to the economic benefits granted ● The DOLE-NCR Reg. Dir. Held that the Mariño Group, as the executive
under the MOA. USTFU, through its President, petitioner Atty. Mariño, wrote officers of USTFU, guilty of violating the provisions of the USTFU
a letter to the UST Treasurer requesting the release to the union of the sum Constitution and By-laws. Also, they violated Article 241(c) and (l) of the
ofP4.2 million, which was 10% of the P42 million economic benefits package Labor Code when among others, they invested in a bank, without prior
granted by the MOA to faculty members belonging to the collective consent of USTFU members, the sum of P9,766,570.01, which formed part
bargaining unit. UST remitted the sum of P4.2 million to USTFU. After of the P42 million economic benefits package. Additionally, the check-off of
deducting from the P42 million economic benefits package several P4.2 million collected by the Mariño Group, as negotiation fees, was invalid.
expenses, a net amount of P6, 389,145.04 remained and was distributed to Under Republic Act No. 6728, 70% of the tuition fee increases should be
the faculty members. allotted to academic and non-academic personnel.
● On 15 December 1994, respondents filed with the Med-Arbiter, DOLE- ● Given that the records were silent as to how much of the P42 million
National Capital Region (NCR), a Complaint for the expulsion of the Mariño economic benefits package was obtained through negotiations and how
Group as USTFU officers and directors. Alleged in their Complaint that the much was from the statutory allotment of 70% of the tuition fee increases,
Mariño Group violated the rights and conditions of membership in USTFU, the DOLE-NCR Regional Director held that the entire amount was within the
particularly by: 1) investing the unspent balance of the P42 million economic statutory allotment, which could not be the subject of negotiation and, thus,
benefits package given by UST without prior approval of the general could not be burdened by negotiation fees.
membership; 2) simultaneously holding elections viva voce; 3) ratifying the ● Bureau of Labor Relations agreed with DOLE-NCR Regional Director. The
CBA involving the P42 million economic benefits package; and 4) approving CA affirmed the BLR decision.

the attorney’s/agency fees worth P4.2 million in the form of check-off.
● On 16 December 1994, UST and USTFU, represented by the Mariño Group, ISSUE: Whether or not the 10% check-off collected by the Mariño Group from the
entered into a new CBA, effective 1 June 1993 to 31 May 1998 (1993-1998 P42 million economic benefits package is valid. NO.
CBA).
● 24 September 1996, petitioner Collantes, as USTFU Secretary- General, HELD:
posted notices in some faculty rooms at UST, informing the union members
of a general assembly. The agenda was the election of new On October 4 ● UST and USTFU stipulated in their 10 September 1992 MOA that the P42
1996, the UST Secretary General headed a general faculty assembly. million economic benefits package granted by UST to the members of the
Respondents were among the elected officers of USTFU (collectively collective bargaining unit represented by USTFU, was chargeable against
referred to as the Gamilla Group). Med-Arbiter DOLE-NCR, nullified the the 70% allotment from the proceeds of the tuition fee increases collected
and still to be collected by UST.

55
● As observed by the DOLE-NCR Regional Director, and affirmed by both the ● The Court further determines that the requisites for a valid levy and
BLR and the Court of Appeals, there is no showing that any portion of the check-off of special assessments, laid down by Article 241(n) and (o),
P42 million economic benefits package was derived from sources other than respectively, of the Labor Code, as amended, have not been complied
the 70% allotment from tuition fee increases of UST. with in the case at bar. To recall, these requisites are: (1) an authorization
● Given the lack of evidence to the contrary, it can be conclusively presumed by a written resolution of the majority of all the union members at the general
that the entire P42 million economic benefits package extended to USTFU membership meeting duly called for the purpose; (2) secretary's record of
came from the 70% allotment from tuition fee increases of UST. Preceding the minutes of the meeting; and (3) individual written authorization for check-
from this presumption, any deduction from the P42 million economic off duly signed by the employee concerned.
benefits package, such as the P4.2 million claimed by the Mariño Group as ● In an attempt to comply with the foregoing requirements, the Mariño Group
attorney’s/agency fees, should not be allowed, because it would ultimately caused the majority of the general membership of USTFU to individually
result in the reduction of the statutorily mandated 70% allotment from the sign a document, which embodied the ratification of the MOA between UST
tuition fee increases of UST. and USTFU, dated 10 September 1992, as well as the authorization for the
● General rule is that attorney’s fees, negotiation fees, and other similar check-off of P4.2 million, from the P42 million economic benefits package,
charges may only be collected from union funds, not from the amounts that as payment for attorney’s fees.
pertain to individual union members. As an exception, special assessments ● As held by the Court of Appeals, however, the said documents constitute
or other extraordinary fees may be levied upon or checked off from any unsatisfactory compliance with the requisites set forth in the Labor Code, as
amount due an employee for as long as there is proper authorization by the amended, and in the USTFU Constitution and By-Laws, even though
employee. individually signed by a majority of USTFU members.
● A check-off is a process or device whereby the employer, on agreement ● The inclusion of the authorization for a check-off of union dues and special
with the Union, recognized as the proper bargaining representative, or on assessments for the Labor Education Fund and attorney’s fees, in the same
prior authorization from the employees, deducts union dues or agency fees document for the ratification of the 10 September 1992 MOA granting the
from the latter's wages and remits them directly to the Union. P42 million economic benefits package, necessarily vitiated the consent
● The Court finds that, in the instant case, the P42 million economic benefits of
USTFU members.
package granted by UST did not constitute union funds from whence the ● The failure of the Mariño Group to strictly comply with the requirements set
P4.2 million could have been validly deducted as attorney’s fees. The P42 forth by the Labor Code, as amended, and the USTFU Constitution and By-
million economic benefits package was not intended for the USTFU coffers, Laws, invalidates the questioned special assessment. Substantial
but for all the members of the bargaining unit USTFU represented, whether compliance is not enough in view of the fact that the special assessment will
members or non-members of the union. diminish the compensation of the union members. Their express consent is
● A close reading of the terms of the MOA reveals that after the satisfaction required, and this consent must be obtained in accordance with the steps
of the outstanding obligations of UST under the 1986 CBA, the balance of outlined by law, which must be followed to the letter. No shortcuts are
the P42 million was to be distributed to the covered faculty members of the allowed.
collective bargaining unit in the form of salary increases, returns on
paycheck deductions; and increases in hospitalization, educational, and Gabriel v. The Honorable Secretary of Labor and Employment, G.R. No.
retirement benefits, and other economic benefits. 115949, March 16, 2000.
● The deduction of the P4.2 million, as alleged attorney’s/agency fees, from FACTS:
the P42 million economic benefits package effectively decreased the share ● Petitioners and 5 others compromise the Executive Board of Solidbank
from said package accruing to each member of the collective bargaining union, the collective bargaining agent for the Solidbank Corporation. Private
unit. respondents are members of said union.

56
● The union’s Executive Board decided to retain the services of their counsel, funding; for the labor organization. As this Court has acknowledged, the
Atty. Lacsina, in connection with negotiations for a new CBA. system of check-off is primarily for the benefit of the union and only indirectly
● A general membership meeting was called where majority of union for the individual employees. The pertinent legal provisions on check-offs
members approved a resolution confirming the decision to engage the are found in Article 222 (b) and Article 241 (o) of the Labor Code.
services of the union’s counsel. The resolution provided that 10% of the total ● Art. 241 has three (3) requisites for the validity of the special assessment
economic benefits that may be secured be given to the counsel at attorney’s for union’s incidental expenses, attorney’s fees and representation
fees. Also, it contained an authorization for Solidbank Corporation to check- expenses. These are: (1) authorization by a written resolution of the majority
off said attorney’s fees the first lump sum of payment of benefits to the of all the members at the general membership meeting called for the
employees under the new CBA as well as turnover said amount to Atty. purpose; (2) secretary’s record of the minutes of the meeting; and (3)
Lacsina or his representative. individual written authorization for check off duly signed by the employees
● Private respondents instituted a complaint against the petitioners and the concerned.
union counsel before the Department of Labor and Employment (DOLE) for ● Clearly, attorney’s fees may not be deducted or checked off from any
illegal deduction of attorney’s fees as well as for quantification of the benefits amount due to an employee without his written consent.
in the 1992 CBA. Petitioners moved for dismissal. ● The Court finds that the General Membership Resolution of October 19,
● Med-arbiter granted the complaint; Secretary of Labor partially granted and 1991 of the SolidBank Union did not satisfy the requirements laid down by
the Order of the Med-Arbiter dated 22 April 1993 is hereby modified as law and jurisprudence for the validity of the ten percent (10%) special
follows: (1) that the ordered refund shall be limited to those union members assessment for union’s incidental expenses, attorney’s fees and
who have not signified their conformity to the check-off of attorney’s fees; representation expenses. There were no individual written check off
and (2) the directive on the payment of 5% attorney’s fees should be deleted authorizations by the employees concerned and so the assessment cannot
for lack of basis. be legally deducted by their employer.
● On MR: affirmed resolution with following modification: (1) the union’s ● Article 222 (b) of the Labor Code prohibits the payment of attorney's fees
counsel be dropped as party litigant; (2) the workers through their union only when it is effected through forced contributions from workers from their
should be made to shoulder the expenses incurred for the attorney's own funds as distinguished from the union funds.
services. Accordingly, the reimbursement should be charged to the union's ● The purpose of the provision is to prevent imposition on the workers of the
general fund/account. duty to individually contribute their respective shares in the fee to be paid
the attorney for his services on behalf of the union in its negotiations with
ISSUE: Whether or not the SOLE erred in ruling that the workers through their union management. The obligation to pay the attorney's fees belongs to the union
should be made to shoulder the expenses incurred for the attorney's services (for and cannot be shunted to the workers as their direct responsibility. Neither
the CB negotiations) and that the reimbursement should be charged to the union's the lawyer nor the union itself may require the individual worker to assume
general fund/account. NO. the obligation to pay attorney's fees from their own pockets.
● Thus the SC held that the SOLE is correct in ruling that the workers through
HELD: their union should be made to shoulder the expenses incurred for the
● Private respondent’s contention: claim that the check-off provision in services of a lawyer. And accordingly, the reimbursement should be
question is illegal because it was never submitted for approval at a general charged to the union's general fund or account. No deduction can be made
membership meeting called for the purpose and that it failed to meet the from the salaries of the concerned employees other than those mandated
formalities mandated by the Labor Code. by law.
● In check-off, the employer, on agreement with the Union, or on prior
authorization from employees, deducts union dues or agency fees from the Vengco v. Trajano, G.R. No. 74453, May 5, 1989.
latter’s wages and remits them directly to the union. It assures continuous FACTS:
57
● The Management of the Anglo-American Tobacco Corporation and the ISSUE: Whether or not Timbungco is guilty of illegally deducting 10% attorneys' fees
Kapisanan ng Manggagawa sa Anglo-American Tobacco Corporation from petitioners' backwages. YES
(FOITAF) entered into a compromise agreement.
● The company is to pay the union members the sum of P150k for their claims HELD:
arising from the unpaid emergency cost of living allowance (ECOLA) and ● Timbungco maintains that the "Kapasiyahan" gave him the authority to
other benefits which were the subject of their complaint before the Ministry make the deduction. This contention is unfounded. Contrary to his claim, the
of Labor. undated "Kapasiyahan" or resolution did not confer upon him the power to
● Respondent Emmanuel Timbungco (union president) received the money deduct 10% of the P150,000.00 despite the alleged approval of the majority
paid in installments and distributed the amount among the union members. of the union workers. Article 241 (o) of the Labor Code provides:
● Petitioners Vengco, et al., who are union members noted that Timbungco ● ART. 241. Rights and conditions of membership in a labor organization. —
was not authorized by the union workers to get the money; and that ten The following are the rights and conditions of membership in a labor
percent (10%) of the P150k had been deducted to pay for attorney's fees organization.
without their written authorization in violation of Article 242(o) of the Labor xxx (o) Other than for mandatory activities under the Code, no special
Code. assessment, attorney's fees, negotiation fees or any other extraordinary
● They demanded from Timbungco an accounting of how the P150k was fees may be checked off from any amount due an employee without an
distributed to the members. Timbungco did not give in to their demand. Thus individual written authorization duly signed by an employee. The
Vengco, et al. filed a complaint with the Ministry of Labor. authorization should specifically state the amount, purpose and beneficiary
● Timbungco alleged among others, that he was authorized by a resolution of the deduction.
signed by the majority of the union members to receive and distribute the ● It is very clear from the provision that attorney's fees may not be deducted
P150k among the workers; that the (10%) attorney's fees was in relation to or checked off from any amount due to an employee without his written
the claim of the local union for payment of emergency cost of living consent except for mandatory activities under the Code. A mandatory
allowance before the Ministry of Labor which is totally distinct and separate activity has been defined as a judicial process of settling dispute laid down
from the negotiation of the CBA; and that the (10%) deduction was in by the law.
accordance with Section II, Rule No. VIII, Book No. III of the Rules and ● It requires the individual written authorization of each employee concerned,
Regulations implementing the Labor Code and therefore, no authorization to make the deduction of attorney's fees valid.
from the union members is required. ● Likewise, the court finds that the other "Kapasiyahan" dated September
● Med-arbiter dismissed the complaint. Vengco et al appealed to the BLR. 18,1981 submitted by Timbungco belied his claim that he was authorized
Director of the Bureau of Labor Relations granted the appeal and ordered by the union workers to receive the sum of P150,000.00 on their behalf. The
Timbungco to render a full accounting of the 150k and to publish in the pertinent portion of the said "Kapasiyahan" provides: 3. Na sa dahilang hindi
union’s bulletin board the list of all recipient union members and the bigla ang pagbabayad sa nasabing "CLAIM" bukod pa sa marami kaming
respective amounts they have received, within ten (10) days from receipt. naghati-hati sa nasabing halaga ipinapasiya naming na kusang-loob na
● OIC Calaycay set aside the decision and ordered an audit examination of kunin ang aming bahagi sa aming kapisanan sa unang linggo ng Disyembre,
the books of account of the union. 1981 at ito'y ipinaalam namin sa Pangulo ng Kapisanan na si Ginoong
● Vengco, et al, sought reconsideration of the order and contended that the Emmanuel Timbungco.
examination of the books of accounts of the union is irrelevant considering ● The above-quoted statement merely indicated the intention of the workers
that the issue involved in the case does not consist of union funds but back to get their claim on the first week of December, 1981 and to inform
pay received by the union members from the company. Timbungco of their intention. Clearly, this statement cannot be construed to
● Dir. Trajano denied the MR and affirmed the audit of the books of account. confer upon Timbungco the authority to receive the fringe benefits for the
workers.
58
● Absent such authority, Timbungco should not have kept the money to · For nearly a year, the Office of the President in four resolutions
himself but should have turned it over to the Union Treasurer. He, therefore, wrestled with the propriety of Saavedra’s ten percent attorney’s
exceeded his authority as President of the Union. fees. In a resolution dated May 29, 1980, Presidential Executive
Assistant Jacobo C. Clave refused to intervene in the matter.
Galvadores v. Trajano, G.R. No. 70067, September 15, 1986. He ruled that the payment of attorney’s fees was a question that
should be settled by the union and its lawyer themselves as
provided in Artile111 of the Labor Code: Art. 111. Attorney’s
Pacific Banking Corporation v. Clave, G.R. No. L-56965, March 7, 1984. fees. — (a) In cases of unlawful withholding of wages the
Doctrine: Attorney’s fees may be charged against union funds in an amount to be culpable party may be assessed attorney’s fees equivalent to
agreed upon by the parties. Any contract, agreement or arrangement of any sort to ten percent of the amount of wages recovered.
the contrary shall be null and void. "(b) It shall be unlawful for any person to demand or accept, in any
judicial or administrative proceedings for the recovery of wages,
Facts: attorney’s fees which exceed ten percent of the amount of
· This case is about the legality of deducting from the monetary wages recovered."cralaw virtu
benefits awarded in a collective bargaining agreement the · The last resolution dated April 13, 1981, Deputy Presidential
attorney’s fees of the lawyer who assisted the union president Executive Assistant Joaquin T. Venus, Jr. ordered the bank to
in negotiating the agreement. pay the union treasurer the said attorney’s fees less the
amounts corresponding to the protesting employees. He held
· 1979, there had been negotiations between the Pacific Banking that the following article 222 of the Labor Code, as amended by
Corporation and the Pacific Banking Corporation Employees Presidential Decree No. 1691, effective May 1, 1980 (before the
Organization (PABECO) for a collective bargaining agreement formalization of the CBA award) had no retroactive effect to the
for 1979 to 1981. Because of a deadlock, the Minister of Labor case:jgc:chanrobles.com.ph
assumed jurisdiction over the controversy. The Deputy Minister
rendered a decision directing the parties to execute a CBA in "ART. 222. Appearances and Fees. — . . . (b) No
accordance with the terms and conditions set forth in his attorney’s fees, negotiation fees or similar charges of any kind
decision arising from any collective bargaining negotiations or
· Lawyer Juanito M. Saavedra’s earliest recorded participation in conclusion of the collective agreement shall be imposed on any
the case was on July 15 and 27, 1979 when he filed a motion individual member of the contracting union: Provided, however,
for reconsideration and a supplemental motion in behalf of the that attorney’s fees may be charged against union funds in an
union. amount to be agreed upon by the parties. Any contract,
· Even before the formalization of the CBA on June 3, 1980, agreement or arrangement of any sort to the contrary shall be
Saavedra on March 24, 1980 filed in the case his notice of null and void."
attorney ‘s lien.
· The union officials requested the bank to withhold around Issue: Whether or not the attorney’s fees can be deducted from the
P345,000 out of the total benefits as ten percent attorney’s fees monetary benefits from the CBA?
of Saavedra. At first, the bank interposed no objection to the
request in the interest of harmonious labor-management Ruling: NO.
relations.In theory, the actual ten percent attorney’s fees may · Under the circumstances, the Office of the President had no jurisdiction
amount to more than one million pesos to make an adjudication on Saavedra’s attorney’s fees. The case was
59
appealed with respect to the CBA terms and conditions, not with respect
to attorney’s fees. Although the fees were a mere incident,
nevertheless, the jurisdiction to fix the same and to order the payment
thereof was outside the pale of Clave’s appellate jurisdiction. He was
right in adopting a hands-off attitude in his first resolution and holding e. Union Information
that the payment of the fees was a question between the lawyer and the
union. Art. 250. Procedure in collective bargaining. The following procedures shall be
· the case is covered squarely by the mandatory and explicit prescription observed in collective bargaining:
of article 222 which is another guarantee intended to protect the
employee against unwarranted practices that would diminish his 1. When a party desires to negotiate an agreement, it shall serve a written
compensation without his knowledge and consent as provided in Art notice upon the other party with a statement of its proposals. The other party
shall make a reply thereto not later than ten (10) calendar days from receipt
242 of the Labor Code:
of such notice;
ART. 242. Rights and conditions of membership in a labor
organization. — The following are the rights and conditions of 2. Should differences arise on the basis of such notice and reply, either party
membership in a labor organization:jgc:chanrobles.com.ph may request for a conference which shall begin not later than ten (10)
calendar days from the date of request.
"x x x
3. If the dispute is not settled, the Board shall intervene upon request of either
"(n) No special assessment or other extraordinary fees may or both parties or at its own initiative and immediately call the parties to
conciliation meetings. The Board shall have the power to issue subpoenas
be levied upon the members of a labor organization unless
requiring the attendance of the parties to such meetings. It shall be the duty
authorized by a written resolution of a majority of all the members of the parties to participate fully and promptly in the conciliation meetings
at a general membership meeting duly called for the purpose. The the Board may call;
secretary of the organization shall record the minutes of the meeting
including the list of all members present, the votes cast, the purpose 4. During the conciliation proceedings in the Board, the parties are prohibited
of the special assessment or fees and the recipient of such from doing any act which may disrupt or impede the early settlement of the
assessment or fees. The record shall be attested to by the disputes; and
president;.
5. The Board shall exert all efforts to settle disputes amicably and encourage
the parties to submit their case to a voluntary arbitrator. (As amended by
"(o) Other than for mandatory activities under the Code, no
Section 20, Republic Act No. 6715, March 21, 1989)
special assessment, attorney’s fees, negotiation fees or any other
extraordinary fees may be checked off from any amount due an
employee without an individual written authorization duly signed by
the employee. The authorization should specifically state the Continental Cement Corporation Labor Union (NLU) v. Continental Cement
amount, purpose and beneficiary of the deduction; Corporation, G.R. No. 51544, August 30, 1990.
· There is no doubt that lawyer Saavedra is entitled to the payment of his
fees but article 222 ordains that union funds should be used for that DOCTRINE: It shall be the duty of any labor organization and its officers to
purpose. The amount of P345,000 does not constitute union funds. It is inform its members on provisions of the constitution and by-laws, collective
money of the employees. The union, not the employees, is obligated to bargaining agreement, the prevailing labor relations system and all their rights
Saavedra.
60
and obligations under existing labor laws. For this purpose, registered labor compliance by private respondent with the 1975 NLRC award in favor
organizations may assess reasonable dues to finance labor relations of petitioner.
seminars and other labor education activities. · Nonetheless, some 110 striking workers did not return to work.
Consequently, on July 26, 1976, private respondent filed with the
FACTS: Department of Labor reports on the dismissal of those who failed to
· On April 21, 1975, the NLRC issued an arbitration award in NLRC comply with the return-to-work order with copies of the reports furnished
Cases No. 2406 and No. 3053 resolving certain demands of the workers affected.
petitioner respecting the working terms and conditions that should be · On July 29, 1976, the president of petitioner and 7 other officers
observed in the establishment of private respondent. However, due to requested admission to work but were informed that their employment
disagreement on the interpretation of the provisions of the award had been terminated by the company.
concerning vacation, sick leaves and standardization of wages, · After due hearing, on March 10, 1977, the NLRC rendered judgment
compliance therewith was delayed. In order to compel private penalizing the said employees.
respondent to immediately implement the award, petitioner staged a ISSUE: Whether the employees were illegally dismissed?
strike on October 25, 1975. It was, however, lifted after the private RULING: NO.
respondent agreed to pay the disputed employees' leaves during the · Petitioner claims that it filed a notice of strike on April 7, 1976. That
period July 1, 1974 to June 30, 1975 in three installments, that is, 50% notice was in connection with a dispute that had been settled by the
on December 20, 1975, 25% on February 25, 1976 and 25% on March Memorandum Agreement between the parties dated May 22, 1976. A
15, 1976. notice of strike is intended to enable the Bureau of Labor Relations to
· The obligation of private respondent to pay the employees their vacation try to settle the dispute amicably. The strike on July 12, 1976 denied the
and sick leaves for the period July 1, 1975 to June 30, 1976 developed Bureau this opportunity.
into a new issue between the parties. Prior to the payment becoming · The non-compliance by the private respondent with the said award did
due, private respondent negotiated with petitioner for a staggered form not threaten the existence of petitioner or that of its members. The
of payment as before due to its financial difficulties and planned dispute did not concern the right of the Union to organize nor the
shutdown of the plant in July. Petitioner at first insisted that its members employees' right to work. It merely involved the non-payment of the
be paid full; however, it subsequently agreed to installment payments vacation and sick leaves of the employees for the past years' services.
but gave warning on July 11, 1976, a Sunday, to the private respondent · The officers had the duty to guide their members to respect the law.
that payment of 50% of the benefits should be made not later than July Instead, they urged them to violate the law and defy the duly constituted
12, 1976 and the remaining 50%., not later than the end of the month. authorities. Their responsibility is greater than that of the members.
· Petitioner staged a strike in the early of July 12, 1976, picketing the Their dismissal from the service is a just penalty for their unlawful acts.
entrance of the premises of private respondent. · It is within the power of the NLRC to order the removal of the officers
· On July 13, 1976, the Minister of Labor issued an order thru the Director of petitioner. This is provided for in the labor law.
of the Bureau of Labor Relations, directing the striking workers to Art. 242. Rights and conditions of membership in a labor
resume work under the terms and conditions prevailing prior to the work organization. — The following are the rights and conditions of
stoppage. The order was served on the parties in the afternoon of the membership in a labor organization:
same date. · xxx xxx xxx
· Nevertheless, on July 14, 1976, only 11 out of the total work force of (p) It shall be the duty of any labor organization and its officers to
about 120 workers in one shift reported for work and were admitted by inform its members on provisions of the constitution and by-laws,
the company. On July 15, 1976, petitioner filed a motion for collective bargaining agreement, the prevailing labor relations
reconsideration of the return-to-work order or its suspension pending system and all their rights and obligations under existing labor laws.
61
For this purpose, registered labor organizations may assess
reasonable dues to finance labor relations seminars and other labor Art 303,LC
education activities. [288] Penalties. 229 Except as otherwise provided in this Code, or unless the acts
Any violation of the above rights and conditions of membership shall complained of hinge on a question of interpretation or implementation of ambiguous
be a ground for cancellation of union registration or expulsion of an provisions of an existing collective bargaining agreement, any violation of the
officer from office, whichever is appropriate. At least 30 per cent of provisions of this Code declared to be unlawful or penal in nature shall be punished
all the members of a union or any member or members specifically with a fine of not less than One Thousand Pesos (P1,000.00) nor more than Ten
concerned may report such violation to the Bureau. The Bureau Thousand Pesos (P10,000.00), or imprisonment of not less than three months nor
shall have the power to hear and decide any reported violation and more than three years, or both such fine and imprisonment at the discretion of the
to mete out the appropriate penalty. court. In addition to such penalty, any alien found guilty shall be summarily deported
· The officers of petitioner misinformed the members and led them into upon completion of service of sentence. Any provision of law to the contrary
staging an illegal strike. If the NLRC is to attain the objective of the Labor notwithstanding, any criminal offense punished in this Code shall be under the
Code to ensure a stable but dynamic and just industrial peace 6 the concurrent jurisdiction of the Municipal or City Courts and the Courts of First
removal of undesirable labor leaders must be effected. Instance.

Art 304, LC
f. Enforcement and Remedies [289] Who are Liable When Committed by Other Than Natural Person. If the offense
• Articles 250, last paragraph; is committed by a corporation, trust, firm, partnership, association or any other entity,
Art 252, LC the penalty shall be imposed upon the guilty officer or officers of such corporation,
Meaning of duty to bargain collectively. The duty to bargain collectively means trust, firm, partnership, association or entity.
the performance of a mutual obligation to meet and convene promptly and
expeditiously in good faith for the purpose of negotiating an agreement with respect Mendoza v. Officers of Manila Water Employees Union (MWEU), G.R. No.
to wages, hours of work and all other terms and conditions of employment including 201595, January 25, 2016.
proposals for adjusting any grievances or questions arising under such agreement FACTS:
and executing a contract incorporating such agreements if requested by either party · Petitioner was a member of the Manila Water Employees Union
but such duty does not compel any party to agree to a proposal or to make any (MWEU), a Department of Labor and Employment (DOLE)-registered
concession. labor organization consisting of rank-and-file employees within Manila
Water Company (MWC)
Art 289,LC · In an April 11, 2007 letter, MWEU through Cometa informed petitioner
Who are liable when committed by other than natural person. If the offense is that the union was unable to fully deduct the increased P200.00 union
committed by a corporation, trust, firm, partnership, association or any other entity, dues from his salary due to lack of the required December 2006 check-
the penalty shall be imposed upon the guilty officer or officers of such corporation, off authorization from him. Petitioner was warned that his failure to pay
trust, firm, partnership, association or entity. the union dues would result in sanctions upon him. Quebral informed
Borela, through a May 2, 2007 letter, that for such failure to pay the
union dues, petitioner and several others violated Section 1(g), Article
IX of the MWEU’s Constitution and By-Laws.
On May 21, 2007, a notice of hearing was sent to petitioner, who attended
the scheduled hearing and recommended that grievance committee’s

62
recommendation and imposition upon them of a penalty of 30 days Article 249. Unfair labor practices of labor organizations. It shall be
suspension, effective June 25, 2007. unlawful for q labor organization, its officers, agents, or
· Petitioner was once more charged with non-payment of union dues, representatives to commit any of the following unfair labor
and was required to attend an August 3, 2007 hearing. Thereafter, practices:
petitioner was again penalized with a 30-day suspension through an (a) To restrain or coerce employees in the exercise of their right to
August 21, 2007 letter by Borela informing petitioner of the Executive self-organization; Provided, That the labor organization shall have
Board’s "unanimous approval" the right to prescribe its own rules with respect to the acquisition or
· On October 2, 2007, petitioner was charged with non-payment of union retention of membership;
dues for the third time. He did not attend the scheduled hearing. This (b) To cause or attempt to cause an employer to discriminate
time, he was meted the penalty of expulsion from the union, per against an employee, including discrimination against an employee
"unanimous approval"of the members of the Executive Board. His pleas with respect to whom membership in such organization has been
for an appeal to the General Membership Assembly were once more denied or terminated on any ground other than the usual terms and
unheeded. conditions under which membership or continuation of membership
· n 2008, during the freedom period and negotiations for a new collective is made available to other members;
bargaining agreement (CBA) with MWC, petitioner joined another union, Issue: Whether or not petitioner was illegally suspended?
the Workers Association for Transparency, Empowerment and Reform, Ruling: YES
All-Filipino Workers Confederation (WATER-AFWC). He was elected · Regarding suspension of a union member, MWEU’s Constitution
union President. Other MWEU members were inclined to join WATER- and By-Laws provides under Article X, Section 4 thereof that "[a]ny
AFWC, but MWEU director Torres threatened that they would not get suspended member shall have the right to appeal within three (3)
benefits from the new CBA. working days from the date of notice of said suspension. In case of
· The MWEU leadership submitted a proposed CBA which contained an appeal a simple majority of vote of the Executive Board shall be
provisions to the effect that in the event of retrenchment, non-MWEU necessary to nullify the suspension.
members shall be removed first, and that upon the signing of the CBA, · When an MWEU member is suspended, he is given the right to
only MWEU members shall receive a signing bonus. appeal such suspension within three working days from the date of
· LA: too premature, referred back to the union notice of said suspension, which appeal the MWEU Executive
· NLRC: Set aside LA decision for lack of jurisdiction- should be with the Board is obligated to act upon by a simple majority vote. When the
Bureau of Labor relations penalty imposed is expulsion, the expelled member is given seven
days from notice of said dismissal and/or expulsion to appeal to the
· CA: NLRC. Citing that "Inter-Union Dispute" refers to any conflict Executive Board, which is required to act by a simple majority vote
between and among legitimate labor unions involving representation of its members. The Board’s decision shall then be approved/
questions for purposes of collective bargaining or to any other conflict disapproved by a majority vote of the general membership
or dispute between legitimate labor unions. "Intra-Union Dispute" refers assembly in a meeting duly called for the purpose.
to any conflict between and among union members, including · The documentary evidence is clear that when petitioner received
grievances arising from any violation of the rights and conditions of Borela’s August 21, 2007 letter informing him of the Executive
membership, violation of or disagreement over any provision of the Board’s unanimous approval of the grievance committee
union’s constitution and by-laws, or disputes arising from chartering or recommendation to suspend him for the second time effective
affiliation of union. On the other hand, the circumstances of unfair labor August 24, 2007, he immediately and timely filed a written
practices (ULP) of a labor organization are stated in Article 249 of the appeal.His appeals were not duly acted upon by the officials of the
Labor Code, to wit: Union.
63
· Contrary to respondents’ argument that petitioner lost his right to (MESALA) and the Meralco Mutual Aid and Benefits Association
appeal when he failed to petition to convene the general assembly (MEMABA) and exerted undue influence on the members of FLAMES.
through the required signature of 30% of the union membership in · On 6 May 2003, the COMELEC issued a Decision, 10 declaring Daya,
good standing pursuant to Article VI, Section 2(a) of MWEU’s et al., officially disqualified to run and/or to participate in the 7 May 2003
Constitution and By-Laws or by a petition of the majority of the FLAMES elections. The COMELEC also resolved to exclude their
general membership in good standing under Article VI, Section 3, names from the list of candidates in the polls or precincts, and further
this Court finds that petitioner was illegally suspended for the declared that any vote cast in their favor shall not be counted
second time and thereafter unlawfully expelled from MWEU due to · 7 May 2003, the COMELEC proclaimed the following candidates,
respondents’ failure to act on his written appeals. The required including some of herein petitioners as winners of the elections,
petition to convene the general assembly through the required · 8 May 2003, private respondents Daya, et al., along with Ong, et al.,
signature of 30% (under Article VI, Section 2[a]) or majority (under filed with the Med-Arbitration Unit of the DOLE-NCR, a Petition13 to: a)
Article VI, Section 3) of the union membership does not apply in Nullify Order of Disqualification; b) Nullify Election Proceedings and
petitioner’s case; the Executive Board must first act on his two Counting of Votes; c) Declare Failure of Election; and d) Declare
appeals before the matter could properly be referred to the general Holding of New Election to be Controlled and Supervised by the DOLE.
membership. Because respondents did not act on his two appeals, · MED- ARBITER: In favor of respondents.
petitioner was unceremoniously suspended, disqualified and · BLR Director: Affirmed MED-Arbiter
deprived of his right. · CA: Affirmed BLR

Issue:
Diokno v. Cacdac, G.R. No. 168475, July 4, 2007. Whether the BLR acted with grave abuse of discretion for taking cognizant of the
Doctrine: Before a party is allowed to seek the intervention of the court, it is a pre- nullification case against respondents?
condition that he should have availed of all the means of administrative processes
afforded him. Hence, if a remedy within the administrative machinery can still be Ruling:
resorted to by giving the administrative officer concerned every opportunity to decide NO. BLR acted with jurisdiction over the said case.
on a matter that comes within his jurisdiction when such remedy should be · Court is emphatic that "before a party is allowed to seek the intervention
exhausted first before the court's judicial power can be sought. The premature of the court, it is a pre-condition that he should have availed of all the
invocation of court's judicial intervention is fatal to one's cause of action. means of administrative processes afforded him. Hence, if a remedy
within the administrative machinery can still be resorted to by giving the
Facts: administrative officer concerned every opportunity to decide on a matter
· The First Line Association of Meralco Supervisory Employees that comes within his jurisdiction when such remedy should be
(FLAMES) is a legitimate labor organization which is the supervisory exhausted first before the court's judicial power can be sought. The
union of Meralco. Petitioners and private respondents are members of premature invocation of court's judicial intervention is fatal to one's
FLAMES. cause of action."
· On 2 May 2003, petitioners filed a Petition with the COMELEC seeking · However there are exceptions to the applicability of the doctrine. Among
the disqualification of private respondents Edgardo Daya et al. the established exceptions are: 1) when the question raised is purely
Petitioners alleged that Daya, et al., allowed themselves to be assisted legal; 2) when the administrative body is in estoppel; 3) when the act
by non-union members, and committed acts of disloyalty which are complained of is patently illegal; 4) when there is urgent need for judicial
inimical to the interest of FLAMES. In their campaign, they allegedly intervention; 5) when the claim involved is small; 6) when irreparable
colluded with the officers of the Meralco Savings and Loan Association damage will be suffered; 7) when there is no other plain, speedy, and
64
adequate remedy; 8) when strong public interest is involved; 9) when from union members in support of a resolution containing the decision
the subject of the proceeding is private land; 10) in quo warranto of the signatories
proceedings; and 11) where the facts show that there was a violation of · A tug-of-war then ensued between the two rival groups, with both
due process. seeking recognition from Bayer and demanding remittance of the union
· As aptly determined by the BLR Director, private respondents Daya, et dues collected from its rank-and-file members. On September 8, 1998,
al., were prejudiced by the disqualification order of the COMELEC. They Remigio's splinter group wrote Facundo, FFW and Bayer informing
endeavored to seek reconsideration, but the COMELEC failed to act them of the decision of the majority of the union members to disaffiliate
thereon. The COMELEC was also found to have refused to receive their from FFW.
written protest. The foregoing facts sustain the finding that private · Remigio also asked Bayer to desist from further transacting with EUBP.
respondents Daya, et al., were deprived of due process. Hence, it Facundo, meanwhile, sent similar requests to Bayer requesting for the
becomes incumbent upon private respondents Daya, et al., to seek the remittance of union dues in favor of EUBP and accusing the company
aid of the BLR. To insist on the contrary is to render their exhaustion of of interfering with purely union matters.
remedies within the union as illusory and vain. · Apparently, the two groups failed to settle their issues. On June 18,
1999, Labor Arbiter Jovencio Ll. Mayor, Jr. dismissed the first ULP
Employees Union of Bayer Philippines, FFW v. Bayer Philippines, Inc., G.R. No. complaint for lack of jurisdiction. The Arbiter explained that the root
162943, June 15, 2005 cause for Bayer's failure to remit the collected union dues can be traced
FACTS: to the intra-union conflict between EUBP and Remigio's group
· Petitioner Employees Union of Bayer Philippines (EUBP) is the · REUBP and Bayer agreed to sign a new CBA. Remigio immediately
exclusive bargaining agent of all rank-and-file employees of Bayer informed her allies of the management's decision. petitioners
Philippines (Bayer), and is an affiliate of the Federation of Free Workers immediately filed an urgent motion for the issuance of a restraining
(FFW). order/injunction before the National Labor Relations Commission
· In 1997, EUBP, headed by its president Juanito S. Facundo (Facundo), (NLRC) and the Labor Arbiter against respondents. Petitioners
negotiated with Bayer for the signing of a collective bargaining asserted their authority as the exclusive bargaining representative of all
agreement (CBA). During the negotiations, EUBP rejected Bayer's rank-and-file employees of Bayer and asked that a temporary
9.9% wage-increase proposal resulting in a bargaining deadlock. restraining order be issued against Remigio's group and Bayer to
Subsequently, EUBP staged a strike, prompting the Secretary of the prevent the employees from ratifying the new CBA.
Department of Labor and Employment (DOLE) to assume jurisdiction · January 26, 2000: The Regional Director of the Industrial Relations
over the dispute. Division of DOLE issued a decision dismissing the issue on expulsion
· Respondent Avelina Remigio (Remigio) and 27 other union members, filed by EUBP against Remigio and her allies for failure to exhaust reliefs
without any authority from their union leaders, accepted Bayer's wage- within the union and ordering the conduct of a referendum to determine
increase proposal. EUBP's grievance committee questioned Remigio's which of the two groups should be recognized as union officers. EUBP
action and reprimanded Remigio and her allies. On January 7, 1998, seasonably appealed the said decision to the Bureau of Labor Relations
the DOLE Secretary issued an arbitral award ordering EUBP and Bayer (BLR). On June 16, 2000, the BLR reversed the Regional Director's
to execute a CBA retroactive to January 1, 1997 and to be made ruling and ordered the management of Bayer to respect the authority of
effective until December 31, 2001. the duly-elected officers of EUBP
· The rift between Facundo's leadership and Remigio's group broadened. ·
On August 3, 1998, barely six months from the signing of the new CBA, · Unfortunately, the said BLR ruling came late since Bayer had already
during a company-sponsored seminar, Remigio solicited signatures signed a new CBA with REUBP on February 21, 2000. The said CBA
was eventually ratified by majority of the bargaining unit.
65
· Aggrieved by the Labor Arbiter's decision to dismiss the second ULP valid CBA with the duly certified and exclusive bargaining agent, the
complaint, petitioners appealed the said decision, but the NLRC denied former indubitably abandons its recognition of the latter and terminates
the appeal. EUBP's motion for reconsideration was likewise denied. the entire CBA.
· a legitimate labor organization cannot be construed to have abandoned
ISSUE: Whether EUBP is entitled to relief? its pending claim against the management/employer by returning to the
RULING: negotiating table to fulfill its duty to represent the interest of its members,
YES. except when the pending claim has been expressly waived or
· An intra-union dispute refers to any conflict between and among union compromised in its subsequent negotiations with the management. To
members, including grievances arising from any violation of the rights hold otherwise would be tantamount to subjecting industrial peace to
and conditions of membership, violation of or disagreement over any the precondition that previous claims that labor may have against capital
provision of the union's constitution and by-laws, or disputes arising must first be waived or abandoned before negotiations between them
from chartering or disaffiliation of the union. may resume. Undoubtedly, this would be against public policy of
· This is the reason why it is axiomatic in labor relations that a CBA affording protection to labor and will encourage scheming employers to
entered into by a legitimate labor organization that has been duly commit unlawful acts without fear of being sanctioned in the future.
certified as the exclusive bargaining representative and the employer · Bayer decided to recognize EUBP as the certified collective bargaining
becomes the law between them. Additionally, in the Certificate of agent of its rank-and-file employees for purposes of its 2006-2007 CBA
Registration issued by the DOLE, it is specified that the registered CBA negotiations is of no moment. It did not obliterate the fact that the
serves as the covenant between the parties and has the force and effect management of Bayer had withdrawn its recognition of EUBP and
of law between them during the period of its duration. Compliance with supported REUBP during the tumultuous implementation of the 1997-
the terms and conditions of the CBA is mandated by express policy of 2001 CBA.
the law primarily to afford protection to labor and to promote industrial · On the matter of damages prayed for by the petitioners, we have held
peace. Thus, when a valid and binding CBA had been entered into by that as a general rule, a corporation cannot suffer nor be entitled to
the workers and the employer, the latter is behooved to observe the moral damages. A corporation, and by analogy a labor organization,
terms and conditions thereof bearing on union dues and representation. being an artificial person and having existence only in legal
If the employer grossly violates its CBA with the duly recognized union, contemplation, has no feelings, no emotions, no senses; therefore, it
the former may be held administratively and criminally liable for unfair cannot experience physical suffering and mental anguish. Mental
labor practice. suffering can be experienced only by one having a nervous system and
· Silva v. National Labor Relations Commission,It was explained that it flows from real ills, sorrows, and griefs of life - all of which cannot be
correlations of Article 248 (1) and Article 261 of the Labor Code to mean suffered by an artificial, juridical person.
that for a ULP case to be cognizable by the Labor Arbiter, and for the
NLRC to exercise appellate jurisdiction thereon, the allegations in the
complaint must show prima facie the concurrence of two things, namely: Rodriguez v. Galvadores, G.R. Nos. 76579-82, August 31, 1988.
(1) gross violation of the CBA; and (2) the violation pertains to the
economic provisions of the CBA. Facts:
· This pronouncement in Silva, however, should not be construed to · Among employees of the Philippines Long Distance Telephone
apply to violations of the CBA which can be considered as gross Company (PLDT), who are members of the same union, the Free
violations per se, such as utter disregard of the very existence of the Telephone Workers Union (FTWU). The disputes concern the validity of
CBA itself, similar to what happened in this case. When an employer the general elections for union officers in 1986, and the increase of
proceeds to negotiate with a splinter union despite the existence of its
66
union dues adopted and put into effect by the incumbent officers · Rey Sumangil and his followers hide themselves off to the Bureau of
subsequent to said elections. Labor Relations. They filed a petition on March 26, 1987 challenging the
· The union's by-laws provide for the election of officers every three (3) resolution for the increase in union dues. citing Article 242 of the Labor
years, in the month of July. Pursuant thereto, the union's Legislative Code which reads as follows:
Council set the provincial elections for its officers
· The same Council also quite drastically raised the fees for the filing of Art. 242. — Rights and conditions of membership in a labor
certificates of candidates which had therefore ranged from P75.00 to organization. — ... Any violation of the above rights and conditions
P100.00. The filing fee for each candidate for president of the labor of membership shall be a ground for cancellation of union
organization was increased to P3,000; that for each candidate for vice- registration and expulsion of officer from office, whichever is
president, secretary general, treasurer and auditor, to P2,000.00; and appropriate. At least thirty percent (30%) of all the members of a
that for assistant secretary, assistant treasurer and assistant auditor, to union or any member or members specially concerned may report
P1,000.00 each. such violation to the Bureau. The Bureau shall have the power to
-The validity of the elections was very shortly challenged on the hear and decide any reported violation to mete the appropriate
ground of lack of (1) due notice and (2) adequate ground rules. penalty.
Carlos Galvadores and his fellow candidates filed on July 22, 1986
a petition with the BLR, praying that the Union's COMELEC be · Again Sumangil and his group went up on appeal to the Director of
directed to promulgate ground rules for the conduct of the provincial Labor Relations, before whom they raised the issue of whether or not
elections the petition in fact had the support of at least 30% of the members, and
· The Union COMELEC proceeded with the general elections in all the said 30%-support was indeed a condition sine qua non for acquisition
PLDT branches in Metro Manila on July 25, 1986. It then reported that by the Med-Arbiters
as of July 15, 1986 the number of qualified voters was 9,429 of which · July 1, 1987 the Director of Labor Relations rendered a decision
6,903 actually voted, the percentage of turn-out being 73% reversing that of the Med-Arbiter. The Director ordered the cessation of
· Livi Marquez and Carlos Galvadores, and their respective groups, the collection of the twenty-nine peso increase and the return of the
forthwith filed separate motions praying that the COMELEC be declared amounts already collected. In the first place, according to her, the
guilty of contempt for defying the temporary restraining order, and for petition was supported by 6,022 signatures, a number comprising more
the nullification not only of the Metro Manila elections of July 25, 1986 than 30% of the total membership of the union (10,413). In the second
but also the provincial elections of July 21 and 22, 1986. place, the Director ruled, even assuming the contrary, the lack of 30%-
· MED-ARBITER: Denied the petitions to nullify the elections, as well as support will not preclude the BLR from taking cognizance of the petition
the motion for contempt, but invalidated the increase in rates of filing where there is a clear violation of the rights and conditions of union
fees for certificates of candidacies. membership because Article 226 of the Labor Code,
· LABOR RELATIONS: Overturned MED-ARB’s decision · Article 242 of the Labor Code, relied upon by the Med-Arbiter, the
· Benedicto Rodriguez, the chairman of the Union COMELEC, claims the Director expressed the view that the 30% support therein provided is
decision was rendered with grave abuse of discretion considering that not mandatory, and is not a condition precedent to the valid presentation
(a) the Med-Arbiter had found no fraud or irregularity in the elections; of a grievance before the Bureau of Labor Relations. The Director ruled,
(b) the election was participated in by more than 73% of the entire union finally, that Sumangil and the other union members had a valid
membership; and (e) the petition for nullity was not supported by 30% grievance calling for redress, since the record disclosed no compliance
of the general membership. with the requirement that the resolution for the increase of union dues
be passed by at least 2/3 vote of the members of the Legislative Council
and be ratified by a majority of the entire membership at a plebiscite.
67
the members of a union or any member or members specially
ISSUE: Whether 1) The election was valid? concerned." The use of the permissive "may" in the provision at once
2) 30% membership support is indispensable negates the notion that the assent of 30% of all the members is
3) Increase of union fees are valid? mandatory. More decisive is the fact that the provision expressly
declares that the report may be made, alternatively by "any member or
RULING: members specially concerned." And further confirmation that the assent
1)NO. of 30% of the union members is not a factor in the acquisition of
· The dates for provincial elections were set for July 14 to 18, 1986. But jurisdiction by the Bureau of Labor Relations is furnished by Article 226
they were in fact held on July 21 to 22, 1986, without prior notice to all of the same Labor Code, which grants original and exclusive jurisdiction
voting members, and without ground rules duly prescribed therefor. The to the Bureau, and the Labor Relations Division in the Regional Offices
elections in Metro Manila were conducted under no better of the Department of Labor
circumstances. It was held on July 25, 1986 in disregard and in defiance
of the temporary restraining order properly issued by the Med-Arbiter 3) NO.
on July 23, 1986, notice of which restraining order had been regularly · The respondent Director found that the resolution of the union's
served on the same date, as the proofs adequately show, on both the Legislative Council to this effect does not bear the signature of at least
Union, President, Manolito Paran, and the Chairman of the Union two-thirds (2/3) of the members of the Council, contrary to the
COMELEC, Benedicto Rodriguez. requirement of the union constitution and by-laws; and that proof is
wanting of proper ratification of the resolution by a majority of the
· Undue haste, lack of adequate safeguards to ensure integrity of the general union membership at a plebiscite called and conducted for that
voting, and absence of notice of the dates of balloting, thus attended purpose, again in violation of the constitution and by-laws. The
the elections in the provinces and in Metro Manila. They cannot but resolution increasing the union dues must therefore be struck down, as
render the proceedings void.The claim that there had been a record- illegal and void, arbitrary and oppressive.
breaking voter turnout of 73%, even if true, cannot purge the elections
of their grave infirmities. The elections were closely contested.
Duyag v. Inciong, G.R. No. L-47775, July 5, 1980.
· It goes without saying that free and honest elections are indispensable FACTS:
to the enjoyment by employees and workers of their constitutionally The plaintiffs in this case are arrastre checkers at E. Razon, Inc. in the South Harbor,
protected right to self-organization. That right "would be diluted if in the Port Area, Manila as well as bona fide members of the Associated Port Checkers
choice of the officials to govern ... (union) affairs, the election is not fairly and Workers Union. Meanwhile, the defendants are the officers of the union, who,
and honestly conducted," and the labor officers concerned and the respectively, are the president (for more than twenty years), treasurer, vice-
president and auditor of the union.
courts have the duty "to see to it that no abuse is committed by any
official of a labor organization in the conduct of its affairs. The reason for this case are the unauthorized increase in their monthly union
contributions, and deduction on their midyear bonus as contribution to the union.
These increases and deduction happened in the years 1972, 1973, 1975, and 1976.
Also, E. Razon remitted the employees’ profit share through the union which the
2) NO. union failed to distribute completely. These actions was lead by the union president,
Rica Manalad. Manalad also allowed disbursements of retirement funds to some
· The very article relied upon militates against the proposition. It states
members without the concurrence of the required number of members. While this
that a report of a violation of rights and conditions of membership in a case was pending, Manalad and the union treasurer, Leano, presented to the court
labor organization may be made by "(a)t least thirty percent (30%) of all that their actions were already ratified by 90% of the members, the med arbiter
68
however, though this case is already moot still chose to look in the case since these and records of the Associated Port Checkers and Workers Union and to submit a
are violations of the constitutional by laws of the union. report on such examination within a reasonable time.
(3) We declare that the five petitioners are entitled to a refund of the union dues
As the case progress, it was discovered that Manalad had other union memberships illegally collected from them. The Director of Labor Relations is ordered to require
and offices assumed in other unions and labor organization which she had the union to make the refund within twenty days from notice to his counsel of the
questionable transactions with using their union. With the given reasons, the Med entry of judgment in this case. Costs against the private respondents.
Arbiter ordered the removal of the private respondents as officers of the union and
directed them to reimburse to the members thereof the amounts illegally collected
from them. 8. Union Chartering and Affiliation
a. Nature and Purpose
On appeal with the Director of Labor Relations, it was reversed. The Director held
that resort to intra-union remedies is not necessary and that the five complainants
have the rights and personality to institute the proceedings for the removal of the Philippine Skylanders, Inc. v. National Labor Relations Commission, G.R. No.
respondents, to recover the amount illegally collected or withheld from them and to 127374, January 31, 2002.
question illegal disbursements and expenditure of union funds. FACTS:
Nov 1993, the Philippine Skylanders Employees Association (PSEA), a local labor
However, the Director ruled that the power to remove the union officers rests in the union affiliated with the Philippine Association of Free Labor Unions (PAFLU)
members and that the Bureau of Labor Relations generally has nothing to do with September (PAFLU), won in the certification election conducted among the rank and
the tenure of union officers which "is a political question". The Director further ruled file employees of Philippine Skylanders, Inc. (PSI). Its rival union, Philippine
that his office has jurisdiction to look into the charge of illegal disbursements of union Skylanders Employees Association-WATU (PSEA-WATU) immediately protested
funds. He directed the Labor Organization Division of the Bureau to examine the the result of the election before the Secretary of Labor.
books of account and financial records of the union and to submit a report on such
examination. This decision was affirmed by the Under Secretary of Labor. Several months later, pending settlement of the controversy, PSEA sent PAFLU a
notice of disaffiliation citing as reason PAFLU's supposed deliberate and habitual
ISSUE: Whether or not the Director of Labor Relations is correct in reversing the dereliction of duty toward its members. PSEA subsequently affiliated itself with the
decision of the Med Arbiter? National Congress of Workers (NCW), changed its name to Philippine Skylanders
Employees Association - National Congress of Workers (PSEANCW), and allowed
HELD: the former officers of PSEA-PAFLU to continue occupying their positions as elected
No, the SC held that the Labor Arbiter did not err in removing the respondents as officers PSEA-NCW.
union officers. The membership of Manalad and Puerto in another union is a On 17 March 1994 PSEA-NCW entered into a collective bargaining agreement with
sufficient ground for their removal under the constitution and by-laws of the union. PSI which was immediately registered with DOLE.
In Manalad's case, his organization of a family-owned corporation competing with.
the union headed by him renders it untenable that he should remain as union PAFLU Secretary General Serafin Ayroso wrote Mariles C. Romulo requesting a
president.The petitioners are entitled to the refund of the union dues illegally copy of PSI's audited financial statement. Ayroso explained that with the dismissal
collected from them. The union should be the proper refund.bThe Director of Labor of PSEA-WATU's election protestthe time was ripe for the parties to enter into a
Relations erred in holding that, as a matter of policy, the tenure of union office being collective bargaining agreement. PSI through its personnel manager Francisco
a "political question is, generally, a matter outside his Bureau's jurisdiction and Dakila denied the request citing as reason PSEA's disaffiliation from PAFLU and its
should be pa upon by the union members themselves. subsequent affiliation with NCW. PAFLU through Serafin Ayroso filed a complaint
for unfair labor practice against PSI, its president Mariles Romulo and personnel
WHEREFORE, (1) that portion of the decision of the med-arbiter, removing manager Francisco Dakila. PAFLU amended its complaint by including the elected
respondents Manalad, Leano and Puerto as union officers, is affirmed. (Respondent officers of PSEA-PAFLU as additional party respondents.
Amparo is no longer an officer of the union.)
(2) We also affirm that portion of the decision of the Director of Labor Relations, Labor Arbiter declared PSEA's disaffiliation from PAFLU invalid and held PSI,
directing the Bureau's Labor Organization Division to examine the books of accounts PSEA-PAFLU and their respective officers guilty of unfair labor practice.
69
by its national president, Atty. Lerum, against Filipino Pipe and Foundry Corporation
NLRC upheld the Decision of the Labor Arbiter and conjectured that since an (Fil Pipe), alleging union busting and non implementation of the Collective
election protest questioning PSEA-PAFLU's certification as the sole and exclusive Bargaining Agreement. A conciliation conference was set, but before the conciliation
bargaining agent was pending resolution before the Secretary of Labor, PSEA could
was done, the FPWU-NLU staged a strike.
not validly separate from PAFLU, join another national federation and subsequently
enter into a collective bargaining agreement with its employer-company. Fil Pipe filed petition to declare the strike illegal and for damages against FPWU-
NLU, NLU-TUCP and Lerum. Later, Fil Pipe moved to dismiss the complaint against
ISSUE: Whether or not PSEA, which is an independent and separate local union, FPWU-NLU.
may validly disaffiliate from PAFLU pending the settlement of an election protest Labor Arbiter declared the strike illegal, and ordered NLU-TUCP to pay damages.
questioning its status as the sole and exclusive bargaining agent of PSI's rank and Lerum was absolved. Appeals were filed. NLRC set aside decision for lack of merit
file employees. in so far as NLU-TUCP and Lerum is concerned.
Filipino Pipe: NLRC erred in declaring that NLU-TUCP and Lerum are not primarily
HELD:
YES. The pendency of an election protest involving both the mother federation and responsible, and that the company has lost its cause of action. They argued that the
the local union did not constitute a bar to a valid disaffiliation. responsibility for the damages allegedly sustained by petitioner company on account
Reasoning In Liberty Cotton Mills Workers Union vs. Liberty Cotton Mills, Inc. the of the illegal strike, should be borne by NLU-TUCP and Lerum, for having directly
SC upheld the right of local unions to separate from their mother federation on the participated in aiding and abetting the illegal strike. Also, that FPWUNLU is a mere
ground that as separate and voluntary associations, local unions do not owe their agent of respondent NLU-TUCP, because FPWUNLU, a local union, cannot act as
creation and existence to the national federation to which they are affiliated but, the principal of respondent NLUTUCP, a mother federation, because it is not a
instead, to the will of their members. The sole essence of affiliation is to increase,
legitimate labor organization.
by collective action, the common bargaining power of local unions for the effective
enhancement and protection of their interests.
ISSUES:
Yet the local unions remain the basic units of association, free to serve their own 1. Whether or not the strike was illegal
interests subject to the restraints imposed by the constitution and by-laws of the 2. Whether or not NLU-TUCP and Lerum are primarily responsible
national federation, and free also to renounce the affiliation upon the terms laid down
in the agreement which brought such affiliation into existence.
HELD:
Policy considerations dictate that in weighing the claims of a local union as against 1. Yes, 1st, the strike staged by FPWU-NLU was baseless. 2nd, the union failed to
those of a national federation, those of the former must be preferred. Parenthetically serve the company a copy of the notice of strike (see Sec 3 Rule XXII, Book V,
though, the desires of the mother federation to protect its locals are not altogether Implementing Rules). 3rd, the strike blatantly disregarded the prohibition on the
to be shunned. It will however be to err greatly against the Constitution if the desires doing of any act which may impede or disrupt the conciliation proceedings (see Sec
of the federation would be favored over those of its members. If it were otherwise, 6)
instead of protection, there would be disregard and neglect of the lowly workingmen. 2. No, The mother union, acting for and in behalf of its affiliate, had the status of an
Disposition Petition is granted.
agent while the local union remained the basic unit of the association, free to serve
the common interest of all its members subject only to the restraints imposed by the
constitution and by-laws of the association. The same is true even if the local union
Filipino Pipe and Foundry Corporation v. National Labor Relations
is not a legitimate labor organization
Commission, G.R. No. 115180, November 16, 1999.
The direct and primary responsibility for the damages fall on the local union FPWU,
FACTS:
being the principal, and not on respondent NLUTUCP, a mere agent of FPWU-NLU
- National Labor Union-Trade Union Congress of the Philippines (NLU-TUCP) filed
which assisted the latter in filing the notice of strike. Being just an agent, the notice
w/ the Ministry of Labor and Employment, in behalf of its local chapter, the Filipino
of strike filed by Lerum is deemed to have been filed by its principal, the FPWU-
Pipe Workers Union-National Labor Union (FPWU-NLU), a notice of strike signed
NLU. Having thus dismissed the claim for damages against the principal, FPWU-
70
NLU, the action for damages against its agent, respondent NLU-TUCP, and Atty. 1.) WON the disaffiliation of SAMANA BAY from ANGLO was valid
Lerum, has no more leg to stand on and should also be dismissed.
Disposition Petition is dismissed. 2.) WON the unilateral ousting by ANGLO of officers and board members
of SAMANA BAY was valid
ALLIANCE OF NACIONALIST AND GENUINE LABOR ORGANIZATION
(ANGLO-KMU) vs. SAMAHAN NG MGA MANGGAGAWANG NAGKAKAISA SA Ruling:
MANILA BAY SPINNING MILLS AT J.P. COATS (SAMANA BAY), G.R. NO.
118562 July 5, 1996 1.) YES.
The Court will not bend the principle that all employees enjoy the right to self-
organization and to form and join labor organizations of their own choosing for the
Doctrine: purpose of collective bargaining due to a mere procedural defect. The law is not
Disaffiliation of a local union from its mother union is valid even if certain without exceptions. Settled is the rule that a local union has the right to disaffiliate
procedural requirements for a valid disaffiliation was not complied with, provided the from its mother union when circumstances warrant (Volkshel
general membership of the local union ratified the disaffiliation action. Labor Union v. Bureau of Labor Relations)
GENERAL RULE: a labor union may disaffiliate from its mother union to form a local/
independent union only during the 60-day freedom period immediately preceding
Facts: the expiration of a CBA.
Petitioner Alliance of Nationalist and Genuine Labor Organization (ANGLO for
short) is a duly registered labor organization. Respondent union Samahan Ng mga EXCEPTION: Even before the onset of the freedom period, disaffiliation may be
Manggagawang Nagkakaisa sa Manila Bay Spinning Mills and J.P. Coats (SAMANA carried out when there is a shift of allegiance on the part of the majority of the
BAY for short) is its affiliate. In representation of SAMANA BAY, ANGLO entered members of the union. (Associated Workers Union-PTGWO vs. NLRC) In this case,
and concluded a CBA with Manila Bay Spinning Mills and J.P. Coats Manila Bay Inc. there was a unanimous confirmation by the members of SAMANA BAY of the
(hereinafter referred to as the corporations) on November 1, 1991. In December disaffiliation - clearly a shift of allegiance of the majority (if not all) of the union
1993, SAMANA BAY decided to disaffiliate from ANGLO because of the latter's members.
dereliction of duty in promoting and advancing SAMANA BAY's welfare. There were
also alleged cases of corruption involving ANGLO officers. The disaffiliation was 2.) NO.
confirmed by the members of SAMANA BAY unanimously. Apparently, ANGLO The mere act of disaffiliation does not divest the local union of its personality and
furnished the corporations copies of SAMANA BAY's neither does this give the mother federation any license to act independently of the
resolution to disaffiliate. The corporations refused to honor the disaffiliation. In local union. A local union does not owe its existence to the mother federation with
response to this, a petition to stop remittance of federation dues to ANGLO was filed which it is affiliated. There is only a contract of agency wherein the mother
by SAMANA BAY with the Bureau of Labor Relations. ANGLO counter-acted by federation acts in representation of its affiliate local union. A local labor union is a
unseating all officers and board members of SAMANA separate and distinct unit, primarily designed to secure and maintain an equality of
BAY and appointing in their stead new officers who were duly recognized by the bargaining power between the employer and their employee-members. By SAMANA
corporations. ANGLO submitted a position paper with the Med-Arbiter, contending BAY'S disaffiliation from ANGLO, the vinculum that previously bond the two entities
that the disaffiliation was void considering that a CBA was still existing and the was completely severed.
freedom period has not set in. The Med-Arbiter resolved that the disaffiliation was
void, but upheld the illegality of the ousting by ANGLO of the SAMANA BAY officers. Therefore, ANGLO was divested of any and all power to act in representation of
Both parties appealed to the DOLE. The DOLE reversed the Med-Arbiter's decision SAMANA BAY. Hence, any act by ANGLO affecting SAMANA BAY, including the
and ultimately ruled for SAMANA BAY. ANGLO's MR to the DOLE was denied hence ouster of its officers, is without force and effect.
it filed a petition for certiorari with the Supreme Court.

Issues: Progressive Development Corporation v. The Honorable Secretary,


Department of Labor and Employment, G.R. No. 96425, February 4, 1992.
71
FACTS:
The controversy in this case centers on the requirements before a local or chapter
of a federation may file a petition for certification election and be certified as the sole b. Registration
and exclusive bargaining agent of the petitioner’s employees.
San Miguel Corporation Employees Union – Philippine Transport and General
Pambansang Kilusan ng Paggawa (KILUSAN) -TUCP (hereinafter referred to as
Kilusan) filed with the Department of Labor and Employment (DOLE) a petition for Workers Organization (SMCEUPTGWO) v. San Miguel Packaging Products
certification election among the rank-and-file employees of the petitioner alleging Employees Union – Pambansang Diwa ng Manggagawang Pilipino (SMPEU-
that it is a legitimate labor federation and its local chapter, Progressive Development PDMP), G.R. No. 171153, September 12, 2007.
Employees Union, was issued charter certificate. Kilusan claimed that there was no
existing collective bargaining agreement and that no other legitimate labor FACTS:
organization existed in the bargaining unit. Petitioner is the incumbent bargaining agent for the regular monthly-paid rank
Petitioner PDC filed its motion to dismiss contending that the local union failed to and file employees of the Three Divisions of San Miguel Corporation (SMC). It
comply with Rule II Section 3, Book V of the Rules Implementing the Labor Code, had been the certified bargaining agent for 20 years—from 1987 to 1997.
as amended, which requires the submission of: (a) the constitution and by-laws; (b) Respondent is registered as a chapter of Pambansang Diwa ng Manggagawang
names, addresses and list of officers and/or members; and (c) books of accounts. Pilipino (PDMP). PDMP issued a Charter Certificate to Respondent. In
compliance with registration requirements, RESPONDENT submitted the
ISSUE: Whether or not a local union need to be a Legitimate Labor Union on despite requisite documents to the BLR for the purpose of acquiring legal personality.
its issuance of charter certificate. Upon submission of its charter certificate and other documents, RESPONDENT
was issued by the BLR a Certificate of Creation of Local or Chapter.
RULING: PETITIONER then filed with the DOLE-NCR a petition seeking the cancellation
Yes. But while Article 257 cited by the Solicitor General directs the automatic of RESPONDENT’s registration and its dropping from the rolls of legitimate labor
conduct of a certification election in an unorganized establishment, it also requires organizations. PETITIONER alleged:
that the petition for certification election must be filed by a legitimate labor a. That RESPONDENT committed fraud and falsification and non-
organization. Article 242 enumerates the exclusive rights of a legitimate labor compliance to registration requirement; and
organization among which is the right to be certified as the exclusive representative b. That PDMP is not a legitimate labor organization, but a trade
of all the employees in an appropriate collective bargaining unit for purposes of union center, hence, it cannot directly create a local or chapter.
collective bargaining. DOLE-NCR dismissed PETITIONER’s petition and ruled that RESPONDENT is
Meanwhile, Article 212(h) defines a legitimate labor organization as “any labor allowed to directly create a local or chapter. However, it found that
organization duly registered with the DOLE and includes any branch or local RESPONDENT did not comply with the 20% membership requirement and,
thereof.” thus, ordered the cancellation of its certificate of registration and removal from
Rule I, Section 1(j), Book V of the Implementing Rules likewise defines a legitimate the rolls of legitimate labor organizations.
labor organization as “any labor organization duly registered with the DOLE and RESPONDENT then appealed to the BLR, which declared that, as a chartered
includes any branch, local or affiliate thereof.” local union, PETITIONER is not required to submit the number of employees
and names of all its members comprising at least 20% of the employees in the
It is important to clarify the relationship between the mother union and the local bargaining unit where it seeks to operate.
union. In the case of Liberty Cotton Mills Workers Union v. Liberty Cotton Mills, Inc., Further, although PDMP is considered as a trade union center, it is a holder of
66 SCRA 512 [1975]), the Court held that the mother union, acting for and in behalf Registration Certificate issued by the BLR, giving it the status of a legitimate
of its affiliate, had the status of an agent while the local union remained the basic labor organization. Thus, PDMP can charter or create a local, such as
unit of the association, free to serve the common interest of all its members subject RESPONDENT.
only to the restraints imposed by the constitution and by-laws of the association. PETITIONER filed a Petition for Certiorari with the CA, which affirmed the
Thus, whereas in this case the petition for certification election was filed by the assailed decision and resolution.
federation which is merely an agent, the petition is deemed to be filed by the chapter,
the principal, which must be a legitimate labor organization. The chapter cannot ISSUE: Is RESPONDENT a legitimate Labor Organization, having been created via
merely rely on the legitimate status of the mother union. chartering by PDMP, which is a Trade Union Center?
72
3. Once a certificate of registration is issued to a union, its legal personality
Held: cannot be subject to collateral attack—it may be questioned only in an
No, it is not. The PDMP failed to comply with the registration independent petition for cancellation.
requirements of the Respondent to become a local or chapter. Although PDMP 4. There is no legal justification to support the conclusion that a trade union
as a trade union center is a legitimate labor organization, it has no power to directly center is allowed to directly create a local or chapter through chartering.
create a local or chapter, such as RESPONDENT. Thus, RESPONDENT cannot be 5. If the law’s intent were otherwise, the law could have so easily and
created under the more lenient requirements for chartering, but must have conveniently included “trade union centers” in identifying the labor
complied with the more stringent rules for creation and registration of an organizations allowed to charter a chapter or local; Since under the pertinent
independent union, including the 20% membership requirement. There is no staPetitioners and applicable implementing rules, the power granted to labor
legal justification to support the conclusion that a trade union center is allowed to organizations to directly create a chapter or local through chartering is given
directly create a local or chapter through chartering. to a federation or national union, then a trade union center is without
authority to charter directly.
If the law’s intent were otherwise, the law could have so easily and
conveniently included “trade union centers” in identifying the labor organizations
allowed to charter a chapter or local. Since under the pertinent status and applicable Coastal Subic Bay Terminal, Inc. v. Department of Labor and Employment –
implementing rules, the power granted to labor organizations to directly create a Office of The Secretary, G.R. No. 157117, November 20, 2006.
chapter or local through chartering is given to a federation or national union, then a
trade union center is without authority to charter directly.
FACTS: Herein private respondents Coastal Subic Bay Terminal, Inc. Rank and File
Union (CSBTI-RFU) and Coastal Subic Bay Terminal, Inc. Supervisory Union
Disposition: Petition GRANTED. RESPONDENT’s Certificate of Registration
(CSBTI-SU) filed separate petitions for certification election before the Med-Arbiter
CANCELLED.
of Regional Office No. III. CSBTI-RFU insists that it is a legitimate labor organization
issued a charter certificate by the Associated Labor Union (ALU) and CSBTI-SU by
I. Definition
the Associated Professional, Supervisory, Office and Technical Employees Union
A legitimate labor organization is defined as “any labor organization duly registered
(APSOTEU). They also alleged that the establishment they sought to operate in was
with the Department of Labor and Employment, and includes any branch or local
unorganized. Petitioner Coastal Subic Bay Terminal, Inc. opposed both petitions
thereof.” Legitimate labor organizations have exclusive rights under the law which
alleging that CSBTI-RFU and CSBTI-SU were not legitimate labor organizations and
cannot be exercised by non-legitimate unions, one of which is the right to be
the proposed bargaining units were not particularly described.
certified as the exclusive representative of all the employees in an appropriate
collective bargaining unit for purposes of collective bargaining.
The Med-Arbiter dismissed both petitions, holding that ALU and APSOTEU are one
II. Registration
and the same federation having a common set of officers. Thus, the two respondent
1. The procedure for registration of a local or chapter of a labor organization
unions were in effect affiliated with only one federation. Both parties appealed to the
is provided in the Implementing Rules, which enunciates a two-fold
Secretary of Labor, who reversed the said decision stating that they have separate
procedure for the creation of a chapter or a local.
legal personalities to file their separate petitions for certification election. APSOTEU
a. The first involves the affiliation of an independent union with a
is also a legitimate labor organization because it was properly registered pursuant
federation or national union or industry union.
to the 1989 Revised Rules and Regulations implementing Republic Act No. 6715.
b. The second involves the direct creation of a local or a chapter
ALU and APSOTEU are also separate and distinct labor unions having separate
through the process of chartering.
certificates of registrations from DOLE. They have different sets of locals. The two
2. The intent of the law in imposing less requirements in the case of a branch
respondent unions are legitimate labor organizations chartered respectively by ALU
or local of a registered federation or national union is to encourage the
and APSOTEU. Thus, the Secretary ordered the holding of separate certification
affiliation of a local union with a federation or national union in order to
election, with the following choices:
increase the local union’s bargaining powers respecting terms and
conditions of labor.
I. For all rank and file employees of CSBTI:

73
1. COASTAL SUBIC BAY TERMINAL, INC. RANK-AND-FILE was Section 2, Rule Ii, Book V of the Implementing Rules. Thus, APSOTEU’s
UNION-ALU- certificate of registration is valid.
TUCP; and
2. NO UNION. Now with the issue: The answer is NO. Once a labor union attains the status of
II. For all supervisory employees of CSBTI: a legitimate labor organization, it continues as such until its certificate of
1. COASTAL SUBIC BAY TERMINAL, INC. SUPERVISORY registration is cancelled or revoked in an independent action for cancellation.
EMPLOYEES The legal personality of a labor organization cannot be collaterally attacked.
UNION-APSOTEU; and Thus, when the personality of the labor organization is questioned in the same
2. NO UNION. manner the veil of corporate fiction is pierced, the action partakes the nature of a
collateral attack. Hence, in the absence of any independent action for cancellation
The latest payroll of the employer, including its payrolls for the last three months of registration against either APSOTEU or ALU, and unless and until their
immediately preceding the issuance of this decision, shall be the basis for registrations are cancelled, each continues to possess a separate legal personality.
determining the qualified list of voters. The CSBTI-RFU and CSBTI-SU are therefore affiliated with distinct and separate
On appeal, the Court of Appeals affirmed the Secretary. Hence, this instant petition. federations, despite the commonalities of APSOTEU and ALU.

ISSUES: Can the supervisory and rank-and-file unions file separate petitions for Under the Implementing Rules, a chartered local union acquires legal personality
certification election? through the charter certificate issued by a duly registered federation or national
union, and reported to the Regional Office in accordance with the rules implementing
HELD: First, with respect to APSOTEU’s status: Petitioner argues that APSOTEU the Labor Code. A local union does not owe its existence to the federation with which
improperly secured its registration from DOLE Regional Director and not from the it is affiliated. It is a separate and distinct voluntary association owing its creation to
BLR. Under Article 235 of the Labor Code, “applications for registration shall be the will of its members. Mere affiliation does not divest the local union of its own
acted upon by the Bureau, which means the BLR and/or the Labor Relations Division personality, neither does it give the mother federation the license to act
in the Regional Offices of the Department of Labor.” independently of the local union. It only gives rise to a contract of agency, where the
Section 2, Rule II, Book V of the 1989 Revised Implementing Rules of the former acts in representation of the latter. Hence, local unions are considered
Labor Code further provides that “any national labor organization or labor federation principals while the federation is deemed to be merely their agent. As such
or local union may file an application for registration with the Bureau or the Regional principals, the unions are entitled to exercise the rights and privileges of a legitimate
Office where the applicant’s principal offices is located.” labor organization, including the right to seek certification as the sole and exclusive
Section 1, Rule III of Book V of the Implementing Rules as amended by bargaining agent in the appropriate employer unit.
Department Order No. 9 also provides that “the application for registration of any
federation, national or industry union or trade union center shall be filed with the The purpose of affiliation of the local unions into a common enterprise is to
Bureau. Where the application is filed with the Regional Office, the same shall be increase the collective bargaining power in respect of the terms and
immediately forwarded to the Bureau…The application for registration of an conditions of labor. When there is commingling of officers of a rank-and-file
independent union shall be filed with and acted upon by the Regional Office where union with a supervisory union, the constitutional policy on labor is
the applicant’s principal office is located.” circumvented. Labor organizations should ensure the freedom of employees to
DOLE issued Department Order No. 40-03 further amending Book V of the organize themselves for the purpose of leveling the bargaining process but also to
above implementing rules, which provides that “applications for registration of labor ensure the freedom of workingmen and to keep open the corridor of opportunity to
organizations shall be filed either with the Regional Office or with the BLR” enable them to do it for themselves.
Even after the amendments, the rules did not divest the Regional
Office and the BLR of their jurisdiction over applications for registration by Under Article 245 of the Labor Code, supervisory employees are not eligible for
labor organizations. The amendments to the implementing rules merely membership in a labor union of rank-and-file employees. The supervisory
specified that when the application was filed with the Regional Office, the employees are allowed to form their own union but they are not allowed to join the
application would be acted upon by the BLR. rank-and-file union because of potential conflicts of interest. Further, to avoid a
In case at bar, APSOTEU’s principal office is in Diliman, Quezon City. Its situation where supervisors would merge with the rank-and-file or where the
registration was filed with the NCR Regional Office. At that time, the law applicable supervisors’ labor union would represent conflicting interests, a local
74
supervisors’ union should not be allowed to affiliate with the national labor organization fully entitled to all the rights and privileges granted by law
federation of unions of rank-and-file employees where that federation actively including the right to file a petition for certification election.
participates in the union activity within the company. Thus, the limitation is not
confined to a case of supervisors wanting to join a rank-and-file union. The ISSUE: Should the petition for certification election be granted?
prohibition extends to a supervisors’ local union applying for membership in a
national federation the members of which include local unions of rank-and-file HELD:
employees. In De La Salle University Medical Center and College of Medicine v. Yes. The petitioner bank failed to show that their Cashiers, Accountant, and Acting
Laguesma, we reiterated the rule that for the prohibition to apply, it is not enough Chief of the Loans Department possessed managerial powers and duties. At best
that the supervisory union and the rank-and-file union are affiliated with a single they only had recommendatory powers subject to evaluation, review, and final
federation. In addition, the supervisors must have direct authority over the rank-and- decision by the bank's management. The job description forms submitted by
file employees. petitioner clearly show that the union members in question may not transfer,
suspend, lay-off, recall, discharge, assign, or discipline employees. Moreover, the
In the instant case, the national federations that exist as separate entities to which forms also do not indicate that said officers could formulate and execute
the rank-and-file and supervisory unions are separately affiliated with, do have a management policies which are normally expected of management officers. Neither
common set of officers. In addition, APSOTEU, the supervisory federation, actively could the union members be treated as confidential employees. Although the
participates in the CSBTI-SU while ALU, the rank-and-file federation, actively cashier serves the bank's management, it could not be deemed to have access to
participates in the CSBTI-RFU, giving occasion to possible conflicts of interest confidential information specifically relating to SRBI's labor relations policies.
among the common officers of the federation of rank-and-file and the federation of
supervisory unions. For as long as they are affiliated with the APSOTEU and ALU, As regards the issue on the violation of the principle of separation of unions, records
the supervisory and rank-and-file unions both do not meet the criteria to attain the show that respondent union was initially assisted by ALU during its preliminary
status of legitimate labor organizations, and thus could not separately petition for stages. A local union maintains its separate personality despite affiliation with a
certification elections. larger national federation. APSOTEU-TUCP had separate legal personality from
ALU and TUCP.
Thus, petition is granted.
The law frowns on a union where the membership is composed of both supervisors
and rank-and-file employees, for fear that conflicts of interest may arise in the areas
Sugbuanon Rural Bank, Inc. v. Laguesma, G.R. No. 116194, February 2, 2000. of discipline, collective bargaining, and strikes. However, in the present case, none
of the members of the respondent union came from the rank-and-file employees of
FACTS:
the bank.
Petitioner is a duly-registered banking institution in Cebu, while private respondent
APSOTEU-TUCP was a labor organization duly-registered with the Labor
Department. APSOTEU filed a petition for certification election of the 5 supervisory Filipino Pipe and Foundry Corporation v. National Labor Relations
employees. SRBI filed a motion to dismiss the union's petition on two grounds. First, Commission, G.R. No. 115180, November 16, 1999. (supra, but different topic)
that the members of union were in fact managerial or confidential employees and
were to be disqualified from forming or joining unions. Second, the Association of Progressive Development Corporation v. The Honorable Secretary,
Labor Unions-TUCP was representing the union. Since ALU-TUCP also sought to Department of Labor and Employment, G.R. No. 96425, February 4, 1992
represent the rank-and-file employees of SRBI, there was a violation of the principle
(supra. but different topic na )
of separation of unions. The union argued that its members were not managerial
employees but merely supervisory employees.
The Med-Arbiter denied petitioner's motion to dismiss. The same was denied on c. Supervisory and Rank-and-File Union Affiliation
appeal to the Secretary of Labor and Employment. Thus, the certification election • Articles 255 and 256, Labor Code.
was ordered. Holy Child Catholic School v. Sto. Tomas, G.R. No. 179146, July 23, 2013.
Subsequently, the petition reached DOLE Regional Office seeking the cancellation De La Salle Unviersity Medical Center and College of Medicine v. Laguesma,
of the respondent union's registration. Respondent DOLE Undersecretary denied G.R. No. 102084, August 12, 1998.
SRBI's appeal for lack of merit. He ruled that APSOTEU-TUCP was a legitimate
75

You might also like